You are on page 1of 110

BLOC 2.

GEOMETRIA

7# Vectors

En context (pàg. 165) 4. Per trobar les components del vector u a la base {v ,w } , hem
a) Resposta oberta a manera de reflexió individual que pot d’assegurar que els vectors que formen la base són lineal-
servir d’introducció als vectors. ment independents i, a continuació, expressar el vector u
com a combinació lineal de v i w . Per a veure això, s’ha de
b) En la pàgina web següent, es pot trobar informació sobre
vectors biològics: comprovar que la relació a u + b v = 0 només es compleix
si a i b són tots dos iguals a 0. Vegem-ho:
http://links.edebe.com/zfcz3w
0 = a + 2b
a (1, 0 ) + b ( 2, 6 ) = 0 a = 0, b = 0
0 = 6b
Internet (pàg. 167)
— L’equipolència de dos vectors es demostra unint-ne els orí- Com que l’única solució és a 0ib 0, els vectors w i v for-
gens i els extrems respectius. Si el polígon resultant és un men una base.
paral·lelogram, els vectors són equipolents.
Figura 1 Ara, expressem el vector u com a combinació lineal de u i v .

Problemes resolts (pàgs. 175 a 177) u = a v +b w ( 2, 3 ) = a (1, 0 ) + b ( 2, 6 )


2 = a + 2b 3 1 1
1. a = 1,b = = u = 1,
Y 3 = 6b 6 2 2
2000

1600 5. Per comprovar que els vectors w i v no formen una base hem
de veure si són linealment dependents. Per a veure això, s’ha
1200
de comprovar que la relació a u + b v = 0 es compleix si a i
2000 m
800 b són no nuls. Vegem-ho:

400 0=a 2b
= 135º  a (1,1) + b ( 2, 2 ) = 0 a = 2b
0=a 2b
–400 O 400 800 1200 1600 2000 X
Com que això es compleix per a qualsevol valor de a i de b,
El desplaçament resultant és la suma dels vectors que repre- aleshores els vectors w i v no formen una base de V 2.
senten cadascun dels sentits de moviment d’en Joan i en
Pere. El mòdul dels recorreguts és de 1 200 m i 2 000 m; 6. Dos vectors són perpendiculars si el seu producte escalar és
aleshores, la suma va des de l’origen del primer vector fins a zero. Així, calculem el producte escalar dels vectors u i v .
l’extrem del segon vector i té un valor de 3 200 m.
u v = ( 1,k ) (12 + k,k ) = 12 k + k 2 = 0
2. Sigui B(x, y) l’extrem del vector AB , aleshores: k = 3, k = 4

AB = ( 3, 1) B A = ( x, y ) (1, 2 ) = ( 3, 1)
7. a) Dos vectors són perpendiculars si el seu producte escalar
x 1= 3 és zero.
x = 2, y = 1 B( 2,1)
y 2= 1
u v = ( 3, 4 ) ( 5,k ) = 15 4k = 0 k = 15 4
3. Per a veure si aquests punts formen un paral·lelogram, hem
de veure si els costats oposats tenen la mateixa longitud. És a b) Dos vectors són paral·lels si es compleix que:
dir, els costats AB i CD han de tenir el mateix mòdul, i els
u1 u2 3 4 20
costats AC i BD, també. Vegem-ho: = = k =
v1 v2 5 k 3
AB = B A = ( 0, 4 ) ( 0, 0 ) = ( 0, 4 ) AB = 02 + 42 = 4
c) Els dos vectors formen un angle de 180º si es compleix el
CD = D C = ( 2, 6 ) ( 2, 3 ) = ( 0, 3 ) AB = 02 + 32 = 3 següent:
AC = C A = ( 2, 3 ) ( 0, 0 ) = ( 2, 3 ) AB = 22 + 32 = 13 u v = u v cos 180º
BD = D B = ( 2, 6 ) ( 0, 4 ) = ( 2, 2 ) AB = 22 + 22 = 8
( 3, 4 ) ( 5,k ) = 25 52 + k 2 cos 180°
Com que els mòduls no són iguals, significa que els costats 15 4k = 5 25 + k 2 ( 1) 15 4k = 5 25 + k 2
oposats no tenen la mateixa longitud; per tant, aquests punts 2
no formen un paral·lelogram. (15 4k ) = 25 ( 25 + k 2 ) k = 20 3

100
BLOC 2. GEOMETRIA > UNITAT 7. VECTORS

8. Per a calcular l’angle, hem de saber quins vectors formen 6 + c1


l’angle. Un dels vectors és AB i l’altre vector és el que té com =4
1 6 + c1 4 + c2 2
a origen el punt A i com a extrem el punt C (0, 12), per AC = B , = ( 4, 6 )
2 2 2 4 + c2
exemple, ja que aquest vector és horitzontal. = 6
2
AB = (12, 20 ) ( 18,12 ) = ( 30, 8 ) c1 = 14, c 2 = 16 C = (14, 16 )
AC = ( 0,12 ) ( 18,12 ) = (18, 0 )
12. Calculem les components del vector AB i, a partir d’elles, de-
AB AC 30 18 + 8 0 30 terminem els punts que demana l’enunciat.
cos = = =
AB AC 302 + 82 182 + 02 964
AB = B A = ( 2, 8 ) ( 2, 0 ) = ( 4, 8 )
30
= arc cos = 14,93º
964 Ara, dividim entre 4:

9. Calculem l’angle que formen els vectors: 1 1 1 1


AB = ( 4, 8 ) = 4, 8 = (1, 2 )
4 4 4 4
v w 6 8+8 4 80
cos = = = = Sumem (1, 2) a les coordenades del punt A:
v w 62 + 82 82 + 42 10 80
80 A1 ( 2, 0) (1, 2) ( 1, 2)
=
10 Per obtenir la divisió següent repetim l’operació, però ara
80 sobre les coordenades del punt A1. El punt resultant és A2
= arc cos = 26, 57° ( 1, 2) (1, 2) (0, 4). Fent una altra vegada el mateix,
10
tenim que A3 (0, 4) (1, 2) (1, 6).
Les components del vector resultant que s’aplica al vaixell és Ara, repetint una altra vegada l’operació, obtenim les coorde-
la suma dels dos vectors: nades de l’extrem B i ja hem acabat.

v + w = ( 6, 8 ) + ( 8, 4 ) = (14,12 ) 13. Per trobar les coordenades del punt B, busquem les coorde-
v +w = 142 + 122 = 18,44 nades del vector AA1 i les sumem a les coordenades del punt
A2.

10. Per calcular la força que s’aplicarà


Figura 15 a la barcassa, calculem la
AA1 = A1 A = (1, 0 )
( 1, 3 ) = ( 2, 3 )
força vertical i la força horitzontal aplicada.
B = A2 + AA1 = ( 3, 3 ) + ( 2, 3 ) = ( 5, 6 )
Considerem el dibuix següent:

Y
Exercicis i problemes (pàgs. 178 a 180)

F = 40
1 VECTORS Pàg. 178

14. Dos vectors són equipolents si tenen el mateix mòdul, la ma-


= 45º  teixa direcció i el mateix sentit. Aleshores, els vectors següents
Figura 2
són equipolents:
O X
= 30º 
Y
F = 55

2
F1 = 40 cos 45° = 40 = 20 2
2 v
Fh = F1 + F2 = 75,91
55 3
F2 = 55 cos ( 30° ) =
2
2
F 1 = 40 sin 45° = 40 = 20 2
2 FV = F 1 + F = 0,78 O X
2
55
F2 = 55 sin ( 30° ) =
2
15. Sabent que els vectors equipolents tenen el mateix mòdul, la
FR = Fh + FV = 75,912 + 0,782 = 75,91 NW mateixa direcció i el mateix sentit, tenim que les parelles de
vectors u i t , v i w , x i t són equipolents.
11. Sigui C (c1, c2) l’extrem del segment AC , i com que B és el
punt mitjà d’aquest segment, es compleix que: 16. Els vectors u ,v , y , x tenen el mateix mòdul.

101
BLOC 2. GEOMETRIA > UNITAT 7. VECTORS

2 OPERACIONS AMB VECTORS Pàg. 178 21. a) Dibuixem dos vectors u i v qualssevol en el pla. Primera-
ment, per a la primera part de la igualtat, dibuixem 3u ,
17. La suma de dos vectors qualssevol, gràficament, consisteix a després desplacem el vector v fins a l’extrem de 3u . A
representar els dos vectors de manera que l’extrem del pri- partir d’aquí, dibuixem 3v . Unim l’origen de 3u amb l’ex-
mer vector coincideixi amb l’origen del segon vector. Alesho- trem de 3v i obtenim el vector 3u 3v . Per a la segona
res, el vector suma d’aquests dos vectors té com a origen part de la igualtat, dibuixem u i desplacem el vector v fins
l’origen del primer vector i l’extrem és l’extrem del segon a l’extrem de u . Després, unim l’origen de u amb l’extrem
vector.
(u v6).
de u v i dibuixem 3 ·Figura
Tenint en compte això, podem descartar les opcions a) i b), ja
que no coincideixen amb el que s’ha explicat anteriorment. Y

D’altra banda, l’opció d) tampoc no pot ser la correcta, ja que 3v


3(u + v )
la resta consisteix a fer l’oposat del vector v, en aquest cas.
v 2v
Per tant, la resposta correcta és l’opció c), que, efectivament,
coincideix amb l’explicació de la representació de suma de 3u + 3v v
vectors.
Figura 3
2(u + v )
3u
u

18. Y
2u
u+v
v

u
u
O X
v

u
b) Dibuixem dos vectors u i v qualssevol en el pla. Primera-
u+v
u u ment, per a la primera part de la igualtat, dibuixem u ,
u–v després desplacem el vector v fins a l’extrem de u .
Unim l’origen de u amb l’extrem de v , i obtenim el vec-
v v tor u -v . Per a la segona part de la igualtat, dibuixem v i
desplacem el vector u fins a l’extrem de v . Després,
v amb
unim l’origen deFigura 7 l’extrem de u i fem el seu oposat
O X per obtenir –(v -u ).

3 COMBINACIÓ
Figura 4
LINEAL
DE VECTORS Pàg. 178 v –u +v

v–u –(v – u )
19. a) Y

u
u –v

u–v
v
O X
2u + v
2u
22. Per veure si el vector u és combinació lineal dels vectors v i w
u hem de veure si existeixen dos nombres reals a i b tal que es
compleixi que u = a v + b w . Per tant:
Figura 5 3 = 3a + 4b
O X
( 3, 4 ) = a ( 3, 3) + b ( 4, 6 )
4 = 3a + 6b
b) Y
1 7 1 7
a= ,b = u = v+ w
–v
15 10 15 10
3u

– 2v Per tant, el vector u es pot expressar com a combinació lineal


2u
dels vectors v i w .
u
3u – 2v
4 BASES DE V2 Pàg. 178
O X
23. Siguin O = ( 0, 0 ), i = (1, 0 ), j = ( 0,1), aleshores:
a) Ov = 9 (1, 0 ) + 4 ( 0,1) = 9i + 4 j
1 3
20. u = 2 v 3 w u+3 w =2 v v = u+ w
2 2 b) Ov = 7 (1, 0 ) + 8 ( 0,1) = 7i + 8 j

102
BLOC 2. GEOMETRIA > UNITAT 7. VECTORS

7 7 c) 2 w u =2 (1,1) (1, 3 ) = ( 2 1, 2 + 3 ) = (1, 5 )


c) Ov = (1, 0 ) 7 ( 0,1) = i 7j
3 3
d) 4 v +u 2 w = 4 ( 2, 1) + (1, 3 ) 2 (1,1) =
24. Vegem si hi ha dos nombres reals a i b tals que:
= ( 8 + 1 2, 4 3 2 ) = ( 9, 1)
t = a v +b w

5 = a + 2b 29. a) 5 ( x, y ) + ( 3, 9 ) 2 ( 6, 8 ) + ( 11,10 ) = ( 0, 0 )
( 5, 1) = a (1, 3 ) + b ( 2, 2 )
1 = 3a 2b
5x + 3 12 11 = 0 5x 20 = 0
a = 1, b = 2
5y 9 16 + 10 = 0 5y 15 = 0
x = 4, y = 3
25. Per a comprovar que els vectors u i v formen una base, hem
de veure si són linealment independents. Per a veure això, b) 2 ( 3, 7 ) + 6 ( x, 2 ) (13, y ) = 2 ( x, y ) + ( 91, 49 )
s’ha de comprovar que la relació a u + b v = 0 només es
compleix si a i b són tots dos iguals a 0. Vegem-ho: 6 + 6x 13 = 2x 91 8x + 72 = 0
14 12 y = 2y 49 3y + 51 = 0
0 = 3a + 9b x = 9, y = 17
a ( 3, 4 ) + b ( 9, 4 ) = 0 a = 0, b = 0
0 = 4a + 4b

Com que l’única solució és a 0ib 0, els vectors u i v for- 21 5 = 16


30. 7 ( 3, k ) + ( 5, 5 ) = (16, 26 )
men una base. 7k 5 = 26
16 = 16
26. Per a comprovar que els vectors u i v formen una base, hem
de veure si són linealment independents. Per a veure això, k =3
s’ha de comprovar que la relació a u + b v = 0 només es
compleix si a i b són tots dos iguals a 0. Vegem-ho: 31. w + v = ( 7, 2 ) ( x, y ) + ( 10, 8 ) = ( 7, 2 )
x 10 = 7
0 = a +b x = 17, y = 6 w = (17, 6 )
a (1,1) + b (1, 1) = 0 a = 0, b = 0 y +8=2
0=a b

Com que l’única solució és a 0ib 0, els vectors u i v for- 32. u = 2 v 3 w u =2 ( 3i + 6 j ) 3 ( 7i 3j =


)
men una base.
= 6i + 12j 21i + 9j = 27i + 21j
Ara, expressem el vector t com a combinació lineal de u i v .
33. Siguin A1 = (a,b), A2 = (c,d), aleshores:
t = a u +b v ( 4, 0 ) = a (1,1) + b (1, 1)
4 = a +b 1 1
a = 2, b = 2 t =2 u+2 v AB = AA1 ( 9, 3 ) = ( a 1,b 3 )
0=a b 3 3
3=a 1
( 3,1) = ( a 1,b 3 )
1=b 3
Figura 8
a = 4, b = 4 A1 = ( 4, 4 )
5 OPERACIONS AMB
COMPONENTS Pàgs. 178 i 179 2 AA1 = AA2 2 ( 3,1) = (c 1,d 3 )
6=c 1
27. a) ( 6, 2 ) = (c 1,d 3 )
Y 2=d 3
B
4
c = 7, d = 5 A2 = ( 7, 5 )
C 3

D
2 6 PRODUCTE ESCALAR DE DOS
A+B+C+D
1
A VECTORS Pàg. 179

–3 –2 –1 O 1 2 X 34. v = ( 5 )2 + 122 = 169 = 13

b) A + B + C + D = (1,1) + ( 2, 3 ) + ( 2, 1) + ( 3, 1) = 35. v w = v 1 w 1 + v 2 w 2 = 5 8 + 12 15 = 140


= (1 2 2 + 3,1 + 3 1 1) = ( 0, 2 )
36. Calculem la projecció del vector u sobre el vector v:
28. a) u + v + w = (1, 3 ) + ( 2, 1) + (1,1) = u v 4 5 + 3 12 56
Projv u = = = = 4,31
v 52 + 122 13
= (1 + 2 + 1, 3 1 + 1) = ( 4, 3 )

b) 2 v u w =2
( 2, 1) (1, 3 ) (1,1) = v w 3 8 + 4 15
37. cos = = =
= ( 4 1 1, 2 + 3 1) = ( 2, 0 ) v w 2
( 3 ) + 42 82 + 152
36 36
= =
25 289 85
103
BLOC 2. GEOMETRIA > UNITAT 7. VECTORS

v w 3 8 + 4 15
cos = = =
v w ( 3 ) + 42
2
82 + 152
36 36 36 v w 16 4 + 8 ( 2)
= = = arc cos = 64,94° 44. cos = = =
25 289 85 Figura 9 85 v w 2 2
( 16 ) + 82 42 + ( 2 )
Y
80 80
= = = 1 = arc cos ( 1) = 180°
14 320 20 80
12
45. Amb les condicions de l’enunciat, tenim que
10
u =8 u12 + u22 = 8 u12 + u22 = 64
8
w
3
6 u1 v 2 = v 1 u2 v1 = u1
u1 u2 4 4
4 = = 3
v1 v2 3 3u2 = 4v 2 v2 = u2 3
v 2 4 v2 = u2
= 64,94º  4
3 3
–6 –4 –2 O 2 4 6 8 X u v = u1 v 1 + u2 v 2 = u1 u1 + u2 u2 =
4 4
3 3
38. ( 2 u ) ( 3 v ) = (2 ( 2, 5 ) ) ( 3 ( 5, 7 ) ) = = (u12 + u22 ) = ( 64 ) = 48
4 4
= ( 4,10 ) (15, 21)10
Figura = 4 15 + 10 ( 21) = 270

v w 4 7 + ( 3) x
39. a) Y 46. cos 60° = = =
v w 42 + ( 3)
2
72 + x 2

4000 km
28 3x 1 28 3x
= =
5 49 + x2 2 5 49 + x 2
5 49 + x 2 = 56 6x x = 2,99
3000 km

2000 km
7 COORDENADES D’UN PUNT
3000 km
O X EN EL PLA Pàgs. 179 i 180

47. El vector AB té com a origen el punt A i com a extrem el punt


b) La distància que ha de recórrer el vaixell fins al lloc on es
B. Aleshores, l’oposat d’aquest vector tindrà com a origen el
troba la balena és de 6 000 km, és a dir, és la suma de
punt B i com a extrem el punt A. Per tant, el vector oposat a
2 000 km i 4 000 km dels quilòmetres recorreguts per la
AB és determinat per:
balena cap al nord.
BA = A B = ( 7, 4 ) ( 8, 7 ) = (15, 11)
40. El mòdul del vector (20, x) és 101; per tant:
( 20, x ) = 101 202 + x 2 = 101 400 + x 2 = 10 201 48. El vector l’origen del qual és el punt (2, 3) i l’extrem és el
punt (7, 9), té com a components:
x 2 = 9 801 x = 9 801 = 99
(7, 9) (2, 3) (5, 12)

41. Amb les condicions de l’enunciat, tenim que 49. a) AB = B A= ( 2,1) (1, 3 ) = ( 3, 2 )
v w 9 9 5 b) BA = A B = (1, 3 ) ( 2,1) = ( 3, 2 )
cos = = =
v w 5 5 25
c) DC = C D = ( 3,1) ( 1, 2 ) = ( 4, 1)
9 5
= arc cos = 36,39° d) CA = A C = (1, 3 ) ( 3,1) = ( 2, 2 )
25

50. Calculem el vector que formen el punt on es troba el vehicle


42. Sabem que el mòdul del vector v és 13; per tant: avariat i el punt on està situada la instal·lació del servei d’as-
sistència mecànica. Després, en calculem el mòdul i aquesta
v = 12 k 2 + 122 = 13 k 2 + 144 = 169 k 2 = 25
distància és la que busquem.
k = 5; k = 5
( 12, 140) (120, 110) ( 132, 30)

43. Per convertir el vector en unitari, calculem el mòdul del vector ( 132, 30 ) = 1322 + 302 = 18 324 = 135,37
i dividim cada component entre el mòdul.
Aleshores, la distància entre el vehicle avariat i el servei d’as-
v = 82 + 152 = 289 = 17 sistència mecànica és de 135,37 km.

v1 v2 8 15 51. Sigui B (x, y), aleshores:


u = , = ,
v v 17 17 AB = B A = ( x, y ) (13, 6 ) = ( x 13, y 6)

104
BLOC 2. GEOMETRIA > UNITAT 7. VECTORS

Com que aquest vector ha de ser equipolent al vector u, ales- ta l’equació següent: y – x 7. D’altra banda, el mòdul del
hores s’ha de complir que: vector és 73, i resulta aquesta equació:
x 13 = 4 x 2 + y 2 = 73 x 2 + y 2 = 5 329
AB = u ( x 13, y 6 ) = ( 4, 9 )
y 6= 9
Resolem el sistema que resulta de les dues equacions ante-
x = 17
B = (17, 3 ) riors:
y = 3
x 2 + y 2 = 5 329
x = 48, y = 55
y x =7
52. a) Dos vectors són ortogonals si el seu producte escalar és
zero. Per tant:
56. Per a comprovar que tots els punts pertanyen a la circumfe-
w u=0 w u=x ( 4 ) + 12 ( 3 ) = 4x 36 = 0 rència de centre (3, 1), hem de verificar que els mòduls dels
vectors formats pels punts i el centre donen el mateix resultat.
4x = 36 x = 9 w = ( 9,12 )
En aquest cas, el mòdul és el radi de la circumferència. Ve-
gem això, en què E (3, 1):
b) Calculem el mòdul dels dos vectors:
2 AE = E A = ( 3,1) ( 7, 4 ) = ( 4, 3 ) AE =
w = ( 9 ) + 122 = 225 = 15
2 2
u =
2 2
( 4 ) + ( 3 ) = 25 = 5 ( 4 ) + ( 3) = 5
=
BE = E B = ( 3,1) ( 2,1) = ( 5, 0 )
c) Per transformar el vector w en unitari, calculem el mòdul
BE = 52 + 02 = 5
del vector i dividim cada component entre el mòdul.
CE = E C = ( 3,1) ( 6, 3 ) = ( 3, 4 )
w1 w 2 9 12 3 4
w = 15 v = , = , = , 2
w w 15 15 5 5 CE = ( 3 ) + 42 = 5
DE = E D = ( 3,1) ( 7, 2 ) = ( 4, 3 )
53. El vector de desplaçament real de la barca és la suma de la DE =
2
( 4 ) + 32 = 5
direcció de la barca i de l’orientació del corrent. Aleshores,
resulta el vector següent: (15, 7) ( 3, 4) (12, 11). Per tant, tots els punts pertanyen a la circumferència de radi
Calculem el mòdul d’aquest vector: 5.

(12,11) = 122 + 112 = 265 = 16,28 57. Per a saber quin tipus de triangle forma els punts A, B i C,
hem de calcular el mòdul dels vectors. És a dir:

54. Començarem calculant el vèrtex C. Com que els vèrtexs han AB = B A = ( 3, 2 ) (1, 3 ) = ( 2, 1)
de formar un quadrat, els vectors AB i AC han de ser perpen- 2
AB = 22 + ( 1) = 5
diculars. D’altra banda, tots els costats del quadrat han de te-
nir la mateixa longitud. Així que: BC = C B = ( 4, 5 ) ( 3, 2 ) = (1, 3 )
AB = ( 2, 3 ) ( 5, 4 ) = ( 3, 7 ) BC = 12 + 32 = 10
AC = ( x, y ) (c1,c2 ) ( 5, 4 ) = (c1 + 5,c2 + 4 ) = ( x, y ) AC = C A = ( 4, 5 ) (1, 3 ) = ( 3, 2 )
AB AC = 0 ( 3, 7 ) ( x, y ) = 0 3x + 7y = 0 AC = 32 + 22 = 13
7y 7
x = k AC = k ,1 Com que els mòduls són tots diferents, significa que el
3 3 triangle format pels punts A, B y C és escalè.
AB = 32 + 72 = 58 k AC = 58
2
7 58
k
3
+ 12 = k
9
= 58 k = 3 SÍNTESI
Figura 11 Pàg. 180

( x, y ) = ( 7, 3 ) (c1 + 5,c2 + 4 ) = ( 7, 3 )
58. a) Y
c1 + 5 = 7 c1 = 2
C = ( 2, 7 )
c2 + 4 = 3 c2 = 7

Per calcular el vèrtex D, fem el següent:

AB = CD B A=D C D =B A +C
D = ( 2, 3 ) ( 5, 4 ) + ( 2, 7 ) = ( 5, 0 )
u

55. A partir de l’enunciat, sabem que la diferència entre la segona O X


component del vector i la primera és igual a 7; per tant, resul-

105
BLOC 2. GEOMETRIA > UNITAT 7. VECTORS
Figura 12

b) Y AB = B A = ( 5, 7 ) ( 0, x ) = ( 5, 7 x )
2
6 AB = 13 52 + ( 7 x) = x2 14x + 74 = 13
x = 5
4

2
63. Per saber quin helicòpter arribarà primer a la posició del ca-
mió, calculem la distància a la qual es troba el camió de cada
helicòpter i la de valor més petit serà la de l’helicòpter que
O Figura 13
–4 –2 2 X arribi primer. És a dir, calculem els mòduls dels vectors AB i
AC :
c) Y
AB = B A= ( 21,100 ) (14,140 ) = ( 35, 40 )
18
2 2
16
AB = ( 35 ) + ( 40 ) = 2 825 = 53,15
14
AC = C A = ( 40, 73 ) (14,140 ) = ( 26, 67 )
2
B
12 AC = 262 + ( 67 ) = 5 165 = 71,87
10
Com que el mòdul entre el punt A i el punt B és més petit,
8 aleshores l’helicòpter que arribarà primer a la posició on es
6 troba el camió és el que està situat en el punt B.
4
64. Aquests punts es poden unir en una sola carretera recta si els
2 tres punts estan alineats. Per veure això, calculem els vectors
CA i CB , i comprovem si són proporcionals.
–12 –10 –8 –6 –4 –2 O 2 X
CA = A C = (12, 21)
( 3, 9 ) = ( 9,12 ) 9 12
59. Dos vectors són perpendiculars si el seu producte escalar és CB = B C = (17, 23 ) ( 3, 9 ) = (14,12 ) 14 12
zero; per tant, hem de trobar un vector u tal que:
Com que la igualtat anterior no es compleix, els tres punts no
u v =0 ( x, y ) ( 4, 3 ) = 0 4x + 3y = 0 es poden unir en una sola carretera recta.
3
x = y
4 65. a) w = 53 282 + x 2 = 53 x = 45
3 b) w u = 44
u = y, y ( 28, x ) ( 5, 3 ) = 44
4
140 + 3x = 44 x = 32

60. Com que el triangle ha de ser rectangle, ha de complir que els c) ( 28, x ) ( 3, 12 ) = 0 84 12x = 0 x =7

vectors BA i AC siguin perpendiculars. Aleshores: 66. Per a veure si aquests punts formen un trapezi, hem d’obser-
var si hi ha dos costats oposats que siguin paral·lels. Per a
BA AC = 0 ( 3 b1, b2 ) ( 2, 2 ) = 0 3 = b1 + b2
això, calculem els vectors dels punts que formin costats que
siguin oposats:
61. Mitjançant un dibuix, podem veure clarament que aquests
dos vehicles no xocaran mai, ja que amb el sentit en què van AB = B A = ( 3, 5 )
(1,1) = ( 2, 4 )
tots dos no s’arribaran a trobar. Si situem aquests punts en un DC = C D = (10, 6 ) ( 7, 1) = ( 3, 7 )
eix de coordenades, observem que el punt d’intersecció entre
les rectes determinades per cada
Figuravector
16 director que segueix 2 4
cada vehicle es troba en el quart quadrant; per tant mai no Com que , els costats formats per aquests punts no
3 7
xocaran. són paral·lels. Ara, fem el mateix per a l’altre parell de costats
oposats:
Y
60 BC = C B = (10, 6 )
( 3, 5 ) = ( 7,1)
B
v AD = D A = ( 7, 1) (1,1) = ( 6, 2 )
C
40
7 1
Com que , aquests costats tampoc no són paral·lels.
6 2
20 Per tant, tenim que aquests punts no formen un trapezi.
u
A
3x + 2x = 14
X
67. a) 3 ( x, 2 ) + ( 2x, 6 ) = (14, 0 )
O 20 40
6 6=0
x = 14
62. La distància entre dos punts consisteix a calcular el mòdul del
vector que formen aquests dos punts. b) 2 ( 3x, 9 ) + 3 ( 3x,12 ) ( 3, 6 ) = ( 15,12 )

106
BLOC 2. GEOMETRIA > UNITAT 7. VECTORS

6x 9x + 3 = 15 3x = 18 0 = 2a 3b
x =6 a ( 2, 5 ) + b ( 3, 4 ) = 0 a = 0, b = 0
18 + 36 6 = 12 12 = 12 0 = 5a + 4b

68. Classifiquem el triangle segons els seus angles a partir de les Com que l’única solució és a 0ib 0, els vectors u i v
condicions següents: formen una base.

2 2 2
Ara, expressem el vector w com a combinació lineal de u i
AC < AB + BC Acutangle v.
2 2 2
AC = AB + BC Rectangle w = a u +b v ( 5,12 ) = a ( 2, 5 ) + b ( 3, 4 )
2 2 2 5 = 2a 3b 56 1
AC > AB + BC Obtusangle ,b =
a=
12 = 5a + 4b 23 23
Calculem aquests vectors directors i els seus mòduls i veiem 56 1
la condició que es compleix: w = u v
23 23
AB = ( 3, 0 ) ( 4, 3 ) = ( 1, 3 )
2
70. a) El centre de la circumferència a partir dels punts que en
AB = ( 1) + 32 = 10 delimiten el diàmetre s’obté calculant el punt mitjà
AC = ( 0,1) d’aquests dos punts.
( 4, 3 ) = ( 4, 4 )
2 a1 + b1 a2 + b2 2 + 4 1+ 3
AC = ( 4 ) + 42 = 32 C = , = , = ( 3, 2 )
2 2 2 2
BC = ( 0,1) ( 3, 0 ) = ( 3,1)
b) Trobem el radi de la circumferència calculant el mòdul del
2
BC = ( 3 ) + 12 = 10 vector format
Figura 14 pels punts C i A o B.
2 2 2
( 32 ) >( 10 ) +( 10 ) AC = C A = ( 3, 2 ) ( 2,1) = (1,1) AC =
32 > 10 + 10 32 > 20 = 12 + 12 = 2
Per tant, aquest triangle és obtusangle. D’altra banda, obser-
c) Y
vem que hi ha dos costats del triangle amb la mateixa longi- 4
tud i un de diferent; per tant, també es tracta d’un triangle
isòsceles. B
3

a) Hem calculat la longitud dels costats del triangle anterior- C


2
ment per classificar el triangle fent els mòduls dels vectors.
A
1
AB = 10 , AC = 32 , BC = 10
O 1 2 3 4 5 X
b) AB + AC = ( 1, 3 ) + ( 4, 4 ) = ( 5, 7 )
c) AB AC = ( 1, 3 ) ( 4, 4 ) = ( 3, 1) 71. a) Els vectors OK i OJ són perpendiculars; per tant, el seu
producte escalar és zero. És a dir, OK OJ = 0.
69. a) 2 u + 3 v 5 w =2 ( 2, 5 ) + 3 ( 3, 4 ) 5 ( 5,12 ) = b) Els vectors KJ i IJ formen un angle de 45º. Per tant:
= ( 4,10 ) + ( 9,12 ) + ( 25, 60 ) = ( 30, 38 )
KJ IJ = KJ IJ cos 45° = 12 12 cos 45° = 101,82
b) 2 u ( 3 v) = 2 ( 2, 5 ) ( 3 ( 3, 4 ) ) =
c) Els vectors OL i OJ formen un angle de 180º. D’altra ban-
= ( 4,10 ) ( 9, 12 ) = 4 9 10 12 = 84 da, hem de calcular el valor dels costats OJ i OL. Per a
v w 3 5 + 4 12 33 això, utilitzem el teorema del cosinus:
c) cos = = =
v w ( 3) +
2
42 52 + 122 65 JL2 = JK 2 + LK 2 2 JK LK cos 135°
33 JL2 = 122 + 122 2 12 12 cos 135° JL = 22,17
= arc cos = 59, 5°
65 22,17
OJ = OL = = 11,09
2
d) Per transformar el vector v en unitari, calculem el mòdul
OJ OL = OJ OL cos 180° =
del vector i dividim cada component entre el mòdul.
= 11,09 11,09 cos 180° = 122,99
v1 v 2 3 4
v =5 t = , = ,
v v 5 5
Avaluació (pàg. 182)
e) Per a comprovar que els vectors u i v formen una base,
hem de veure si són linealment independents. Per a veu- 1. a) BA = A B = ( 3, 7 ) ( 5, 4 ) = ( 8,11)
re això, s’ha de comprovar que la relació a u + b v = 0
només es compleix si a i b són tots dos iguals a 0. Alesho- b) AB = B A = ( 5, 4 ) ( 3, 7 ) = ( 8, 11)
res: ( 8,11) = ( 8, 11) BA = AB

107
BLOC 2. GEOMETRIA > UNITAT 7. VECTORS

a1 + b1 a2 + b2 3+5 7 4 8. Primerament, calculem els vectors que formen el triangle:


c) PM = , = , = (1, 3 2 )
2 2 2 2 AB = B A = ( 3,10 )
( 6, 5 ) = ( 3, 5 )
d) 2 AB = 2 ( 8, 11) = (16, 22 ) BC = C B = (1, 2 )
( 3,10 ) = ( 2, 8 )
AC = C A = (1, 2 ) ( 6, 5 ) = ( 5, 3 )
2. Sigui C (x, y).
Ara, calculem els angles formats per aquests vectors:
a) AC = u C A=u ( x 3, y 7 ) = ( 4, 5 )
x 3=4 cos A =
AB AC
=
( 3) ( 5 ) + 5 ( 3) =0
x = 7, y = 2 C = ( 7, 2 ) AB AC 2
( 3 ) + 52
2
( 5 ) + ( 3)
2
y 7= 5
A = arc cos ( 0 ) = 90°
b) AC + 2 BC = 3 u ( x 3, y 7) + 2 ( x + 2, y + 3 ) =
cosC =
BC AC
=
( 2) ( 5 ) + ( 8 ) ( 3) =
x 3 + 2x + 4 = 12 3x = 11 BC AC (
2 2
2) + ( 8 )
2
( 5 ) + ( 3)
2
= (12, 15 )
y 7 + 2y + 6 = 15 3y = 14 34
=
11 14 11 14 2 312
x = ,y = C = ,
3 3 3 3
34
C = arc cos = 45°
2 312
3. AB = B A= ( 2,14 ) ( 14, 9 ) = (12, 5 )
B = 180° A C = 180° 90° 45° = 45°
AB = 122 + 52 = 169 = 13
9. Dos vectors són perpendiculars si el seu producte escalar és
4. Sigui w = (x, y ), aleshores: zero. Per tant:

w + 3 v = (1, 4 ) ( x, y ) + 3 ( 8, 6 ) = (1, 4 ) ( 5, x ) ( 9,15 ) = 0 5 ( 9 ) + x 15 = 45 + 15x = 0


x + 24 = 1 15x = 45 x =3
x = 23, y = 22 w = ( 23, 22 )
y 18 = 4
10. a) v w = ( 12, 5 ) (15, 8 ) = ( 12 ) 15 + 5 8 = 140
5. w 3 v = ( 3, 5 ) 3 (1, 4 ) = ( 3, 5 ) ( 3,12 ) = b) cos =
v w
=
4 12 + ( 6 ) 8
=
= 3 3 + 5 12 = 51 v w 42 + ( 6 )
2
122 + 82
0
= =0
6. t = a v +b w ( 5, 6 ) = a ( 1, 2 ) + b ( 2, 3 ) 52 208
5 = a + 2b = arc cos ( 0 ) = 90°
a = 3, b = 4
6 = 2a 3b
c) v w = ( 3, 4 ) (1, 1) = 3 1 + ( 4 ) ( 1) = 7
2
7. a) ( 40, 9 ) = ( 40 ) + 92 = 1681 = 41 v w 0 3 + ( 2) 4 8
d) cos = = = =
v w 02 + ( 2)
2
32 + 42 2 5
b) cos =
( 40, 9 ) ( 3, 7 ) = 40 3 + 9 7
=
( 40, 9 ) ( 3, 7 ) 2
( 40 ) + 92 32 + 72 8 8
= = arc cos = 143,13°
10 10
57 57
= = arc cos = 100, 52°
41 58 41 58 11. Per transformar aquests vectors en vectors unitaris, calculem
el mòdul del vector i dividim cada component entre el mòdul.
c) ( 40, 9 ) (1,10 ) = 40 1 + 9 10 = 40 + 90 = 50 2
a) (15, 8 ) = 152 + ( 8 ) = 289 = 17
d) Primerament, normalitzem el vector (–40, 9) calculant-ne
el mòdul i dividint cada component entre aquest. 15 8
v = ,
17 17
2
( 40, 9 ) = ( 40 ) + 92 = 1681 = 41 2 3 4
40 9 b) ( 3, 4 ) = 32 + ( 4 ) = 25 = 5 v = ,
v = , 5 5
41 41
4 0
c) ( 4, 0 ) = 42 + 02 = 4 v = , = (1, 0 )
Com que aquest vector és unitari, té mòdul 1. Volem que el 4 4
mòdul d’aquest vector sigui dos, aleshores, ho multipli-
2 2 3
quem tot per dos: d) ( 2, 3 ) = 22 + ( 3 ) = 13 v = ,
13 13
40 9 80 18
2 v =2 , = , 2 4 7
41 41 41 41 e) ( 4, 7 ) = ( 4 ) + 72 = 65 v = ,
65 65

108
BLOC 2. GEOMETRIA > UNITAT 7. VECTORS

2 2
f) ( 5, 3 ) = ( 5 ) + ( 3 ) = 34 1 + c1
=0
1 1 + c1 2 + c2 2
5 3 AC = M , = ( 0, 2 )
v = , 2 2 2 2 + c2
34 34 =2
2
c1 = 1, c 2 = 6 C = ( 1, 6 )
12. a) u v = ( 3, 4 ) ( 5,12 ) = 3 ( 5 ) + ( 4 ) 12 = 63 6 + d1
=0
1 6 + d1 1 + d2 2
2 BD = M , = ( 0, 2 )
b) u = 32 + ( 4 ) = 25 = 5; 2 2 2 1 + d2
=2
2 2
v = ( 5 ) + 122 = 169 = 13
d1 = 6, d 2 = 3 D = ( 6, 3 )
u v 63 63
c) cos = = =
u v 5 13 65 Zona (pàg. 183)
63 — Els vectors i l’animació audiovisual
= arc cos = 165,75°
65 t L’interès principal dels gràfics vectorials és poder ampliar la
mida d’una imatge a voluntat sense la pèrdua de qualitat que
13. SiguinC (c1, c2) i D (d1, d2) els altres dos vèrtexs del paral- es produeix en els mapes de bits.
lelogram. Busquem el punt mitjà de les dues diagonals del t Una part de la resposta a aquesta pregunta es pot trobar en la
paral·lelogram, que són el punt M: pàgina web que s’esmenta en l’enunciat de l’apartat anterior.

109
BLOQUE
BLOC 2. 2.
GEOMETRIA
GEOMETRÍA

8# Ecuaciones, DEL
ELEMENTS sistemas
PLA e inecuaciones
3x + 2y 2a + 3 = 0
M = AC r =
2x 3y + 7 = 0
6a 23 4a + 15
M = ,
13 13
En context (pàg. 185) 6a 23 1+c
=
a) Resposta oberta a manera de reflexió individual que pot 1 1+c a 13 2
M = (A + C) = ,
servir d’introducció als elements del pla. 2 2 2 4a + 15 a
=
b) Respostes suggerides: 13 2
c =3
— El camí més curt no és sempre el més ràpid, ja que no
a=6
depèn només de la longitud del camí sinó també de la
velocitat de la partícula. A = ( 1, 6),B = (3, 0), AC: 3x + 2y 9=0

— Significa que totes les trajectòries de la corba tenen la El valor de b per al vèrtex B pot ser qualsevol valor, ja que no
mateixa durada, independentment del punt de la corba li podem imposar cap condició; només que B pertany a la
on comencin.
recta y 0, per tant B (b, 0) i la recta AB és de la forma 6x
— El temps que emprarà una partícula a lliscar fins a arri- (b 1)y 6b 0.
bar a la posició d’equilibri estable és independent de la
posició inicial de la partícula sobre la trajectòria cicloi-
3. Hem de trobar dues altures i que es tallin.
dal.
t L’altura sobre el costat AC passa per B i té com a vector di-
rector un vector normal de la recta AC; és a dir,
Problemes resolts (pàgs. 198 i 199)
x y 6
n = (6,10) = 5x 3y + 18 = 0
1. La recta AC és la perpendicular a la recta r, per tant, passa 6 10
per A i té vector director n = (3,11) 11x + 3y = 0. El punt t L’altura sobre el costat BC passa per A i té com a vector di-
mitjà de A i C és la intersecció entre r i AC: rector un vector normal de la recta BC; és a dir,
3x 11y = 65 3 11 3 11 x+4 y 2
x = ,y = M = , n = (10, 6) = 3x 5y + 22 = 0
11x + 3y = 0 2 2 2 2 10 6
x+3 y + 11 3 11
M = , = , C = (0, 0) 5x 3y + 18 = 0 3 7
2 2 2 2 x = 3 2, y = 7 2 O = ,
3x 5y + 22 = 0 2 2
La recta s és l’altura respecte del costat AC des del vèrtex B, ja
que és paral·lela a la recta r. Com que B pertany a aquesta 4. Sigui P(x, y) un punt qualsevol del pla. La condició que resulta
recta, B (t,( 3t 37)/11). Ara imposem la condició que de l’enunciat és la següent: d(P, A)2 d(P, B)2 10
l’àrea del triangle és de 40,5 u2:
2 2

d(A,C)d(B, AC) 11t + 3 ( 3t + 37


11 ) ( (x 2)2 + (y 1)2 ) +( x 2 + (y 5)2 ) = 10
= 40, 5 130 = 81
2 130 (x 2)2 (y 1)2 10 x2 y2 2x 6y 10 0
t = 6 x = 6, y = 5 B = ( 6, 5)
5. Sabem que el tercer vèrtex és en l’eix OX; per tant, serà
La recta BC passa per B i té com a vector director BC = (6, 5) 5x + 6y = 0
d’aquesta forma: C (c, 0). Ara, calculem la base i l’altura del
BC = (6, 5) 5x + 6y = 0 i la recta AB passa per A i té com a
triangle:
vector director AB = ( 9, 6) 2x + 3y 27 = 0 .
t Base d(B,C) = (c ( 3))2 + (2 0)2 = c 2 + 6c + 13
2. El costat BC està sobre l’eix d’abscisses; per tant, B (b, 0) i
C (c, 0) i la recta BC és la recta y 0. La recta x 1 és t Altura
l’altura sobre el costat BC ja que és perpendicular a la recta
y 0, per tant A ( 1, a). Ara, busquem el punt mitjà del 2 1+ ( c 3) 2 2c 4 4c
d(A,BC) = =
costat AC de dues maneres diferents i igualem els resultats. 22 + ( (c + 3))2 c 2 + 6c + 13
La recta perpendicular a r que passa per A i que té vector di-
rector n (2, 3) és la recta AC: 3x 2y 2a 3 0. 4 4c
c 2 + 6c + 13
ba c 2 + 6c + 13
A= 10 =
3x + 2y 2a + 3 = 0 2 2
M = AC r =
2x 3y + 7 = 0 20 = 4 4c
6a 23 4a + 15 20 = 4 4c
M = , c = 6 oc = 4 C = ( 6, 0) oC = (4, 0)
13 13 20 = 4 + 4c
6a 23 1+c
=
1 1+c a 13 2
M = (A + C) = ,
2 2 2 4a + 15 a
=
110 13 2
c =3
a=6
BLOC 2. GEOMETRIA > UNITAT 8. ELEMENTS DEL PLA

Exercicis i problemes (pàgs. 200 a 204) t Diagonal BE: Passa pel punt B i té com a vector director
x y 6
BE = ( 2, 6) = 3x + y = 6 .
2 6
1 EQUACIONS DE LA RECTA Pàgs. 200 i 201 t Diagonal CE: Passa pel punt C i té com a vector director
x 4 y 4
CE = ( 6, 4) = 2x + 3y = 4.
x 0 y ( 3) x y +3 6 4
6. a) = =
2 3 2 3
b) La recta passa pel punt B i té com a vector director: 11. La recta passa per A i té com a vector director un vector nor-
mal de la recta s, és a dir,
x = 1 + 2k
BA = (4, 2) = 2 (2, 1) ,k R x 1 y 3
y =1 k n = (2, 3) = 3x 2y + 3 = 0
2 3
Aleshores,
7. t 7FDUPSJBM x, y) (0, 1) k ( 1, 2), k R 3 x y
x =0 y = ,y = 0 x = 1 + =1
x = k 2 1 3/2
t Paramètrica: ,k R
y = 1 + 2k
12. a) La recta OX és la recta y = 0 P = (2, 0), u OX = (1, 0).
x 0 y ( 1) x y +1
t Contínua: = =
1 2 1 2 La recta OY és la recta x = 0 P = (0, 2), v OY = (0,1).
x y +1 b) La bisectriu B13 és x = y P = (1,1),w B 13 = ( B, A) = (1,1) .
t General: = 2x + y + 1 = 0
1 2
La bisectriu B24 és x = y P(1,1), z B 24 = ( B, A) = ( 1,1).
x y +1
t Explícita: = 2x + y + 1 = 0 y = 2x 1
1 2 13. t $
 PTUBUAB: Passa pel punt A i té com a vector director
t Punt-pendent: x +2 y 3
AB = (6, 2) = x 3y + 11 = 0
2 6 2
m= = 2 y + 1 = 2(x 0) = 2x
1 t Costat AC: Passa pel punt A i té com a vector director
x =0 y = 1
x y x +2 y 3
t Canònica: 1 + =1 AC = (6, 5) = 5x + 6y 8=0
y =0 x = 12 1 6 5
2
t Costat BC: Passa pel punt B i té com a vector director
BC = (0, 7). Per tant, la recta BC és x 4.
8. a) Equació paramètrica P = ( 3, 0), u = (2, 1)
b) Equació contínua P = (2, 1), u = (3, 4) 5 2 4+4
t La mediana AM1, en què M1 = , = ( 4, 3 2 ) és
2 2
c) x = 2 P = (2, 0), u = ( B, A) = (0,1)
el punt mitjà del costat BC, passa per A i el seu vector director és 
d) x = 0 y =17 P = ( 0,1 7 ) , u = ( B, A) = ( 7, 2)
x +2 y 3
AM1 = (6, 3 2) = x + 4y 10 = 0 .
e) Equació punt-pendent P = ( 10, 2), u = ( 1, 8) ja que 6 32
el pendent és m 8 8/ 1.
2+4 3 2
x y t La mediana BM2, en què M 2 = , = (1,1 2 ) és
9. a) P i Q pertanyen a una equació canònica + = 1. 2 2
1 3
el punt mitjà del costat AC, passa per B i el seu vector director és
b) L’equació de la recta passa pel punt P i el seu vector direc-
x 4 y 5
tor és BM 2 = ( 3, 9 2) = 3x 2y 2 = 0
3 92
x 5 y 2
PQ = ( 4, 6) = 3x 2y 11 = 0
4 6 3+5 2+4
t La mediana CM3, en què M 3 = , = (1, 4 ) és
2 2
10. Calculem les diagonals: el punt mitjà del costat AB, passa per C i el seu vector director
t Diagonal AC: Passa pel punt A i té com a vector director x 4 y +2
és CM 3 = ( 3, 6) = 2x y + 6 = 0.
x+3 y 3 3 6
AC = (7,1) = x + 7y = 24 .
7 1
t Diagonal AD: Passa pel punt A i té com a vector director 14. a) La recta a passa pels punts A ( 8, 0) i B (0, 4) i el seu
x+3 y 3 vector director és
AD = (5, 3) = 3x + 5y = 6 .
5 3 x+8 y
AB = (8, 4) a: = x + 2y = 8
t Diagonal BD: Passa pel punt B i té com a vector director 8 4
x y 6 b) La recta b passa pels punts A ( 3, 0) i B (0, 4) i el seu
BD = (2, 6) = 3x y = 6.
2 6 vector director és:

111
BLOC 2. GEOMETRIA > UNITAT 8. ELEMENTS DEL PLA

x+3 y t La recta HI passa pel punt H i té com a vector director


AB = (3, 4) b: = 4x 3y = 12
3 4 x 12 y 8
HI = (1, 11) = 11x + y 140 = 0.
1 11
c) La recta c és y 11, que és paral·lela a l’eix X.
t La recta KL passa pel punt K i té com a vector director
d) La recta d és x 7, que és paral·lela a l’eix Y.
x 8 y 10
KL = (1, 1) = x+y 18 = 0.
e) La recta e passa pels punts A (0, 0) i B (7; 7,5) i el seu 1 1
vector director és: t La recta LM és y 9, que és paral·lela a l’eix X.
x y
AB = (7, 7,5) e: = 7,5x + 7y = 0 17. Sigui y 2 m(x 2) la recta que passa pel punt P. Com que
7 7,5
la recta talla amb els semieixos positius, tenim que:
f) La recta f passa per A ( 18, 0) i B (0, 7) i el seu vector
x =0 y = 2m + 2 A = (0, 2m + 2)
director és:
2 + 2m 2 + 2m
x + 18 y y =0 x = B = ,0
AB = (18, 7) f: = 7x 18y = 126 m m
18 7
Imposem la condició de l’enunciat:
15. Sigui y 4 m(x 3) la recta que passa pel punt A. Com que d(O, A) d(O,B) 2 + 2m
2

la recta talla els eixos de coordenades, tenim que: =9 ( 2m + 2)2 = 18


2 m
x =0 y = 3m + 4 B = (0,3m + 4) 2 + 2m
2m + 2 = 4m 2 + 10m + 4 = 0
4 3m 4 3m m
y =0 x = C = ,0 1
m m m1 = 2,m2 =
2 2
4 3m
d(B,O) = d(C,O) (3m + 4)2 = m1 : y 2 = 2(x 2) 2x + y = 6
m
m2 : y 2 = 1 2 (x 2) x + 2y = 6
4 3m
3m + 4 =
m 3m 2 + 7m + 4 = 0
18. Calculem les rectes r i s:
4 3m 3m 2 + m 4=0
3m 4= t La recta r passa per A i el seu vector director és el de la recta BC
m
m = 1,m = 43 y 4 = 1(x + 3) x+y 1= 0 x+3 y 6
BC = ( 10, 10) = x y + 9 = 0.
m = 1,m = 43 y 4 = 1(x + 3) x y +7= 0 10 10
t La recta s passa per B i té com a vector director un vector
Nota: No considerem el valor m 4/3 ja que el resultat és normal de r:
una recta que talla en l’origen de coordenades i, aleshores, la
x 13 y 8
distància a cada punt de tall seria zero. n = (1, 1) = x+y 21 = 0
1 1
16. t -BSFDUBAB és y 11, que és paral·lela a l’eix X. x y +9=0
x = 6, y = 15 D = (6,15)
x+y 21 = 0
t La recta BC passa pel punt B i té com a vector director
x 8 y 11
BC = ( 4, 4) = x y + 3 = 0. 19. Per a saber si és possible unir els punts de reg en una línia
4 4
recta, s’ha de comprovar si els punts A, B i C estan alineats.
t La recta CD és y 7, que és paral·lela a l’eix X. És a dir, si es compleix que:
b1 a1 b2 a2 5 2 1 3
t La recta DE passa pel punt D i té com a vector director = =
c1 b1 c2 b2 6,5 5 3 ( 1)
x 7 y 7
DE = ( 2, 3) = 3x 2y 7 = 0. 3 4
2 3 = 2=2
1,5 2
t La recta EJ passa pel punt E i té com a vector director
x 5 y 4 Per tant, és possible unir els tres punts en una línia recta.
EJ = (3,1) = x + 3y 7 = 0.
3 1
20. Siguin C (x, y) i D (m, n) els punts de divisió.
t La recta EF passa pel punt E i té com a vector director
x 5 y 4 2=x 9
EF = (6, 3) = x + 2y 13 = 0. 1 1
6 3 AB = AC (6, 2) = (x 9, y 1) 2
3 3 = y 1
t La recta FG passa pel punt F i té com a vector director 3
x 11 y 1 x = 11
FG = ( 2, 4) = 2x y 21 = 0.
2 4 y =53
t La recta AH passa pel punt A i té com a vector director 4=m 9
2 2
x 14 y 11 AB = AD (6, 2) = (m 9,n 1) 4
AH = ( 2, 3) = 3x 2y 20 = 0 . 3 3 =n 1
2 3 3
m = 13
n=73
112 5 7
C = 11, , D = 13,
3 3
x = 11
y =53
BLOC 2. GEOMETRIA > UNITAT 8. ELEMENTS DEL PLA
4=m 9
2 2
AB = AD (6, 2) = (m 9,n 1) 4
3 3 =n 1
3
m = 13 23. Sigui r : y 2 mx la recta que passa pel punt A i siguin B i C
els punts de tall de r amb les altres dues rectes. Aleshores:
n=73
5 7 14 2(8m 5)
C = 11, , D = 13, B =r { 5x y + 16 = 0} B = ,
3 3 m 5 m 5
14 2(4m + 1)
C =r { x + 3y = 8} C = ,
21. Calculem els vèrtexs del rombe: 3m 1 3m 1

t A = { x + 2y 16 = 0} { 3x + 4y 52 = 0} A = (4,10) Imposem que A sigui el punt mitjà del segment BC.


t La recta perpendicular a l’equació de la diagonal és una al-
14 14 2(8m 5) 2(4m + 1)
tra de les diagonals del rombe. Aquesta recta passa per Q i + +
té vector director: A= m 5 3m 1 , m 5 3m 1 = (0, 2)
2 2
x 8 y 7
n = (3, 4) = 4x 3y 11 = 0
3 4 m= 2 y 2 = 2x 2x + y = 2
B = { x + 2y 16 = 0} { 4x 3y 11 = 0}
B = (14,15)
t Q és el punt mitjà del segment AC; per tant: 2 POSICIÓ RELATIVA
4 + c1 DE DUES RECTES Pàgs. 201 i 202
=8
4 + c1 10 + c 2 2
Q = (8, 7) = ,
2 2 10 + c 2 24. u r = (3, 2), u s = (n + 1,n)
=7
2 3 2
C = (12, 4) = n=2
n +1 n
t Q és el punt mitjà del segment BD; aleshores:
25. a) u r = ( 1, 3), u s = ( 2,1) Les rectes són secants ja que
14 + d1
=8 els vectors directors no són proporcionals. Calculem el
14 + d1 15 + d 2 2
Q = (8, 7) = , punt d’intersecció P entre les dues rectes:
2 2 15 + d 2
=7
2 t =2 x
x =2 t y +1
D = (2, 1) y +1 2 x =
y = 1 + 3t t = 3
3
22. Calculem els vèrtexs del triangle: 3x + y 5=0
t MA és el punt mitjà entre B i C; aleshores: 3x + y 5=0
6 7 6 7
x +2 y 3 x = ,y = P = ,
M A = 1 2 (B + C ) = (3, 7) b1 + c1 = 6,b2 + c 2 = 14 = 5 5 5 5
2 1
t MB és el punt mitjà entre A i C; per tant:
b) u r = (2, 3), u s = ( 1, 3) Les rectes són secants perquè
MB = 1 2 ( A + C ) = (12,10) a1 + c1 = 24, a2 + c 2 = 20 els vectors directors no són proporcionals.
t MC és el punt mitjà entre A i B; per tant: 3x 2y + 1 = 0 1 1
P = x= ,y = 1 P = ,1
MC = 1 2 ( A + B ) = (7, 3) a1 + b1 = 14, a2 + b2 = 6 y = 3x + 2 3 3

Resolent aquest sistema d’equacions resulta: c) u r = ( 1, 7), u s = ( 1, 7) Les rectes són paral·leles per-
què els vectors directors són iguals.
a1 = 16,b1 = 2,c1 = 8, a2 = 0,b2 = 6,c 2 = 20
A = (16, 0),B = ( 2, 6),C = (8, 20) d) u r = (1, 1), u s = ( 2, 3) Les rectes són secants perquè
els vectors directors no són proporcionals. Calculem el
Calculem les equacions dels costats del triangle: punt d’intersecció P:
t La recta r passa per MA i té com a vector director
x = 2+t t =x 2
x = 3 + 5t r : x 2= 1 y
M AC = (5,13) ,t R y = 1 t t = 1 y
y = 7 + 13t x+y 1= 0
t La recta p passa per MB i té com a vector director x 1
t =
x =1 2t 2 x 1 y 1
x = 12 + 2t s: =
MB A = (4, 10) = 2(2, 5) ,t R y = 1 + 3t y 1 2 3
y = 10 5t t =
3
t La recta s passa per MC i té com a vector director 3x + 2y 5=0
x = 7 + 3t x+y 1= 0
MCB = ( 9, 3) = 3(3,1) ,t R y = 2, x = 3 P = ( 3, 2 )
y = 3 + 1t 3x + 2y 5=0

113
BLOC 2. GEOMETRIA > UNITAT 8. ELEMENTS DEL PLA

26. Les rectes r i s són paral·leles si BC = ( 2, 6), AC = ( 12, 6) BC AC = 60, BC = 40 ,


2 3 7 ( 3) 21 60 2
= k = = AC = 180 cos C = =
7 k 2 2 40 180 2
27. Per a saber si els dos submarins xocaran en algun moment, C = arc cos
2
= 45º
s’ha de calcular si les trajectòries que segueixen aquests sub- 2
marins es tallen o no. u1 = ( 3, 4),u2 = ( 3, 4) Els vectors
directors són iguals, de manera que les trajectòries seguides
són paral·leles i, per tant, no es tallen.
37. Sigui ur = ( 1,1) mr = 1 i ms mr el pendent de l’altra
recta. Aleshores:
28. Resposta suggerida: ms mr ms + 1
tg 45º = 1= ms = 0
t Exemple 1: Siguin r : 2x 3y 5 0 i s : 2x 3y 3 0, 1 + msmr 1 ms
com que 2/2 és diferent de 3/ 3, les rectes són secants.
L’única recta amb pendent 0 i que talla amb l’eix OX és la
t Exemple 2: Siguin r : 5x 2y 1 0 i s : 10x 4x 3 0, recta y 0.
com que 5/10 2/ 4 i diferent d’1/3, les rectes són paral-
leles. a a 1 1 a 1
38. a) = =
3a (3a + 1) 3 (3a + 1)
29. Com que l’equació és canònica, aquesta recta passa pels
punts A(2, 0) i B(0, 3). Per tant, el feix de rectes paral·leles a 1
3a 1 = 3a 3 a=
aquesta recta tindrà el mateix vector director, que és el se- 3
güent: AB = ( 2, 3) 3x + 2y + C = 0,C
b) n r = (a, a 1),u s = (3a + 1,3a)
30. u r = (5, 3),u s = (k,10) n r = ( 3, 5) i u s ,n r han de ser pro- Aquests dos vectors han de ser proporcionals; per tant,
porcionals 2 n r = 2 ( 3, 5) = ( 6,10) k = 6 s’ha de complir que a 3a 1 a 1/2. Calculem el
punt de tall per a a 1/2:
31. Equació punt-pendent: y 3 m(x 2), m Rix 2.
1 3
x y 3=0
32. u r = (2,1),u s = ( 1, 3) u r u s = 1, u r = 5 , us = 10 2 2
x = 1,5, y = 1,5
3 1 3
ur us 1 2 x+ y =0
cos = = = 2 2 2
ur us 5 10 10
P = ( 1,5, 1,5 )
2
= arc cos = 81º 52
10
3 DISTÀNCIES Pàgs. 202 i 203

33. u = AB = ( 2, 2),v = AC = (2, 1)


39. a) d(P,Q) = ( 7 5)2 + (5 0)2 = 81 + 25 = 106
u v = 6, u = 8,v = 5
b) d(R,S) = ( 2 ( 1))2 + ( 3 7)2 = 1 + 100 = 101
u v 6 3 10
cos = = =
u v 8 5 10 40. En la demostració de la unitat, s’utilitza que pel fet que PHA
és un triangle rectangle, en què P (p, q) és el punt, A (a, b)
3 10 és un punt de la recta r i H és el punt de tall entre r i la
= arc cos = 18º 26
10 seva perpendicular passant per P, es compleix que
PA n
34. u = (3, 1),v = (1, 2) u v = 1, u = 10 , v = 5 d(P, r ) = PH = PA i en la demostració d’aquesta
PA n
1 1 2 pàgina només s’utilitza la projecció d’AP (p a, q b).
cos = = =
10 5 50 10
41. Calculem el punt de tall entre r i s:
2
= arc cos = 81º 52 12
10 x +2 y +2
=
3 4
x = 14, y = 18 P = ( 14, 18)
2x + 3y 5=0 x = 4+k
x 4=y
35. A = r s = x = 1, y = 1 A = (1,1) y =k
x y =0

La recta passa pels punts A (1, 1) i B (2, 1) i el seu vector Sigui t : 4x y 4, aleshores:
x 1 y 1
director és AB = (1, 2) = 2x + y 3 = 0 A p1 + B p2 + C ( 4)( 14) + ( 1)( 18) 4
1 2 d(P,t ) = = =
A2 + B 2 ( 4)2 + ( 1)2
36. L’angle és delimitat per les rectes BC i AC; per tant, necessi- 70
= u
tem saber els vectors directors d’aquestes rectes. 17

114
BLOC 2. GEOMETRIA > UNITAT 8. ELEMENTS DEL PLA

42. Sigui P (x, y) el punt simètric de P respecte de Q. Com que — Trapezis: tenen dos costats paral·lels.
Q és el punt mitjà de P i P tenim que:
t Trapezi rectangle: té un angle recte.
3+x
=2 t Trapezi isòsceles: té dos costats no paral·lels iguals.
3+x 9+ y 2
Q = , = (2, 3) P = (7, 3)
2 2 9+ y t Trapezi escalè: no té cap costat igual ni té angle recte.
=3
2
— Trapezoides: no tenen cap costat igual ni paral·lel.

Els vectors directors dels costats del quadrilàter són


43. La recta paral·lela a y 2x 6 és de la forma y 2x k, i
passa per Q, per tant, k 2 i t: y 2x 2. AB = (4, 1),BC = ( 1, 4),CD = ( 4,1), AD = ( 1, 4); observem
que els costats són paral·lels dos a dos, ja que són proporcionals.
A p1 + B p2 + C 2 3 + 1 ( 5) 2 1 La longitud dels costats és d(A,B) = d(B,C) = d(C,D) = d(A,D) =
d(P,t ) = = =
A2 + B2 22 + 12 5
d(A,B) = d(B,C) = d(C,D) = d(A,D) = 17 , per tant, tots els costats són iguals. Com

44. Calculem la distància que hi ha entre els punts M i A, i entre que AB BC 0, hi ha un angle com a mínim que no és rec-
M i B. te; aleshores no pot ser un quadrat, així que es tracta d’un
rombe. En calculem l’àrea:
d(M, A) = (103 32)2 + (22 12)2 = 5141
d(A,C) d(B,D) 32 + 32 52 + ( 5)2
d(M,B) = (30 32)2 + (100 12)2 = 7748 A= = = 15 u 2
2 2
Com que d(M, A) d(M, B), aleshores el participant que arri-
barà primer serà el participant A. 49. La recta perpendicular a s passa per P i té vector director
7 x y 1
45. La recta r està expressada en forma vectorial; un punt que n = ( 1, 2) = 2x + y 15 = 0 . Ara, cal-
passa per aquesta recta és P (0, 1). D’altra banda, l’equació 1 2
general de la recta s és 4x 6y 2 0. culem el punt de tall entre aquesta recta i la recta s:

4 0 + 6 1+ 2 8 4 x+3
d(P, s) = = = x = 3 + 2t t =
42 + 62 52 13 s: 2 x 2y + 5 = 0
y = 1+t
t = y 1
46. Per a demostrar que els punts A, B, C i D formen un quadrat, x 2y + 5 = 0
s’ha de comprovar que tots els costats mesuren el mateix i H = x = 5, y = 5 H = (5, 5)
2x + y 15 = 0
que els costats són perpendiculars entre ells, és a dir, que es
compleix el següent: AB BC = BC CD = CD AD = 0 Sigui P (x, y) el punt simètric de P respecte de la recta s.
Com que H és el punt mitjà de P i P tenim que:
t d(A,B) = (6 0)2 + (4 9)2 = 61
7+x
t d(B,C) = (11 6)2 + (10 4)2 = 61 =5
7+x 1+ y 2
H = , = (5, 5)
t d(C,D) = (5 11)2 + (15 10)2 = 61 2 2 1+ y
=5
2
t d(D, A) = (0 5)2 + (9 15)2 = 61 x =3
P = (3, 9)
t AB = (6, 5),BC = (5, 6) AB BC = 0 y =9

t BC = (5, 6),CD = ( 6, 5) BC CD = 0 50. Els vèrtexs són les interseccions entre les rectes r, s i t.
t CD = ( 6, 5), AD = (5, 6) AB BC = 0 2x + 3y = 3
t A = r s = x = 3, y = 3 A = ( 3, 3)
6x y = 21
( 2) 3 + 4 k 1 7 + 4k
47. d(P, r ) = =4 =4
( 2)2 + 42 2 5 2x + 3y = 3
t B = r t = x = 3, y = 1 B = (3, 1)
7±8 5 2x + 7y = 13
k =
4
6x y = 21
tC = s t = C = ( 4, 3)
48. Les condicions per a cadascun dels quadrilàters són: 2x + 7y = 13
— Paral·lelograms: tenen els costats paral·lels dos a dos.
Per a calcular l’àrea, necessitem saber l’altura i la base:
t Quadrat: té els quatre costats iguals i els angles rectes.
t Base d(B,C) = ( 4 3)2 + ( 3 ( 1))2 = 53
t Rectangle: té els costats iguals dos a dos i els quatre
angles rectes. t Altura
t Rombe: té els quatre costats iguals. ( 2) ( 3) + 7 3 + 13 40
d(A,BC) = d(A,t ) = =
t Romboide: té els costats iguals dos a dos. ( 2)2 + 72 53

115
BLOC 2. d(P,C)
GEOMETRIA > UNITAT 8. ELEMENTS DEL PLA
= d(Q,C)
2 2
5y 1 2 5y 1 2
( 1) + (y 3) = 5 + (y 1)
3 3
2 2
40 5y + 2 2 5y 16 2
53 + (y 3) = + (y 1)
b a 53 3 3
A= = = 20 u 2
2 2 34y 2 34y + 85 34y 2 178y + 265 5
= y = = 1,25
9 9 4
51. Els vèrtexs són les interseccions de les quatre rectes. Obser-
5 1,25 1
vem que les rectes r i s i les rectes t i u són paral·leles; per C = ,1,25 = (1,75;1,25 )
tant, no les podem tallar. 3

2x y +3=0
t A = s u = x = 0, y = 3 A = (0, 3) 54. Una equació d’un altre costat del quadrat és la recta perpen-
y = 2x 3 + 3 dicular a x 2y
1 que passa per Q. Aquesta recta té vector
2x y +3=0 x +1 y +5
director n = ( 1, 2) = 2x y = 7.
tB = s t = B = ( 1, 5; 0) 1 2
2x + 3y + 3 = 0
t A = { x + 2y = 1} { 2x y = 7} A = ( 3, 1)
y = 2x 1
tC = r t = x = 0, y = 1 C = (0, 1) tB = { x + 2y = 14 } { 2x y = 7} B = (0, 7)
2x + 3y + 3 = 0
t El vèrtex C és la intersecció entre la recta x 2y 14 i la
y = 2x 1
tD = r u= x = 1, 5, y = 2 D = (1, 5; 2) circumferència de radi d(A,B) = 45 i centre B.
y = 2x 3 + 3
x + 2y = 14
És un paral·lelogram, ja que té els costats paral·lels dos a C = C = ( 6, 10) o C = (6, 4)
x 2 + (y + 7)2 = 45
dos. Per a calcular l’àrea, necessitem saber-ne la base i l’al-
tura: t El vèrtex D és la intersecció entre la recta x 2y 1 i la
circumferència de radi d (A, B) i centre A.
t Base d(A,D) = (1,5 0)2 + (2 3)2 = 3,25
t La recta AD passa pel punt A i té com a vector x + 2y = 1
D = D = ( 9, 4) o D = (3, 2)
x y 3 (x + 3)2 + (y + 1)2 = 45
AD = (1,5, 1) = x + 1,5y 4,5 = 0
1,5 1 Finalment, calculem la recta CD que pot ser de dues formes,
1 0 + 1,5 ( 1) 4,5 6 ja que tenim dos valors de C i D:
Altura d(C, AD) = =
1,52 + 12 3,25 t La recta CD passa per C( 6, 10) i té vector director
CD = ( 9, 4) ( 6, 10) = ( 3, 6) 2x + y + 32 = 0.
6
A=b a = 3,25 = 6 u2
3,25 t La recta CD passa per C(6, 4) i té vector director
CD = (3, 2) (6, 4) = ( 3, 6) 2x + y 8 = 0.
52. Calculem la recta AB i la longitud d’aquest costat:
55. Sigui r : y mx n. Les condicions de l’enunciat són:
t La recta AB passa per A i té com a vector director
x =0 y =2 2 = m 0+n n=2
AB = (6, 3) AB : x + 2y 13 = 0 .
y =0 x = 3 0 = 3m + n m=23
td(A,B) = (9 3)2 + (2 5)2 = 45 2
r: y = x +2
Ara, busquem els vèrtexs C i D: 3
t El vèrtex C és la intersecció entre la recta perpendicular a 5 23 1+ 2 13 3
AB (amb vector director n (1, 2)) que passa per B i la t d(C, r ) = = = 13 u
(2 3)
2
+( 1)2 13 3
circumferència de radi d(A, B) i centre B.
2x y 16 = 0 t La recta paral·lela a r és de la forma s : 2x 3y k 0 i com
C = C = (6, 4) o C = (12, 8) que té ordenada en l’origen 6, x 0 i y 6, per tant, substi-
(x 9)2 + (y 2)2 = 45
tuïm en la recta s i resulta k 18. Aleshores, la recta s
queda 2x 3y 18 0.
t El vèrtex D és la intersecció entre la recta perpendicular a
AB (amb vector director n (1, 2)) que passa per A i la cir-
cumferència de radi d(A, B) i centre A.
56. a) La recta perpendicular a r passa per A i té vector director
n = ( 1, 2) 2x + y 5 = 0 . Ara, calculem el punt de tall
2x y 1= 0 entre aquesta recta i la recta r:
D = D = (0, 1) o D = (6,11)
(x 3)2 + (y 5)2 = 45 2x + y 5=0
H = x = 1, y = 3 H = (1, 3)
x + 2y = 5
53. Sigui C (x, y) un punt qualsevol de la recta 3x 5y 1. Un
punt qualsevol de la recta és com C ((5y 1)/ 3, y). Ara, Sigui A (x, y) el punt simètric de A respecte de la recta r.
imposem la condició de l’enunciat: Com que H és el punt mitjà de A i A tenim que:

d(P,C) = d(Q,C) x 1
=1
x 1 y +7 2
5y 1
2
5y 1
2 H = , = (1, 3) A = (3, 1)
( 1) + (y 3)
2
= 5 + (y 1)
2
2 2 y +7
3 3 =3
2
2 2
5y + 2 2 5y 16 2
+ (y 3) = + (y 1)
3 3
116 34y 2 34y + 85 34y 2 178y + 265 5
= y = = 1,25
9 9 4
5 1,25 1
C = ,1,25 = (1,75;1,25 )
3
BLOC 2. GEOMETRIA > UNITAT 8. ELEMENTS DEL PLA

La recta A P passa per A i té com a vector director b) DEF és isòsceles si té dos costats iguals, i és rectangle si té
A P = (4,16) 4x y 13 = 0. El punt M de xoc serà: un angle recte:

4x y 13 = 0 31 33 d(D,E ) = ( 0,5 5)2 + ( 2,5 7)2 11


M = x = ,y =
x + 2y = 5 7 7
d(E ,F ) = (9,1 + 0,5)2 + ( 7,8 + 2,5)2 11
31 33
M = , d(F ,D) = (5 9,1)2 + (7 + 7,8)2 15
7 7

b) La recta perpendicular a y 0 (eix OX) que passa per A DE = ( 5,5; 9,5),EF = (9,6; 5,3) DE EF 0
és la recta x 3. El punt de tall entre aquestes dues rec- c) BDF és isòsceles si té dos costats iguals:
tes és H  (3, 0). Sigui A (x, y) el punt simètric de A res-
pecte d’OX i com que H és el punt mitjà de A i A tenim d(B,D) = (5 5)2 + (7 + 1)2 = 8
que:
d(D,F ) = (9,1 5)2 + ( 7,8 7)2 15
x+3
=3 d(F ,B) = (5 9,1)2 + ( 1 + 7,8)2 8
x+3 y +7 2
H = , = (3, 0) A = (3, 7)
2 2 y +7 d) PDF és equilàter si tots els seus costats són iguals:
=0
2
d(P,D) = (5 19,7)2 + (7 3)2 15
La recta A B passa per A i té com a vector director
A B = (15,12) 4x 5y 47 = 0. El punt M de xoc és d(D,F ) = (9,1 5)2 + ( 7,8 7)2 15

4x 5y 47 = 0 d(F ,P) = (19,7 9,1)2 + (3 + 7,8)2 15


M = x = 11,75, y = 0
y =0
M = (11,75; 0 )
4 LLOCS GEOMÈTRICS Pàg. 203
57. Calculem els punts E, P i F.
58. El lloc geomètric és la circumferència de centre P i radi 3, és
t El triangle ABE és equilàter; per tant: a dir, (x 2)2 (y 3)2 32 9.
d(A,B) = d(B,E ) = d(A,E ) = 32 ,E = (x, y )
59. Tot punt P (x, y) que pertanyi a la mediatriu compleix:
d(B,E ) = (x 5)2 + (y + 1)2 = 32 d(P, A) = d(P,B)
d(A,E ) = (x 1)2 + (y 3)2 = 32 (x 2)2 + (y 0)2 = (x + 1)2 + (y 4)2
E = ( 0,5; 2,5) (x 2)2 + y 2 = (x + 1)2 + (y 4)2 3x + 4y = 6,5
o
E = (6,5; 4,5) 60. Un punt P (x, y) pertany a les bisectrius si:

t La recta AC passa per A i té com a vector director x + y +1 x y +2


d(P, r ) = d(P, s) =
AC = (8, 0) y = 3 i la recta BE passa per B i té com a 12 + 12 12 + ( 1)2
vector director BE = ( 5,5; 1,5) 1,5x + 5,5y + 13 = 0 Com que dues rectes determinen quatre angles, per a trobar
en què E ( 0,5; 2,5). les dues bisectrius hem de tenir en compte els dos signes de
les arrels del denominador. Així, si designem t1 i t2 les bisec-
1,5x + 5,5y + 13 = 0
P = P = (19,7; 3) trius, obtenim dues equacionss:
y =3
1
t1: y = = 0,5
t La recta perpendicular a DC: x y 12 0 té com a vector t 1 : 2 (x + y + 1) = 2 (x y + 2) 2
director n = (1,1) x y 16, 7 = 0 i el punt de tall entre t 2 : 2 (x + y + 1) = 2 (x y + 2) 3
aquestes dues rectes és: t2: x = = 1,5
2
x y 16,7 = 0
H = H = (14,4; 2,4)
x+y 12 = 0 61. Sigui r : x 0 (eix Y) i s : y 0 (eix X). Si imposem la condició
de l’enunciat, en què P (x, y) és el punt, resulta:
Sigui F (x, y) el punt simètric de P respecte de la recta DC.
Com que H és el punt mitjà de F i P tenim que: x y
d(P,r ) 2 = 3d(P, s) 2=3 x 3y 2=0
1 1
x + 19,7 y +3
H = , = (14,4; 2,4) F = (9,1; 7,8)
2 2 62. El lloc geomètric dels punts P (x, y) del pla que formen un tri-
a) CEF és equilàter si tots els seus costats són iguals: angle isòsceles és la mediatriu, ja que la distància dels punts
a cada punt de la base és la mateixa. Per tant:
d(E ,F ) = (9,1 + 0,5)2 + ( 7,8 + 2,5)2 11
d(P, A) = d(P,B)
d(F ,C) = (9 9,1)2 + (3 + 7,8)2 11
(x 6)2 + (y + 2)2 = (x + 1)2 + (y 3)2
d(C,E ) = ( 0,5 9)2 + ( 2,5 3)2 11 (x 6)2 + (y + 2)2 = (x + 1)2 + (y 3)2 7x + 5y = 15

117
BLOC 2. GEOMETRIA > UNITAT 8. ELEMENTS DEL PLA

63. Sigui Q (x, y) el punt que compleix la condició de l’enunciat: d(P, A) = d(P,C) x 2 + (y + 3)2 = (x 0)2 + (y 0)2
x + y +1 9
d(Q,P) = d(Q, r ) (x + 1)2 + (y 1)2 = x 2 + (y + 3)2 = x 2 + y 2 y = = 1,5
12 + 12 6
(x + y + 1)2 8x + 6y = 7
(x + 1)2 + (y 1)2 =
2 Q = x =2 Q = (2, 1,5)
y = 1,5
x2 + y2 2xy + 2x 6y + 3 = 0

64. Trobem les equacions dels costats del triangle: 66. Siguin P (x, y) els punts que compleixen la condició següent:

t Costat AB: passa pel punt A i té com a vector director d(P, A)d(P,B) = 1
x +7 y +3 (x 2)2 + (y 6)2 (x 1)2 + (y + 2)2 = 1
AB = (7, 7) = x y + 4 = 0.
7 7 ( (x 2)2 + (y 6)2 ) ( (x 1)2 + (y + 2)2 ) = 1
t Costat AC: passa pel punt A i té com a vector director x4 6x 3 + 2x 2 y 2 8x 2 y + 53x 2 6xy 2 + 8xy
x +7 y +3 100x + y4 8y 3 3y 2 + 100y + 199 = 0
AC = (12, 2) = x 6y 11 = 0 .
12 2
t Costat BC: passa pel punt B i té com a vector director
67. Calculem la bisectriu de l’angle C i el costat AB:
x y 4 t Bisectriu de l’angle C:
BC = (5, 5) = x+y 4 = 0.
5 5 — Costat AC : passa pel punt A i té vector director
Ara, n’hi ha prou de trobar dues bisectrius i intersecar-les. x 5 y 10
AC = (6, 6) = x + y 15 = 0
t Un punt P(x, y) pertany a la bisectriu de A si: 6 6

x y +4 x 6y 11 — Costat BC : passa pel punt B i té vector director


d(P, AB) = d(P, AC) =
12 + ( 1)2 12 + ( 6)2 x+3 y 2
BC = (14, 2) = x 7y + 17 = 0
14 2
t 1 : 37 (x y + 4) = 2 (x 6y 11)
t Un punt P (x, y) pertany a les bisectrius si:
t 2 : 37 (x y + 4) = 2 (x 6y 11)
x+y 15 x 7y + 17
t 1 : 3,3x + 1,7y + 28,2 = 0 d(P, AC) = d(P,BC) =
t 2 : 5,3x + 10,3y 6,2 = 0 12 + 12 12 + ( 7)2
t 1 : 50 (x + y 15) = 2 (x 7y + 17)
t Un punt P(x, y) pertany a la bisectriu de B si:
t 2 : 50 (x + y 15) = 2 (x 7y + 17)
x y +4 x+y 4
d(P, AB) = d(P,BC) = t1 : x + 3y 8=0
12 + ( 1)2 12 + 12
t 2 : 3x y +1 = 0
t 1 : 2 (x y + 4) = 2 (x + y 4) t1: y = 4
t2: x = 0 t La recta del costat AB passa pel punt A i té com a vector direc-
t 2 : 2 (x y + 4) = 2 (x + y 4)
x 5 y 10
tor AB = ( 8, 8) = x y +5 = 0
5,3x + 10,3y 6,2 = 0 8 8
I = y = 0,6 I = (0; 0,6)
x =0 3x y +1 = 0
D = x = 2, y = 7 D = (2, 7)
x y +5 = 0
65. Primerament, trobem els vèrtexs del triangle:
3x 4y = 12 68. Siguin P (x, y) els punts del pla que equidisten de r i s:
t A = r { x = 0} A=
x =0 2x y +5 2x y +1
d(P, r ) = d(P, s) =
y = 3 A = (0, 3) 22 +( 1)2 22 + ( 1)2
3x 4y = 12 5(2x y + 5)2 = 5(2x y + 1)2 2x y +3=0
tB = r { y = 0} B= x =4 B = (4, 0)
y =0 Les tres rectes són paral·leles.
t Com que la recta s’interseca amb els eixos de coordenades,
el vèrtex C serà l’origen de coordenades, és a dir, C (0, 0). 69. Siguin P (x, y) el punt que equidista de les rectes a, b i c:

Ara, n’hi ha prou de trobar dues mediatrius i intersecar-les: d(P, a) = d(P,b) = d(P,c)
t Mediatriu de AB : P (x, y) pertany a aquesta mediatriu si: 6x + y 26 x+y 1 x+y 5
= =
62 + 12 12 + 12 ( 1)2 + 12
d(P, A) = d(P,B)
2(6x + y 26)2 = 37( x + y 5)2
(x 0)2 + (y+ 3)2 = (x 4)2 + (y 0)2 2(x + y 1)2 = 2( x + y 5)2
x 2 + (y + 3)2 = (x 4)2 + y 2 8x + 6y = 7 x = 1,4
P = (1,4; 3)
t Mediatriu de AC : P (x, y) pertany a aquesta mediatriu si: y =3

118
BLOC 2. GEOMETRIA > UNITAT 8. ELEMENTS DEL PLA

70. Calculem els pendents, en què Q (x, y): 4x + 3y 11 = 0 74 7


x = ,y =
y 4 3x 4y 10 = 0 25 25
t AQ = (x, y 4) mAQ = ,x 0
x 74 7
H = ,
y 1 25 25
t BQ = (x 2, y 1) mBQ =
x 2
Sigui C (x, y) el punt simètric de C respecte de la recta
1 y 4 1 y 1 AB. Com que H és el punt mitjà de C i P tenim que:
mAQ = mBQ =
4 x 4 x 2
3xy 15x 8y + 32 = 0 1+ x 5+y 74 7
H = , = ,
2 2 25 25
71. Calculem els pendents, en què Q (x, y): 173 139
C' = ,
25 25
y 2
t AQ = (x + 1, y 2) mAQ = ,x 1
x +1
e) Base d(A,B) = ( 2 2)2 + ( 4 ( 1))2 = 25 = 5
y 3
t BQ = (x 5, y 3) mBQ = ; x 5 b a 5 6,6
x 5 A= A= = 16,5 u 2
2 2
y 2 y 3
mAQ = 3mBQ =3
x +1 x 5 74. t "OHMFDFOUSBUFOF:
2xy + 7x 8y + 19 = 0
EF = (3, 7),DF = (6, 4),EF DF = 46, EF = 58 ,
72. Siguin A (x , 0), B (0, y ) i E (x, y ). Aleshores, la recta 46 23
AC és 8x x y 8x 0. Com que E pertany a la recta AC, E DF = 52 cos F = =
58 52 754
ha de ser de la forma següent: E (( x y 8x )/8), y ). Ara,
23
imposem la condició que l’àrea del trapezi és 14 u 2: F = arccos = 33º 6 '
754
x y + 8x
x +
(B + b) h 8 t Angle centrat en G:
A= = 14 y = 28
2 2
EG = (6, 3),DG = (9, 0),EG DG = 54, EG = 45 , DG = 9
xy 2 16 x y 224 = 0
54 2 2
cosG = = G = arc cos = 26º 34 '
9 45 5 5

t Angle centrat en H:
SÍNTESI Pàg. 204
EH = (4, 1),DH = (7, 2),EH DH = 26, EH = 17 ,
73. a) P (x, y) pertany a la mediatriu si d (P, A) d (P, B)
26 26
DH = 53 cos H = =
(x + 2)2 + (y + 4)2 = (x 2)2 + (y + 1)2 17 53 901
(x + 2)2 + (y + 4)2 = (x 2)2 + (y + 1)2 26
H = arc cos = 29º 58 '
4x 3y = 7,5 901

b) AB és la recta que passa per A i té com a vector director


75. a) Sigui A (x, y) el punt simètric de A respecte de B. Com
x +2 y +4 que B és el punt mitjà de A i A, tenim que:
AB = (4, 3) = 3x 4y 10 = 0 .
4 3
2+x
3 ( 1) + ( 4) 5 10 33 =0
Altura d(C, AB) = = = 6,6 u 2+x 1+ y 2
32 + 42 5 B= , = (0, 3) A ' = ( 2, 7)
2 2 1+ y
= 3
2 1 4+5 2
c) La mediana BM, en què M = , = ( 1,5; 0,5 )
2 2 b) La recta AB passa pel punt A i té vector director
2 1 4+5
= , = ( 1,5; 0,5 ) és el punt mitjà de AC, passa per B i el seu x 2 y 1
2 2 AB = ( 2, 4) = 2x y 3 = 0.
vector director és 2 4

x 2 y +1 La seva recta perpendicular passa per C i té com a vector


BM = ( 3,5;1,5) =. 1,5x 3,5y = 0,5
3,5 1,5 x 3 y +2
director n = (2, 1) = x + 2y + 1 = 0.
2 1
d) La recta perpendicular a AB passa per C i té vector director
x +1 y 5 La intersecció de la recta que uneix A i B amb la seva per-
n = (3, 4) = 4x + 3y 11 = 0 . pendicular és:
3 4
2x y 3=0
Ara, calculem el punt de tall entre aquesta recta i la recta H = x = 1, y = 1 H = (1, 1)
AB: x + 2y + 1 = 0

119
BLOC 2. GEOMETRIA > UNITAT 8. ELEMENTS DEL PLA

Sigui C (x, y) el punt simètric de C respecte de la recta b) Primera solució: Els vèrtexs A i C estaran en la mediatriu
AB. Com que H és el punt mitjà de C i C resulta que: del segment BD, que és la recta r, i a una distància de M
de 164 u . A i C seran de la forma x t, i y (30 5t )/4.
3+x
=1 Aleshores, s’ha de complir que:
3+x 2+ y 2
H = , = (1, 1)
2 2 2+ y 30 5t
2
= 1 d(M, A) = d(M,C) = 164 (2 t )2 + 5 =
2 4
C = ( 1, 0 )
= 164 t = 6,t = 10 A = ( 6,15),C = (10, 5)
D
 7FHFNTJFMTDPTUBUTEFMRVBESJMËUFSTØOUPUTJHVBMTPOP JTJ
Els vèrtexs B i D estaran en la mediatriu del segment AC,
els costats són perpendiculars entre ells:
que és la recta s, i a una distància de M de 41 u . B i D
seran de la forma x t, i y (17 4t )/ 5. Per tant:
t d(A,C) = (3 2)2 + ( 2 1)2 = 10
2
17 + 4t
d(M,B) = d(M,D) = 41 (2 t )2 + 5 =
t d(A, C ) = ( 1 2)2 + (0 1)2 = 10 5
= 41 t = 7,t = 3 B = (7, 9),D = ( 3,1)
t d(B, C ) = ( 1 0)2 + (0 + 3)2 = 10
Segona solució: S’efectua el mateix procediment, però
canviant la recta r per la s, i viceversa. El resultat és el se-
t AC = ( 3, 1), C B = (1, 3) AC C B = 0 güent: E ( 2, 10), F (12, 13), G (6, 0) i H   ( 8, 3).
c) Calculem les equacions dels costats del rombe a partir de
t C B = (1, 3),BC = (3,1) C B BC = 0 la primera solució de l’apartat b).
t La recta AB passa per A i té com a vector director
t BC = (3,1),CA = ( 1, 3) BC CA = 0 x +6 y 15
AB = (13, 6) = 6x + 13y 159 = 0
13 6
És un quadrat i A = d(B, C ) d(B,C) = 10 10 = 10 u 2 0.
t La recta BC passa per B i té com a vector director
76. a) Calculem el punt comú a aquesta família de rectes:
x 7 y 9
mx + (m 1)y + (m + 2) = 0 mx + my y +m+2 = 0 BC = (3, 14) = 14x + 3y 125 = 0.
3 14
x + y +1 = 0
m(x + y + 1) y +2= 0 P = ( 3, 2) t La recta CD passa per C i té com a vector director
y +2= 0
x 10 y +5
CD = ( 13, 6) = 6x + 13y + 5 = 0
b) Substituïm el punt P en la família de rectes: 13 6
m + 2(m 1) + (m + 2) = 0 m=0 y =2 t La recta DA passa per D i té com a vector director
x+3 y 1
c) La recta de la família que és paral·lela a r ha de complir DA = ( 3,14) = 14x + 3y + 39 = 0.
3 14
m m 1 1
que: = m= x 3y + 9 = 0.
1 3 4 79. La bisectriu del primer quadrant té com a equació y x; així,
el punt C serà de la forma (x, x).
77. Hem de calcular el punt simètric de A respecte de la recta r :
x y 3. La recta perpendicular a r passa per A i el seu vector Com que és un triangle isòsceles, el punt ha de complir
x 3 y 4 que:
director és n = ( 1,1) = x+y 7=0
1 1 d(C, A) = d(C,B) (x 2)2 + (x + 2)2 = (x 7)2 + (x 3)2
x+y = 3 (x 2)2 + (x + 2)2 = (x 7)2 + (x 3)2 x = 2,5
H = x = 5, y = 2 H = (5, 2).
x+y 7=0 C = (2,5; 2,5)

Sigui A (x, y) el punt simètric de A respecte de la recta r. Com Calculem els costats del triangle:
que H és el punt mitjà de A i A tenim que:
t El costat AB passa pel punt A i té com a vector director
3+x 4+y x =7 x 2 y +2
H = , = ( 5, 2 ) A ' = ( 7, 0 ) AB = (5, 5) = x + y = 4.
2 2 y =0 5 5

L’equació del raig reflectit és la recta A C que passa per A i el t El costat BC passa pel punt B i té com a vector director
x 7 y 3
seu vector director és A 'C = (4, 8) 2x + y = 14. BC = ( 4, 5; 0, 5) = 0, 5x 4, 5y + 10 = 0
4, 5 0, 5
0.
78. a) ur = ( 4, 5),us = ( 5, 4) ur us = 20 20 = 0 les
rectes són perpendiculars. Calculem el punt de tall: t El costat AC passa pel punt A i té com a vector director
x 2 y +2
5x + 4y = 30 AC = (0, 5; 4, 5) = 4, 5x + 0, 5y + 10 = 0
M = x = 2, y = 5 M = (2, 5) 0, 5 4, 5
4x + 5y = 17 0.

120
BLOC 2. GEOMETRIA > UNITAT 8. ELEMENTS DEL PLA

80. m 1 és el pendent de la recta x y 0, i m el pendent de 3x + 5y 9=0 4


la recta que busquem: G = x = ,y = 1
6x + 7y + 1 = 0 3
m m 3 1 m 4
tg 30º = = G = ,1
1 + mm 3 1 m 3
3+ 3
m = Aquests tres punts estan alineats, ja que es compleix que:
3+ 3
b1 a1 b2 a2 52 ( 1) 2 ( 1)
3+ 3 = = 3= 3
m = c1 b1 c2 b2 43 52 1 2
3+ 3
La recta d’Euler passarà pel punt O i tindrà com a vector di-
3+ 3
s:y 2= (x + 1) 7 x +1 y +1
3+ 3 rector OI = ,3 = 6x 7y = 1 .
2 72 3
3+ 3
t :y 2= (x + 1)
3+ 3 82. Calculem els vèrtexs oposats A i D (x, y), en què M (0, m) és
el punt de tall de les diagonals:
81. t 0SUPDFOUSF OIJ IB QSPV EF USPCBS EVFT BMUVSFT J JOUFSTF-
car-les. 5x + 14y 179 = 0
A= x = 5, y = 11 A = ( 5,11)
— L’altura sobre el costat AB passa per C i té com a vector 7x 4y + 79 = 0
director un vector normal de la recta AB, és a dir, M t 5y = 50 y = 10 M = (0,10)
x +2 y 3 1 x 5 y + 11
n = ( 2, 8) = 4x + y + 5 = 0. M = (A + D ) = , = (0,10) D = (5, 9)
2 8 2 2 2
— L’altura sobre el costat AC passa per B i té com a vector La recta r BD és paral·lela a la recta r i passa per
director un vector normal de la recta AC, és a dir, D r : 5x + 14y 101 = 0 i la recta s CD és paral·lela
x +1 y +1 a la recta s i passa per D s : 7x 4y + 1 = 0
n = (2, 9) = 9x 2y + 7 = 0.
2 9
5x + 14y 179 = 0
4x + y + 5 = 0 B = x = 9, y = 16 B = (9,16)
O = x = 1, y = 1 7x 4y + 1 = 0
9x 2y + 7 = 0
O = ( 1, 1) 7x 4y + 79 = 0
C = x = 9, y = 4 C = ( 9, 4)
5x + 14y 101 = 0
t Circumcentre: hem de trobar dues mediatrius i interse-
car-les.
83. Teorema de Napoleó: Si es construeixen tres triangles equilà-
— Un punt P (x, y) pertany a la mediatriu AB si: ters a partir dels costats d’un triangle qualsevol, a l’interior o a
l’exterior, aleshores els centres dels triangles equilàters for-
d(P, A) = d(P,B)
men també un triangle equilàter.
(x 7)2 + (y 1)2 = (x + 1)2 + (y + 1)2
t Calculem les longituds de cada costat del triangle i els nous
(x 7)2 + (y 1)2 = (x + 1)2 + (y + 1)2 4x + y = 12 vèrtexs dels nous triangles equilàters:
— Un punt P (x, y) pertany a la mediatriu AC si: — Triangle ABD:
d(P, A) = d(P,C)
d(A,B) = (0 8)2 + (6 0)2 = 100 = 10
(x 7)2 + (y 1)2 = (x + 2)2 + (y 3)2
d(B,D) = 10 x 2 + (y 6)2 = 10
(x 7)2 + (y 1)2 = (x + 2)2 + (y 3)2 D =
d(A,D) = 10 (x 8)2 + y 2 = 10
18x + 4y = 37
4x + y = 12 D = ( 1,2; 3,9) o D = (9,2; 9,9)
5
I = x = ,y = 2
18x + 4y = 37 2 Per calcular el centre d’aquest triangle, trobem dues
altures i les intersequem (per a D ( 1,2; 3,9)).
5
I = ,2
2 La recta perpendicular a AB passant per D és 8x   6y 
13,8 0 i la recta perpendicular a BD passant per A
t Baricentre: n’hi ha prou de trobar dues medianes i interse- és 1,2x 9,9y 9,6 0.
car-les.
8x 6y 13,8 = 0
— La mediana CM, en què M (3, 0) és el punt mitjà del C1 = C1 = (2,2; 0,7)
1,2x + 9,9y 9,6 = 0
costat AB, passa per C i té com a vector director
x +2 y 3 — Triangle ACE:
CM = (5, 3) = 3x + 5y 9 = 0.
5 3
d(A,C) = (3 8)2 + (11 0)2 = 146
— La mediana BM, en què M (5/2, 2) és el punt mitjà
d(A,E ) = 146 (x 8)2 + y2 = 146
del costat AC, passa per B i té com a vector director E =
7 x +1 y +1 d(C,E ) = 146 (x 3)2 + (y 11)2 = 146
BM = ,3 = 6x 7y 1 = 0.
2 72 3 E = ( 4;1,2) o E = (15; 9,8)

121
BLOC 2. GEOMETRIA > UNITAT 8. ELEMENTS DEL PLA

Per calcular el centre d’aquest triangle, trobem dues Avaluació (pàg. 206)
altures i les intersequem (per a E (15; 9,8)).
1. t 7FDUPSJBM x, y) (2, 3) k(2, 5), k R
La recta perpendicular a AC passant per E es 5x  11y 
32,8 0 i la recta perpendicular a CE passant per A x = 2 + 2k
és 12x 1,2y 96 0. t Paramètrica: ,k R
y = 3 5k

5x 11y + 32,8 = 0 x 2 y +3
C2 = C2 = (8,7; 7) t Contínua: =
12x 1,2y 96 = 0 2 5
x 2 y +3
— Triangle BCF: t General: = 5x + 2y 4=0
2 5

d(B,C) = (3 0)2 + (11 6)2 = 34 5


t Explícita: 5x + 2y 4=0 y = x +2
2
d(B,F ) = 34 x2 + (y 6)2 = 34
F = 5 5
d(C,F ) = 34 (x 3)2 + (y 11)2 = 34 t Punt-pendent: m = y +3= (x 2)
2 2
F = ( 2,8;11,1) o F = (5,8; 5,9)
x =0 y =2
t Canònica: x y
Per calcular el centre d’aquest triangle, trobem dues 4 + =1
altures i les intersequem (per a F ( 2,8; 11,1)). y =0 x = 45 2
5
La recta perpendicular a BC passant per F és 3x 5y 
47,1 0 i la recta perpendicular a BF passant per C és 2. a) La recta BC passa pel punt B i té vector director
2,8x 5,1y 47,7 0. x +1 y 6
BC = (4, 8) = 2x + y 4=0
3x + 5y 47,1 = 0 4 8
C3 = C 3 = (0,1; 9,4) x =0 y =4 x y
2,8x 5,1y + 47,7 + =1
y =0 x =2 2 4
Finalment, comprovem que el triangle C1C2C3 és equilàter;
per a això, calculem les longituds dels costats: 2 ( 4) + 1 2 4 10
b) Altura d(A,BC) = = =2 5
22 + 12 5
d(C1,C2 ) = d(C2 ,C 3 ) = d(C 3 ,C1) 9 4+3 2 2 1
c) La mediana BM, en què M = ,0 , =
2 2 2
Nota: Agafem aquests valors de D, E i F perquè són els que
és el punt mitjà de AC, passa per B i té com a vector direc-
formen els triangles exteriors al triangle ABC; si agaféssim els
1
altres valors formaríem els triangles interiors i es faria seguint 1 x = 1+ k
tor BM = , 6 2 ,k R.
el mateix procediment. 2
y = 6 6k

84. 7FJFNRVFO (0, 0) i B (8, 0) L’angle exterior d’un pentà- d) Base triangle: d(B,C) = (3 + 1)2 + ( 2 6)2 = 80 = 4 5
gon regular és (3 · 180º)/5 108º. Per mitjà del teorema del
cosinus:
d(B,C) = (3 + 1)2 + ( 2 6)2 = 80 = 4 5 A=
b a
=
( 4 5 ) (2 5 ) = 20 u 2
2 2
OC = OD = BE = 82 + 82 2 8 8 cos 108º =
= 128 + 128 cos 72º e) u AC = (7, 4),v BC = (4, 8) u AC v BC = 60, u AC = 65 ,

C = (OC cos 36º,OC s in 36º ) = (10,5; 7,6) 60 3 13


v BC = 80 cos = =
65 80 13
D = 4;( OD 2 42 ) = ( 4; 112 + 128 cos 72º ) = (4;12,3)
3 13
E = ( BE cos 36º + 8,BE s in 36º ) = ( 2,5; 7,6) = arc cos = 33º 41 24
13
1 2,5 + 4 7,6 + 12,3
H = (E + D) = , = (0,8;10)
2 2 2 3. a) ur = ( 1, 2),us = ( 4, 5) Les rectes són secants, ja que
1 4 + 10,5 12,3 + 7,3 els vectors directors no són proporcionals.
G = (D + C) = , = (7,3;10)
2 2 2
2x + y + 5 = 0
La recta BH passa per B i té com a vector director P = x +2 y 4 x = 2, y = 1 P = (2, 1)
=
BH = ( 7,2;10) 10x + 7,2y 80 = 0 i la recta OG passa per 4 5
O té com a vector director OG = (7,3;10) 10x 7,3y = 0. b) ur = (1, 2),us = (2, 4) Les rectes són paral·leles, ja que
els vectors directors són proporcionals. Calculem la distàn-
10x + 7,2y 80 = 0 cia entre elles:
F = Centre = F = (4; 5,5)
10x 7,3y = 0 x =0 y =1 P = (0,1) s

P a (5 d(O,B)) d(F ,OB) 5 8 5,5 2 0 1 1+ 7 6 6 5


A= = = = 110 u 2 d(r , s) = d(r ,P) = = =
2 2 2 22 + ( 1)2 5 5

122
BLOC 2. GEOMETRIA > UNITAT 8. ELEMENTS DEL PLA

4. a) AB = (5, 2),DC = (10, 4), AD = ( 11, 2),BC = ( 6, 4) Els t La recta AC passa per A i té com a vector director
costats AB i DC són paral·lels, ja que els seus vectors di- x+4 y 5
AC = (5, 2) = .
rectors són proporcionals, mentre que els costats AD i BC 5 2
no són paral·lels. t La recta CD és perpendicular a la recta AC, per tant, tindrà
t d(A,B) = (6 1)2 + (2 0)2 = 29 5 5
com a vector director n = (2, 5) m= y 7= (x 1
2 2
t d(A,D) = ( 10 1)2 + (2 0)2 = 125
5 5
n = (2, 5) m= y 7= (x 1).
t d(C,D) = ( 10 0)2 + (2 6)2 = 116 2 2
t d(B,C) = (0 6)2 + (6 2)2 = 52 t La recta DE passa pel punt D, que és la intersecció entre la
recta CD i la recta y 0, per tant, D (19/5, 0) i el seu vector
t AB AD = (5, 2) ( 11, 2) = 51 0
director és DE = ( 19 5 , 2 ) 10x + 19y + 38 = 0.
t AD DC = ( 11, 2) (10, 4) = 102 0
x =0 y = 2 x y
+ =1
t DC BC = (10, 4) ( 6, 4) = 44 0 y =0 x = 19 5 19 5 2
Per tant, com que dos dels seus costats són paral·lels i cap
no té la mateixa longitud ni són perpendiculars dos a dos, 6. Triangle ABC:
resulta que ABCD és un trapezi escalè.
t Base d(A,B) = ( 1 2)2 + (2 5)2 = 18
2 6 5 2 + 30 32
b) Altura a = d(B,DC) = = en
22 + ( 5)2 29 1 ( 1) + ( 1) 5 + 3 3
t Altura d(C, AB) = = en què
x + 10 y 2 12 +( 1)2 2
què DC és = 2x 5y + 30 = 0, B d (A ,B)
10 4 la recta AB és x y 3 0.
ib d(C, D).
Triangle ABM, M (x, y):
32
a(B + b) 29
( 116 + 29 ) t Base d(A,B) = 18
A= = = 48 u 2
2 2
x y +3 x y +3
x 1 y t Altura d(M, AB) = =
c) La recta AB és = 2x 5y 2 = 0. Busquem 12 +( 1)2 2
5 2
una recta perpendicular a AB, per tant, tindrà com a vector 3 x y +3
18 18
director n (2, 5); a més a més, sabem que passarà per D, 2 2
AABC = AABM =
x + 10 y 2 2 2
de manera que = 5x + 2y + 46 = 0 . El
2 5 x y +3 =3
punt de tall entre les dues rectes és el següent:
2x 5y 2=0 x = 226 29 7. Calculem les equacions dels costats del triangle:
5x + 2y + 46 = 0 y = 102 29 t La recta AB passa per A i té com a vector director
226 102 x +2 y 2
H = , AB = (2, 3) = 3x 2y + 10 = 0.
29 29 2 3

Sigui D (x, y) el punt simètric de D respecte de la recta t La recta BC passa per B i té com a vector director
AB. Com que H és el punt mitjà de D i D resulta que: x y 5
BC = (4, 6) = 3x + 2y 10 = 0.
4 6
10 + x 2+ y 226 102
H = , = , t La recta AC passa per A i té com a vector director
2 2 29 29
x +2 y 2
10 + x 226 AC = (6, 3) = x + 2y 2 = 0.
=
x = 162 29 6 3
2 29
2+ y 102 y = 262 29 t Un punt P (x, y) pertany a les bisectrius si:
=
2 29
3x 2y + 10 3x + 2y 10
D' = ( 162 29 , 262 29 ) d(P, AB) = d(P,BC) =
32 + ( 2)2 32 + 22

5. t -BSFDUBAB és la recta x 4 0, que és paral·lela a l’eix Y. t 1 : 13 (3x 2y + 10) = 13 (3x + 2y 10)

t La recta BG passa per B i té com a vector director t 2 : 13 (3x 2y + 10) = 13 (3x + 2y 10)
BG = (2, 2) (x, y ) = ( 4, 2) + (2, 2)k,k . t1 : y = 5
t La recta GF és la recta y 4, que és paral·lela a l’eix X. t2 : x = 0
x =0
t La recta FE passa per F i té com a vector director D = y =1 D = (0,1)
x =1 k x + 2y 2=0
FE = ( 1, 6) ,k
y = 4 6k
. Comprovem el teorema de la bisectriu:

123
BLOC 2. GEOMETRIA > UNITAT 8. ELEMENTS DEL PLA

d(D, A) = ( 2 0)2 + (2 7x
1)2+ =3y 5 80 = 0 575 205 575 205
x =
proporcionals ,y = B = ,
3x + 7y + 5 = 0 58 58 58 58
d(D,C) = (4 0)2 + ( 1 1)2 = 20
d(B, A) = ( 2 0)2 + (2 5)2 = 13 El punt C on es reflecteix el raig és el punt mitjà del segment
proporcionals A B , aleshores:
d(B,C) = (4 0)2 + ( 1 5)2 = 52
1 169 575 31 205 186 59
C = + , + = ,
8. Les dues condicions de l’enunciat es tradueixen en: 2 58 58 58 58 29 29

2b
a b a= 11. Busquem els costats i els vèrtexs del triangle DEF:
r s = 3
2 3
2 t La recta DE és la recta y 6, paral·lela a l’eix X i E (9, 6).
d(O, r ) = 3 =3
a2 + b 2 t La recta EF és la recta x 9, paral·lela a l’eix Y.
4 2
a= ,b = t La recta DF és la recta perpendicular a la recta BC. Un vec-
3 13 13 tor director de BC és (2, 5), per tant, un vector normal serà
(5, 2) i la recta DF serà, aleshores: 2x 5y 25 0.
9. Calculem els peus de la perpendicular des de Q:
t La recta AC passa per A i té com a vector director 2x 5y + 25 = 0 5
D = DF DE D = ,6
x +2 y 3 y =6 2
AC = (4, 4) = x + y 1 = 0 i una recta
4 4 2x 5y + 25 = 0 43
perpendicular a aquesta passa per Q i té vector director F = DF EF F = 9,
x =9 5
n = ( 1, 1) x y 1 = 0. Si intersequem aquestes dues
x+y 1= 0 d(D,E ) d(F ,E ) (9 5 2)2 (6 43 5)2 169
rectes, tenim: x = 1, y = 0 D = (1, 0). A= = = u2
x y 1= 0 2 2 20

t La recta AB passa per A i té com a vector d(D,E ) d(F ,E ) (9 5 2)2 (6 43 5)2 169
A = director = = u2
x +2 y 3 2 2 20
AB = (4, 2) = x 2y + 8 = 0 i u n a
4 2
recta perpendicular a aquesta passa per Q i té vector direc-
tor n = (1, 2) 2x + y 11 = 0 . Si intersequem aquestes Zona (pàg. 207)
dues rectes, tenim: — Impossible?
x 2y + 8 = 0 14 27 14 27 t Resposta suggerida: La geometria projectiva parteix dels
x = ,y = E = ,
2x + y 11 = 0 5 5 5 5 principis següents:

t La recta BC passa per B i té com a vector director — Dos punts defineixen una recta.
BC = (0, 6) x = 2 i una recta perpendicular a aquesta — Tot parell de rectes es talla en un punt.
passa per Q i té vector director n = (1, 0) y = 3 . Si inter-
t Resposta suggerida: La geometria euclidiana és la geome-
sequem aquestes dues rectes, resulta que F (2, 3).
tria que estudia les propietats del pla i l’espai tridimensio-
Aquests punts estan alineats si es compleix que: nal; la geometria projectiva estudia les incidències de punts
i rectes sense tenir en compte la mesura. En la projectiva,
e1 d1 e2 d2 14 5 1 27 5 0 9 9
= = = es permet demostrar tot el demostrable en euclidiana, sen-
f1 e1 f2 e2 2 14 5 3 27 5 4 4 se haver de recórrer a una mètrica.
t Quan dues rectes són paral·leles es diu que es tallen en un
10. Calculem les dues rectes perpendiculars a r que passen per A punt en l’infinit conegut com a punt impropi.
i per B i intersequem cadascuna amb la recta r.
t La recta perpendicular a r que passa per A té vector director — Mesuradors làser de distàncies
x 1 y 5 t Fórmula: D ct /2, en què c és la velocitat de la llum i t és la
n = ( 3, 7) = 7x + 3y 22 = 0
3 7 quantitat de temps per al viatge d’anada i tornada entre el
mesurador i la destinació.
7x + 3y 22 = 0 169 31 169 31
t x = ,y = A = , t Substituïm 300 m en la fórmula anterior, i obtenim el temps
3x + 7y + 5 = 0 58 58 58 58
de vol:
169 31 169 31
,y = A = , t 300 ct /2 t 600/c s, en què c és la velocitat de la llum,
58 58 58 58 que equival a 299 792 458 m/s. Per tant, el temps de vol és
t La recta perpendicular a r que passa per B té vector direc- de 2 × 10 6 segons.
x 8 y 8 t En el cas de distàncies curtes, es cometen imprecisions a
tor n = ( 3, 7) = 7x + 3y 80 = 0
3 7 l’hora de determinar el moment de la sortida i de l’arribada
del feix lluminós. En el cas de distàncies llargues, hi pot
7x + 3y 80 = 0 575 205 575 205
t x = ,y = B = , haver petites variacions en la velocitat de la llum, reflexions
3x + 7y + 5 = 0 58 58 58 58 no volgudes, etc.

124
BLOC 2. GEOMETRIA > UNITAT 8. ELEMENTS DEL PLA

t Resposta suggerida: Perquè viatgen en forma justa a relaci- — Negacions del postulat:
ons constants per l’atmosfera i viatgen distàncies molt més
t Per un punt exterior a una recta no es pot traçar cap
llargues sense perdre intensitat, és menys probable que es
dispersi i conserva una gran part de la intensitat original que recta paral·lela a la donada.
té quan es reflecteix en l’objectiu. t Per un punt exterior a una recta es pot traçar més
— El 5è postulat d’Euclides a debat d’una recta paral·lela a la donada.

t Quatre primers postulats d’Euclides: Els matemàtics més crítics sobre aquest tema van ser Gio-
vanni G. Saccheri, Carl F. Gauss, John Playfair, Nikolai Lo-
1. Dos punts qualssevol determinen un segment de recta.
batxevski, János Bolyai, Eugenio Beltrami, Karl Weierstrass,
2. Un segment de recta es pot estendre indefinidament en Felix Klein, Henri Poincaré i Bernhard Riemann.
una línia recta.
t Resposta suggerida: La geometria hiperbòlica és la geometria
3. Es pot traçar una circumferència donats un centre i un que pren com a postulat la segona negació esmentada en el
radi qualssevol. punt anterior. És la geometria intrínseca de la pseudoesfera,
4. Tots els angles rectes són iguals entre si. en què la suma dels angles d’un triangle és inferior a 180º.
D’altra banda, la geometria el·líptica és la que pren com a
t Reformulacions del cinquè postulat: postulat la primera negació. Per un punt exterior a una geo-
— John Playfair: Per un punt exterior a una recta es pot dèsica no passa cap paral·lela a ella i la suma dels angles
traçar una i només una paral·lela a aquesta recta. d’un triangle és superior a 180º.

125
BLOQUE
BLOC 2. 1.
GEOMETRIA
SOL_TIT_BLOQUE_NEGRO

15
9## Sol_Tit_Ud
Còniques

En context (pàg. 209) Per tant, l’equació de l’òrbita és


x2
+
y2
= 1.
1 455 642 1 451610
a) Resposta suggerida:
5. Les estacions de ràdio estan separades 500 milles, que són
— Es pensava que la Terra era el centre de l’Univers i que els focus de la hipèrbola; per tant, c 500/2 250. Utilitzant
els astres, incloent-hi el Sol, giraven al voltant seu. la velocitat de les ones de ràdio i la diferència de temps (2 640 mi-
— Nicolau Copèrnic. crosegons 2,64 · 10 3 segons) resulta que 2a  186 000 ·
· 2,64 · 10 3 491,04 a 491,04/2 245,52. I de la relació
— Ho va demostrar Johannes Kepler.
c 2 a 2 b 2 tenim que b 47,11.
b) Resposta suggerida:
6. 4x 2 4x 8y 9 0 x 2 x 1/4 2y 9/4 1/4
Els coneixements astronòmics maies eren propis de la
(x 1/2) 2(y 1), on x0 1/2, y0 1, p 1.
2
classe sacerdotal i el poble dirigia la seva vida d’acord amb
les prediccions que duien a terme. Se’n pot consultar més El focus de x 2 2y és F (0, 1/2) i la seva directriu és y 1/2.
informació a:
Per tant, el vèrtex és V (1/2, 1), el focus F (1/2, 1/2  1)
http://links.edebe.com/4sp (1/2, 3/2) i la recta directriu és y 1/2 1 1/2.

7. y2 4(x 2y) y 2 8y 4x y 2 8y 16 4x 16
Problemes resolts (pàgs. 223 i 224) (y 4) 4(x 4), on x0
2 4, y0 4, p 2.

1. Per a trobar l’equació de la circumferència, hem de calcular el El focus de y 2 4x és F (1, 0) i la seva directriu és x 1.
radi, que, en aquest cas, és la distància del centre C a la recta Per tant, el vèrtex és V ( 4, 4), el focus F (1 ( 4), 4)
r. Podem expressar la recta r com 3x 2y 3 0: ( 3, 4) i la recta directriu és x 1 ( 4) 5.
18 + 2 + 3 13
r = d(C,r ) = = = 13
32 + 22 13 Exercicis i problemes (pàgs. 225 a 228)
Aleshores, l’equació de la circumferència queda així:

(x + 6)2 + (y 1)2 = ( 13 )
2
(x + 6)2 + (y 1)2 = 13 1 CIRCUMFERÈNCIA Pàg. 225

2. Per a trobar l’equació de la circumferència, hem de calcular el 8. El lloc geomètric dels punts que disten 10 metres del punt P
radi, que és la distància del centre C a l’eix d’abscisses, que és la és la circumferència de radi 10 i centre P. És a dir, la circum-
ferència que té com a equació:
0+4+0 4
recta r: y 0 r = d(C, r ) = = =4
02 + 12 1 (x 3)2 (y 8)2 100

Aleshores, l’equació de la circumferència és la següent 9. El lloc geomètric del pla format pels punts que disten 7 unitats
del punt A és la circumferència de radi 7 i centre A. És a dir,
(x 1)2 + (y 4)2 = 42 (x 1)2 + (y 4)2 = 16
la circumferència que té com a equació:
3. Per a trobar l’equació de l’òrbita terrestre, hem de determinar (x 4)2 (y 5)2 49
els paràmetres a i b. Siguin P i A els vèrtexs de l’el·lipse corres-
ponents al periheli i a l’afeli, respectivament, i F el focus on es 10. Equació de la circumferència: (x a)2 (y b)2 r 2.
troba el Sol. Aleshores:
a) (x 2)2 (y 1)2 32 9
a (d (P, F) d (A, F))/2 (1 470 000 000 153 000 000)/2
b) (x 3)2 y2 42 16
1,5 · 108 a 2 2,25 · 1016
c a d (P, F) 3 · 106 c2 9 · 1012 c) (x 1)2 (y 5)2 130, on r és la distància entre el centre
a2 b2 c2 b2 a2 c2 2,249 1 · 1016 i P: r = d(C,P) = ( 4 + 1)2 + ( 6 5)2 = 130

Per tant, l’equació de l’òrbita terrestre és aquesta: 11. Com que A i B són diametralment oposats, pertanyen a una
x2 y2 circumferència on el centre, C, és el punt mitjà dels punts A i
+ =1 B, i el radi és la distància entre el centre i el punt A o B. Per
2,25 1016 2,2491 1016
tant:
4. Sigui F el centre de la Lluna, que alhora és un focus de l’òrbita 1 1+ 5 8 6
C = (A + B ) = , = ( 3,1)
lunar. I siguin P i A els vèrtexs de l’òrbita. Aleshores, 2 2 2
a (d (P, F) d (A, F) 2 · 1 075)/2 (68 195 2 · 1 075)/2 
r = d(C, A) = d(C,B) = (1 3)2 + (8 1)2 = 53
1 206,5 a 2 1 455 642; c a 68 1 075 63,5 c 2 
4 032,25 b 2 a 2 c 2 1 451 610. Aleshores, l’equació és (x 3)2 (y 1)2 53.

126
BLOC 2. GEOMETRIA > UNITAT 9. CÒNIQUES

12. L’equació de la circumferència concèntrica ha de tenir el ma- 2+3 2+3 13 2 13 5 5


teix centre que la circumferència donada. Calculem el centre d(C, r ) = = = > =r
22 + ( 1)2 5 10 2
(a, b):
La recta r és exterior a la circumferència.
m 2a 4 2a a 2; n 2b 2 2b b 1
b) Podem escriure la recta com x 4y 5 0.
Per tant, l’equació de la circumferència queda així:
6+5 1 0 5
(x 2)2 (y 1)2 22 4 d(C, s) = = =0< =r La recta
12 + 42 17 2
13. a) Representa una circumferència, ja que defineix l’equació s és secant a la circumferència; de fet, hi passa pel centre.
d’una circumferència de centre C (a, b) (3, 12) i radi
c) Podem escriure la recta com 2x y 3 0.
r = 625 = 25
2 32 3 52 5
b) Aquesta equació defineix una equació de circumferència. d(C,t ) = = = =r La recta t
+ 22 12 5 2
Calculem el centre i el radi:
és tangent a la circumferència.
m 2a 4 2a a 2; n 2b 10 2b
b 5 C (2, 5); p a 2 b 2 r 2 17. De les expressions de m, n i p obtenim:
13 22 ( 5)2 r 2 r 4
4 2a a 2; 2 2b b 1 C (2, 1)
c) Aquesta equació defineix una equació de circumferència. Figura 01
Calculem el centre i el radi: 0 22 12 r2 r = 5
Y
m 2a 1/2 2a a 1/4; n 2b 2 2b
1
b 1 C ( 1/4, 1)
p a2 b2 r2 1/16 ( 1/4)2 ( 1)2 r2 r 1
–1 0 1 2 3 4 5 X
d) Aquesta equació no representa una circumferència, ja que C = (2, –1)
–1
el terme 3xy no pertany al model d’equació general d’una
circumferència.
–2

14. Substituïm el punt P en l’equació: 12 12 2m 4m


4m2 0 m 1, m 1/2. Calculem el centre i el radi per –3

a aquests dos valors:


t m 1 x2 y2 2x 4y 4 0 18. El punt d’intersecció entre les rectes és:
m 2a 2 2a a 1; n 2b 4 2b b 2  x + 3y + 3 = 0
  C (1, 2); p a 2 b 2 r 2 4 12 ( 2)2 r2 r 3 x = 0, y = 1 C = (0, 1)
x + y +1 = 0
t m 1/2 x2 y2 x 2y 1 0
I l’equació de centre C i radi 5 és x 2 (y 1)2 25.
1 2a a 1/2; n 2b 2 2b b 1
C ( 1/2, 1) 19. Siguin P (x, y) els punts que compleixen la condició de
l’enunciat, que és:
p a2 b2 r2 1 ( 1/2)2 12 r2 r 1,5
d(A,P)2 + d(B,P)2 = 9 x 2 + y 2 + (1 x)2 + (1 y )2 = 9
15. De les expressions de m, n i p obtenim: 2x 2 + 2y 2 2x 2y 7=0 x2 + y2 x y 72=0
4 2a a 2; 2 2b b 1; 4 22 12 r2 r 3 1 2a a 1/2; 1 2b b 1/2 C (1/2, 1/2);
Calculem ara les distàncies entre els punts A, B i C amb el p a2 b2 r2 7/2 ( 1/2)2 ( 1/2)2 r2 r 2
centre de la circumferència P (2, 1) i les comparem amb el
radi r : 20. De les expressions de m, n i p obtenim:
td(A,P) = (2 5)2 + (1 4)2 = 18 > 3 = r El punt A 6 2a a 3; 2 2b b 1 C (3, 1)
és exterior a la circumferència.
15 32 12 r2 r 5
td(B,P) = (2 + 1)2 + (1 1)2 = 3 = r El punt B pertany a) La intersecció entre les dues circumferències té dues soluci-
a la circumferència. ons; per tant, les circumferències es tallen en dos punts. En
td(C,P) = (2 2)2 + (1 + 1)2 = 2 < 3 = r El punt C és conseqüència, les dues circumferències són secants.
interior a la circumferència. b) La intersecció entre les dues circumferències té dues solu-
cions; per tant, les circumferències es tallen en dos punts.
16. Calculem les distàncies entre les rectes i el centre de la cir- En conseqüència, són secants.
cumferència i les comparem amb el radi d’aquesta.
c) La intersecció entre les dues circumferències no té solució;
De les expressions de m, n i p obtenim:
per tant, les circumferències no es tallen. Calculem el cen-
2 2a a 1; 3 2b b 3/2 C (1, 3/2) tre i el radi de la circumferència:
2 12 ( 3/2)2 r2 r = 5 2 22 2a a 11; 6 2b b 3 C (11, 3)
a) Podem escriure la recta com 2x y 3 0. 121 112 ( 3)2 r2 r 3

127
BLOC 2. GEOMETRIA > UNITAT 9. CÒNIQUES

Per tant, l’equació de la circumferència és:


d(C,C ') = (11 3)2 + ( 3 1)2 = 80
d(C,C ') > r + r '
r +r'= 5+3 = 8 (x 2)2 (y 1)2 49/5

Per tant, les circumferències són exteriors. 26. La circumferència circumscriu el triangle; és a dir, passa per
tots els vèrtexs del triangle i, en conseqüència, es compleix
21. Sigui P (x, y) el centre de la circumferència. Per tant, s’ha de
d(P, A) = d(P,B) = d(P,C), en què P (x, y) és el centre de la
complir que d (P, A) d (P, B) d (P, C).
circumferència que busquem.
(x 2)2 + (y 1)2 = (x + 3)2 + (y + 4)2
(x 1)2 + (y 3)2 = (x 4)2 + (y 4)2
(x + 3)2 + (y + 4)2 = (x + 6)2 + (y 5)2
(x 4)2 + (y 4)2 = (x 3)2 + (y 1)2
x = 3, y = 1
11 11
P = ( 3,1) r = d(P, A) = 5 x = , y =
4 4
Aleshores, l’equació de la circumferència de radi 5 i centre P 11 11 50
és (x 3)2 (y 1)2 25 x 2 y 2 6x 2y 15 0. P = , r = d(P, A) =
4 4 4
2 2 2
22. La bisectriu del quart quadrant té com a equació y x, per 11 11 50 50
x + y = =
tant, el centre és de la forma C (x, x). La distància entre el 4 4 4 16
centre i l’origen de coordenades és 2; és a dir:

d(C,O) = 2 x2 + x2 = 2 x = 2 C = ( 2, 2 ) 27. Busquem el centre i el radi de la circumferència:

m = 2a = 2 2 , n = 2b = 2 2 , p = 0 0 2a a 0; 0 2b b 0 C (0, 0)

x2 + y 2 2 2x +2 2y = 0 9 02 02 r2 r 3
t La recta és tangent a la circumferència si d (C, s) r.
23. El centre està sobre la recta x 2y 3, per tant, és de la forma
C (3 2y, y). S’ha de complir que la distància entre aquest 0+0+k
d(C, s) = r =3 k =3 5 k = ±6,71
punt i els punts A i B sigui la mateixa. Aleshores: 12 + 22

d(C, A) = d(C,B) (3 2y 1)2 + (y 3)2 = t La recta és exterior a la circumferència si d (C, s) > r.


= (3 2y 3)2 + (y 5)2 y = 3 C = (9, 3)
0+0+k
d(C, s) > r >3 k > 6,71 i k < 6,71
r = d(C, A) = (9 1)2 + ( 3 3)2 = 10 12 + 22

24. Busquem el centre i el radi de la circumferència amb les ex-


pressions de m, n i p:
2 EL·LIPSE Pàg. 226
0 2a a 0; 0 2b b 0 C (0, 0); 1 02 02
r2 r 1 d(C, A) = (a + 1)2 + (2a)2 = 5a 2 + 2a + 1 28. t -FMrMJQTFFTUËDFOUSBEBFOMPSJHFOJa 5ib 4. Per tant,
l’equació de l’el·lipse queda així:
t Si d (C, A) < r, el punt A és interior a la circumferència.
x2 y2 x2 y2
d(C, A) < r 5a 2 + 2a + 1 < 1 5a 2 + 2a + 1 < 1 + =1 + =1 16x 2 + 25y 2 = 400
52 42 25 16
5a 2 + 2a < 0 a(5a + 2) < 0 25<a<0
t L’el·lipse està centrada en l’origen i a 1ib 3. Per tant,
t Si d (C, A) r, el punt A pertany a la circumferència. l’equació de l’el·lipse queda així:
d(C, A) = r 5a 2 + 2a + 1 = 1 5a 2 + 2a + 1 = 1
x2 y2 y2
5a 2 + 2a = 0 a(5a + 2) = 0 a = 2 5, a = 0 + =1 x2 + =1
12 32 9
t Si d (C, A) > r, el punt A és exterior a la circumferència.
29. a) L’equació reduïda compleix que d (P, F) d (P, F ) 2a 
d(C, A) > r 5a 2 + 2a + 1 > 1 5a 2 + 2a + 1 > 1
  2 8 2a a 5. De la relació a 2 b 2 c 2 i que
5a 2 + 2a > 0 a(5a + 2) > 0 a < 2 5, a > 0 c d (F, F )/2 6/2 3 tenim que 52 b 2 32 b 4.
Per tant, l’equació reduïda de l’el·lipse és:
25. Com que la circumferència és concèntrica a x 2 y 2 4x 2y
0 han de tenir el mateix centre; el busquem: x2 y2 x2 y2
+ =1 + =1
52 42 25 16
4 2a a 2; 2 2b b 1 C (2, 1)
La recta s: 2x y 2 0 és tangent a la circumferència; per b) L’excentricitat és e c/a e 3/5.
això s’ha de complir que d (C, s) r.
a2 25 i b2 16 c2 a2 b2 25 16 c 3
4 +1+ 2 7
d(C, s) = r =r r = Després, com que els focus estan situats sobre l’eix d’abs-
22 + ( 1)2 5
cisses, tenim que F (3, 0) i F ( 3, 0).

128
BLOC 2. GEOMETRIA > UNITAT 9. CÒNIQUES

30. a) L’el·lipse és determinada per la seva equació reduïda amb d) La distància focal és 8; per tant c 8/2 4 i b 3. Alesho-
els focus en l’eix OX: res, de la relació a 2 b2 c2 a2 32 42
a 5.
x2 y2 x2 y2
+ =1 + =1 x2 y2
25 9 52 32 I l’equació de l’el·lipse resulta + = 1.
25 9
A partir de l’equació, tenim que a 2 25 a 5 i b 2  9
b 3, per tant, c 2 a 2 b 2 16 c 4. e) A partir del focus, sabem que c 1,5 3/2 i a partir de
l’excentricitat e c/a 0,8 4/5 (3/2)/a a (3/2)/
Per tant, els focus F (0, 4) i F (0, 4,).
/(4/5) 15/8 a 2 225/64. I de la relació a 2 b 2 c 2
Els vèrtexs són V (0, 5) i V (0, 5) i l’excentricitat és tenim que b 2 (15/8)2 (3/2)2 81/64. Per tant, l’equació
e c/a 4/5. de l’el·lipse és:
x2 y2 x2 y2 64x 2 64y 2
b) 9x 2 4y 2 36 + =1 Com que el denomina- + =1 + =1
4 9 225 64 81 64 225 81
dor de x 2 és més petit que el de y 2, els focus estan situats
f) El valor de b és 6 i, com que e 1/3 c/a aleshores a 3c.
sobre l’eix OY a2 9 a 3 i b2 4 c2 a2 b2
De la relació a 2 b2 c2 (3c)2 62 c2
3 2 2 c
3 c 5 .
a 9 2 2 a2 32. Per tant, l’equació de l’el·lipse
Per tant, els focus són F (0, 5 ) i F (0, 5 ); els x 2 y 2
vèrtexs són V (0, 3) i V (0, 3) i l’excentricitat és és + = 1.
32 36
e c/a 5 3.
x2 y2 33. a) a d (C, V) a= (9 4)2 + (2 2)2 a 5
c) 5x 2 6y 2 30 + =1 Com que el denomina-
6 5
c d (C, F) c = (7 4)2 + (2 2)2 c 3
dor de x 2 és més gran que el de y 2, els focus estan situats
sobre l’eix OX a 2 6 a 6 i b2 5 c2 a2 b2 a2 b2 c2 52 b2 32 b 4
1 c 1. (x c1)2 (y c 2 )2 (x 4)2 (y 2)2
+ =1 + =1
Per tant, els focus són F (1, 0) i F ( 1, 0); els vèrtexs a2 b2 25 9
són V ( 6 , 0) i V ( 6 , 0) i l’excentricitat és e c/a
b) La distància focal mesura 16; per tant, c 16/2 8. L’eix
1 6 = 6 6. major mesura 20; aleshores a 20/2 10. De la relació fo-
namental a 2 b2 c 2 tenim que 10 2 b 2 82
x2 y2 b 6.
d) x 2 4 2y 2 + =1 Com que el denominador
4 2
(x c1)2 (y c 2 )2 (x 1)2 (y 4)2
de x 2 és més gran que el de y 2, els focus estan situats so- + =1 + =1
bre l’eix OX a 2 4 a 2 i b 2 2 c 2 a 2 b 2 2 a2 b2 100 36
c 2.
34. a) El centre de l’el·lipse és C (3, 4). D’altra banda, i tenint
Per tant, els focus són F ( 2 , 0) i F ( 2 , 0); els en compte que el denominador de x 2 és més gran que el
vèrtexs són V (2, 0) i V ( 2, 0) i l’excentricitat és denominador de y 2, sabem que els focus estan en l’eix OX,
e c/a 2 2. 64 a 2 i 16 b 2, per tant, utilitzant la relació a 2   b 2 c 2
tenim que 64 16 c 2 c 7.
31. La suma de les distàncies d’un punt qualsevol als focus és 10;
Aleshores, els focus són F (3 c, 4) ( 4, 4) i F
per tant, es compleix que 10 2a a 5. Sabem que c  3.
(3  c, 4) (10, 4).
De la relació a 2 b2 c 2 tenim que 5 2 b2 32
b 4. Aleshores, l’equació de l’el·lipse queda així: b) El centre de l’el·lipse és C (2, 1). D’altra banda, i tenint en
compte que el denominador de x 2 és més petit que el deno-
x2 y2 x2 y2
+ =1 + =1 minador de y 2, sabem que els focus estan en l’eix OY, 4  a 2
a2 b2 25 16 i 1 b 2, per tant, utilitzant la relació a 2 b 2 c 2 tenim que
4 1 c2 c 3.
32. a) Sabem que a 5 i c 4 b2 a2 c2 9 b 3. Ales-
x2 y2 Aleshores, els focus són F (2, 1 c) (2, 1 3) i
hores, l’equació de l’el·lipse és + = 1.
25 9 F (2, 1 c) (2, 1 3 ).
b) La longitud de l’eix major és 6; per tant, a 6/2 3. La
35. La distància focal és de 369 200; per tant, c 369 200/2
longitud de l’eix menor és 4; per tant, b 4/2 2. Alesho-
184 600. De la fórmula de l’excentricitat, tenim que e c/a
x2 y2 0,84 184 600/a a 184 600/0,84 219 761,904 8. Per
res, l’equació de l’el·lipse és + = 1.
9 4 tant, la distància màxima a la qual es troba la Terra de la Lluna
és a c 404 361,904 8 km.
c) La longitud de l’eix major és 12; per tant, a 12/2 6 i
c d (F, F )/2 10/2 5. I el valor de b resulta de la relació
36. Calculem els punts d’intersecció, A i B, entre la recta i
a2 b2 c2 62 b 2 52 b 2 11. Per tant, l’equació
l’el·lipse i després calculem el punt mitjà d’aquests dos
x2 y2
de l’el·lipse és + = 1. punts.
36 11

129
BLOC 2. GEOMETRIA > UNITAT 9. CÒNIQUES

x + 2y 1= 0 x = 1, y = 1 c) La longitud de l’eix real és 12; per tant, a 12/2 6.


El punt P compleix que |d (P, F ) d (P, F)| 2a en què
x 2 + 2y 2 = 3 x = 5 3, y = 1 3
F   (c, 0) i F ( c, 0).
5 1
A = ( 1,1),B = , (8 + c)2 + 142 (8 c)2 + 142 = 12 c = 12 2
3 3
2
1 1+ 5 3 1 13 1 1 b2 = c2 (
a 2 = 12 2 ) 62 b 2 = 252
P = (A + B) = , = ,
2 2 2 3 3
x2 y2
Per tant, l’equació de la hipèrbola és = 1.
36 252
37. El punt P compleix que d (P, F) d (P, F ) 2a
d) Sabem que el valor de a és 1 i que l’equació de les asímp-
( 4 3)2 + (0 1)2 + (4 3)2 + (0 1)2 = 2a b b
totes és y = ± x = ±2x =2 b = 2, per tant,
50 + 2 = 2a 5 2 + 2 = 2a a=3 2 a a
c = d(F ,F ') 2 = (4 + 4)2 2=82=4 y2
l’equació de la hipèrbola és x 2 = 1.
2 4
a2 = b2 + c2 (3 2 ) = b2 + 42 b = 2
x2 y2 x2 y2 42. a) Sabem que a 3ic 5 b2 c2 a2 16, per tant,
2
+ 2
=1 + =1 x2 y2
18 2 l’equació de la hipèrbola és
(3 2 ) ( 2) 9 16
= 1.

38. Busquem la diagonal del quadrat, d, que és la distància focal b) Sabem que a 8ic 10 b2 c2 a2 36, per tant,
de l’el·lipse. Hi apliquem el teorema de Pitàgores: x2 y2
l’equació de la hipèrbola és = 1.
64 36
d2 a2 a2 2a 2 d a 2 c d/2 c a 2 2
a 2 c) Sabem que a 1 i que l’equació de les asímptotes és
e c/a e= a= 2 2 b b
2 y =± x = ±5x =5 b = 5 ; per tant, l’equació
a a
39. Sigui e c/a l’excentricitat d’una el·lipse i e c /a l’excentrici- y2
de la hipèrbola és x 2 = 1.
tat de l’altra el·lipse. Com que han de ser iguals, e e 25
c/a c /a . Com que a, b, c i a , b , c formen dos triangles
d) Sabem que a 6 i que el punt P compleix que |d (P, F ) 
i c/a c /a és una relació de proporcionalitat, implica que els
d (P, F)| 2a en què F (0, c) i F (0, c).
semieixos menors b i b són proporcionals.
( 5)2 + (9 c)2 ( 5)2 + (9 + c)2 = 12 c = 2 14
2
3 HIPÈRBOLA Pàgs. 226 i 227 b2 = c2 (
a 2 = 2 14 ) 62 b 2 = 20

x2 y2
40. a) a 1 i b 3 i els focus estan col·locats en l’eix OY; per tant, Per tant, l’equació de la hipèrbola és = 1.
la gràfica és la quarta. 36 20

x2 y2 e) Sabem que c 1 i a 1/2 b2 c2 a2 3/4, per tant,


b) 9x 2 25y 2 225 =1 a 5ib 3. Per l’equació de la hipèrbola és la següent:
25 9
tant, la gràfica corresponent és la primera. y2 x2 4x 2
=1 4y 2 =1
c) a 2 i b 1, per tant, la gràfica corresponent és la terce- 14 34 3
ra.
f) A partir de l’equació de les asímptotes, tenim que
y2 x2 b 1 b 1
d) 16y 2 x2 144 =1 a 3, b 12 i els y =± x =± x = a = 2b . El punt (5, 2)
9 144 a 2 a 2
focus estan col·locats en l’eix OY, per tant, la gràfica és la passa per la hipèrbola; per tant,
segona.
52 22 25 4
=1 =1
41. a) El punt P compleix que |d (P, F ) d (P, F)| 2a a2 b2 a2 b2 .
2 2 Si resolem el sistema entre les dues condicions anteriors,
(8 3)2 + 5 3( ) (8 + 3)2 + 5 3 ( ) = 2a resulta que a 2 9 i b 2 9/4. Aleshores, l’equació de la hi-
x2 y2
a=2 pèrbola és =1 x 2 4y 2 = 9.
9 94
El valor de c és 3 i de la relació c 2 a 2 b 2 b2 5,
x2 y2 43. a) Sabem que a 2 4 a 2; b 2 16 b 4; c 2 a2
aleshores l’equació de l’el·lipse és = 1.
4 5 b2 20 c 20 = 2 5 .
b) Els focus estan situats en la recta x 3 paral·lela a l’eix OY Per tant, els focus són F (2 5 , 0) i F ( 2 5 , 0); els
i, aleshores, a 4/2 2, c 8/2 4 b 2 c 2 a 2 12 i vèrtexs són V (2, 0) i V ( 2, 0) i les asímptotes són
(x 3) 2 (y + 3)2 b 4
l’equació de la hipèrbola és = 1. y =± x y =± x y = ±2x .
12 4 a 2

130
BLOC 2. GEOMETRIA > UNITAT 9. CÒNIQUES

x2 y2 b) El centre de la hipèrbola és C (1, 2). Sabem que a 2 5


b) 9x 2 4y 2 36 =1 a2 4 a 2;
4 9 a 5 ; b2 5/4 b 5 2; c2 a2 b2 25/4
b2 9 b 3; c 2 a2 b2 13 c 13 . c 5/2.
Aleshores, els focus són F (1 c, 2) ( 3/2, 2) i F
Per tant, els focus són F ( 13 , 0) i F ( 13 , 0); els
(1 c, 2) (7/2, 2) i els vèrtexs són V (1 a, 2)
vèrtexs són V (2, 0) i V ( 2, 0) i les asímptotes són
b 3 (1 5 , 2) i V (1 a, 2) (1 5 , 2).
y =± x y =± x.
a 2

c) x 2 y2 4 0 = 1
x2
a2 4
y2
a 2;
4 PARÀBOLA Pàgs. 227 i 228
4 4
b 2 4 b 2; c 2 a b
2 2 8 c 8 = 2 2 i obser- 48. Com que el punt P està contingut en la paràbola, substituïm
vem que els focus estan situats sobre l’eix OY. aquest punt en la seva equació: ( 6)2 2p( 1) 36 2p
p   18.
Per tant, els focus són F (0, 2 2 ) i F (0, 2 2 ); els
vèrtexs són V (0, 2) i V (0, 2) i les asímptotes són 49. a) En aquest cas, l’eix de simetria és l’eix OX. Per tant, 8
b 2p p 4 i concloem que F ( p/2, 0) ( 2, 0) i la
y =± x y = ±x .
a directriu d és x p/2 4/2 x 2.
x2 y2 b) En aquest cas, l’eix de simetria és l’eix OY. Per tant, 12 
d) x2 y2 8 0 = 1 a2 8 a 8 = 2 2;
8 8 2p p 6 i concloem que F (0, p/2) (0, 3) i la direc-
b2 8 b triu d és y p/2 6/2 y 3.
8 = 2 2 ; c 2 a 2 b 2 16 c 4.
c) L’equació reduïda de y 5x 2 és x 2 y/5 i l’eix de simetria
Per tant, els focus són F (4, 0) i F ( 4, 0); els vèrtexs és l’eix OY. Per tant, 1/5 2p p 1/10 i concloem que
són V (2 2 , 0) i V ( 2 2 , 0) i les asímptotes són F (0, p/2) (0, 1/20) i la directriu d és y p/2
b y 1/20.
y =± x y = ±x .
a d) L’equació reduïda de 8x 2 12y 0 és x 2 3y/2 i l’eix de si-
metria és l’eix d’ordenades. Per tant, 3/2 2p
44. a) Si observem la gràfica, veiem que a 2 i c 4
p 3/4 i concloem que F (0, p/2) (0, 3/8) i la direc-
b2 c 2    a 2 12 i l’equació de la hipèrbola és
triu d és y p/2 y 3/8.
x2 y2
= 1.
4 12 50. a) L’equació reduïda de 6y 2 12x 0 és y 2 2x i l’eix de si-
b) Si observem la gràfica, veiem que a 4, els focus estan metria és l’eix OX. Per tant, 2 2p p 1 i concloem que
situats sobre l’eix OY i el punt P (3, 5) compleix que F (p/2, 0) (1/2, 0) i la directriu d és x    p/2 x
|d (P, F ) d (P, F)| 2a, en què F (0, c) i F (0, c). 1/2.

32 + ( 5 c)2 32 + ( 5 + c)2 = 2 a = 8 c =4 2 b) L’equació reduïda de 15x 2 42y és x 2 14y/5 i l’eix de


simetria és l’eix d’ordenades. Per tant, 14/5 2p
2
b2 = c2 a2 = 4 2 ( ) 42 b 2 = 16 p 7/5 i concloem que F (0, p/2) (0, 7/10) i la
directriu d és y p/2 y 7/10.
x2 y2
Per tant, l’equació de la hipèrbola és = 1.
16 16 51. a) El focus està situat en la meitat negativa de l’eix d’abscisses
i el vèrtex en el punt (0, 0). Com que F ( 2, 0) i la directriu
45. Com que és una hipèrbola equilàtera, significa que els seus és x 2, podem deduir que 2 p/2 p 4. L’equació
semieixos són iguals, és a dir, a b i c2 2a 2 c a 2. reduïda de la paràbola és: y 2 2 · 4x 8x.
x2 y2
Per tant, l’equació de la hipèrbola és = 1 , les b) El vèrtex és (0, 0) i la directriu, y 6; per tant, la directriu
a2 a2 està situada per sobre del vèrtex. Aleshores, a partir de la
b a directriu, tenim que p/2 6 p 12. L’equació reduïda
asímptotes són y = ± x y =± x = ±x i l’excentricitat
a a de la paràbola és: x 2 2 · 12y 24y.
c a 2 c) El focus està situat en el semieix negatiu de l’eix d’ordena-
és e = e = = 2.
a a des. La directriu és y 5; per tant, el vèrtex està situat per
sobre del focus i p/2 5 p 10. L’equació reduïda de la
46. A partir dels focus, sabem que c 5. La diferència de distàn- paràbola és: x 2 2 · 10y 20y.
cies d’un punt qualsevol als focus és 8; per tant,
8 2a a 4 b 2 c 2 a 2 9 i l’equació de la hipèrbola d) Com que l’eix de simetria és l’eix OY, la directriu és perpen-
dicular a aquest eix; és a dir, paral·lela a l’eix OX. També
x2 y2
resulta la següent: = 1. passa pel punt P ( 3, 3) que està en el segon quadrant;
16 9 per tant, la paràbola estarà situada en els quadrants pri-
47. a) El centre de la hipèrbola és C (4, 1). Sabem que mer i segon, i la directriu estarà per sota del vèrtex.
a2 16 a 4; b 2 9 b 3; c 2 a 2 b 2 25 c 5. D’aquesta manera, l’equació de la paràbola és del tipus
x 2 2py, en què substituïm el punt P en aquesta equació,
Aleshores, els focus són F (4 c, 1) ( 1, 1) i
ja que pertany a la paràbola:
F (4 c, 1) (9, 1) i els vèrtexs són V (4 a, 1)
(0, 1) i V (4 a, 1) (8, 1). ( 3)2 2p · 3 p 3/2 x2 2 · 3/2y x2 3y

131
BLOC 2. GEOMETRIA > UNITAT 9. CÒNIQUES

52. a) Tenint en compte que el focus és a l’esquerra del vèrtex Per tant, el vèrtex és V (x0, y0) (4, 7/2) i el focus F
i, en conseqüència, la directriu és paral·lela a l’eix OY, (x0, y0 p/2) (4, 7/2 3/2) (4, 2).
V (5, 2) (a, b) i F (3, 2) (a p/2, b). Per tant:
56. y x 2/8 2 x2 8y 16 x2 8(y 2)
3 a p/2 3 5 p/2 p 4
a) Els punts de tall amb l’eix OX corresponen a la solució de
I l’equació de la paràbola és:
l’equació quan y 0, per tant:
(y 2)2 2 · ( 4) · (x 5) (y 2)2 8(x 5)
0 x 2/8 2 x 4ix 4 A (4, 0) i B ( 4, 0)
b) Tenint en compte que el focus està per sobre del vèrtex i,
Els punts de tall amb l’eix OY corresponen a la solució de
en conseqüència, la directriu és paral·lela a l’eix OX,
l’equació quan x 0, per tant:
V   ( 2, 2) (a, b) i F ( 2, 5) (a, b p/2). Per tant:
y 0/8 2 y 2 C (0, 2)
5 b p/2 5 2 p/2 p 6
b) Els vèrtexs que limiten l’eix major són ( 4, 0) i (4, 0); per tant,
I l’equació de la paràbola és:
a 4. El vèrtex que limita una part de l’eix menor és (0, 2);
(x 2)2 2 · 6 · (y 2) (x 2)2 12(y 2) Figura 02 manera, l’equació de l’el·lipse és:
per tant, b 2. D’aquesta
c) Tenint en compte que el focus és a la dreta del vèrtex i, en x2 y2 x2 y2
+ =1 + =1 x 2 + 4y 2 = 16
conseqüència, la directriu és paral·lela a l’eix OY, V   (1, 4) 4 2 2 2 16 4
(a, b) i F (3, 4) (a p/2, b). Per tant: Y

3 a p/2 3 1 p/2 p 4 2
c

I l’equació de la paràbola és: 1

(y 4)2 2 · 4 · (x 1) (y 4)2 8(x 1) A F F A


–4 –3 –2 –1 0 1 2 3 4 X
53. Com que la directriu és y 0 i sabem que y b p/2, alesho-
res 0 b p/2. –1

D’altra banda, F (2, 4) (a, b p/2) i, per tant, a 2 i 4  –2 B

 b p/2.
Si resolem les dues equacions anteriors amb les incògnites b i p, Calculem el valor de c per poder calcular l’excentricitat. De
resulta que b 2 i p 4. Aleshores, l’equació de la paràbola és: la relació a 2 b 2 c 2 resulta que:

(x 2)2 2 · 4 · (y 2) (x 2)2 8(y 2) c2 42 22 12 c = 12 = 2 3


Finalment, calculem el valor de l’excentricitat:
54. Tenint en compte que el focus és a l’esquerra del vèrtex i, en
conseqüència, la directriu és paral·lela a l’eix OY, V (5, 2) c 2 3 3
(a, b) i F (3, 2) (a p/2, b). Aleshores, l’equació de la e = e = =
a 4 2
paràbola és:
3 a p/2 3 5 p/2 p 4
I l’equació de la paràbola és:
SÍNTESI Pàg. 228

(y 2)2 2 · ( 4) · (x 5) (y 2)2 8(x 5) 9y 2 x2 y2


57. a) 4x 2 9y 2 36 x2 + =9 + =1
4 9 4
55. a) y 2 6y 8x 17 0 y2 6y 8x 17
Observem que l’equació correspon a una el·lipse de centre
Sumem 9 a cada membre de l’equació: C (0, 0), en què a 2 9 a 3 i b2 4 b 2.
y2 6y 9 8x 17 9 y2 6y 9 8x 8 9y 2 x2 y2
b) 16x 2 9y 2 144 x2 =9 =1
El primer membre és el quadrat d’una resta, i del segon 16 9 16
membre traiem factor comú 8: Observem que l’equació correspon a una hipèrbola de
y2 6y 9 8x 8 (y 3)2 8(x 1) centre C (0, 0), en què a 2 9 a 3 i b 2 16
b 4.
en què y0 3, x0 1ip 8/2 4.
c) 9x 2 9y 2 25 x2 y2 25/9
Per tant, el vèrtex és V (x0, y0) (1, 3) i el focus F    (x0
p/2, y0) (1 4/2, 3) (3, 3). Observem que l’equació correspon a una circumferència
de centre C (0, 0), en què r 2 25/9 r 5/3.
b) x2 8x 6y 5 0 x2 8x 6y 5
d) y 2 14x
Sumem 16 a cada membre de l’equació:
Observem que l’equació correspon a una paràbola de cen-
x2 8x 16 6y 5 16 x2 8x 16 6y 21
tre C (0, 0), en què 14 2p p 7. Aleshores, la direc-
El primer membre és el quadrat d’una resta, i del segon triu és d: x p/2 7/2.
membre traiem factor comú 6:
58. a) Si A B i un dels dos és zero, l’equació de la cònica cor-
x2 8x 16 6y 21 (x 4)2 6(y 7/2)
respon a una paràbola, ja que només tindríem un terme
en què y0 7/2, x0 4ip 6/2 3. elevat al quadrat.

132
BLOC 2. GEOMETRIA > UNITAT 9. CÒNIQUES

b) Si A B i són diferents de zero, l’equació de la cònica cor- 62. a) Suposem que el Sol és un dels focus de l’òrbita; i obser-
respon a una circumferència ja que si fossin diferents seria vem l’equació reduïda a 5 800 i b 1 500. De la relació
una el·lipse o una hipèrbola. a2 b2 c 2 resulta que c 2 5 800 2 1 500 2
31 390 000 c 5 603. Aleshores, la distància màxima de
c) Si A B i tenen el mateix signe, l’equació de la cònica
Plutó al Sol és a c 5 800 5 603 11 403 milions de qui-
correspon a una el·lipse.
lòmetres i la distància mínima és a c 5 800 5 603
d) Si A B i tenen el signe diferent, l’equació de la cònica 197 milions de quilòmetres.
correspon a una hipèrbola.
b) Com hem calculat en l’apartat anterior, a 5 800 i c
5 602,68. Per tant, l’excentricitat és:
59. 4x 2 3y 2 48 0 4x 2 3y 2 48
e c/a 5 603 / 5 800 0,97
3y 2 x2 y2 x2 y2
x2 + = 12 + =4 + =1
4 3 4 12 16 63. a) x 2 y 2 2x 6y 1 0
Aquesta equació correspon a una el·lipse i, com que el deno- Observem que l’equació de la cònica correspon a una cir-
minador de x 2 és més petit que el denominador de y 2, els fo- cumferència. Busquem el centre i el radi:
cus estan situats en l’eix OY i a 2 16 i b 2 12. De la relació
a 2 b 2 c 2 resulta que c 2 16 12 4 c 2. Per tant, m 2a 2 2a a 1; n 2b 6 2b
els focus són: F (0, c) (0, 2) i F (0, c) (0, 2). b 3 C ( 1, 3)

D’altra banda, com que el centre pertany a la recta 2x 3y 7 p a2 b2 r2 1 ( 1)2 ( 3)2 r2 r 3


0, serà de la forma C ((3y 7)/2, y). x2 y2 x2 y2
b) 8x 2 3y 2 120 =5 =1
Com que busquem l’equació d’una circumferència, la condi- 3 8 15 40
ció que s’ha de complir és que d (C, F) d (C, F ).
Observem que l’equació de la cònica correspon a una hi-
3y 7
2
3y 7
2 pèrbola de centre C (0, 0), en què a 2 15 a
+ (y 2)2 = + (y + 2)2 y =0
2 2 15 i b 2 40 b 40 = 2 10 .
3y 7 7 7 x2 x2 y2
x = = C = ,0 c) x 2 4y 2 100 + y 2 = 25 + =1
2 2 2 4 100 25
El radi de la circumferència és la distància entre el centre C i el punt Observem que l’equació correspon a una el·lipse de centre
2 C (0, 0), en què a 2 100 a 10 i b 2 25 b 5.
7 65
FoF r = d(C,F ) = 0 + (0 2)2 = . d) y 2 36x
2 4
Veiem que l’equació correspon a una paràbola de centre
Per tant, l’equació de la circumferència de radi r i centre C és
2
C  (0, 0), en què 36 2p p 18. Aleshores, la directriu
2
7 65 65 és d: x p/2 18/2 9.
x+ + y2 = = .
2 4 4
64. La condició que estableix l’enunciat es tradueix en el següent
(si P (x, y) són els punts del pla que compleixen aquesta
60. Es tracta de resoldre els exercicis que es proposen a la pàgina condició):
web indicada en l’enunciat.
d(P, A) d(P,B) = 1 x 2 + (y 3)2 x 2 + (y + 1)2 = 1
61. Busquem en primer lloc els punts d’intersecció entre les dues 2 2

circumferències: ( x 2 + (y 3)2 ) = (1 + x 2 + (y + 1)2 )


4x 2 60y 2 + 120y 45 = 0
x2 + y 2 2x + 10y + 17 = 0
x = 1, y = 2; x = 4, y = 5
x2 + y 2 14x 2y + 5 = 0 Hi apliquem l’exercici 58: com que A 4 60 B i els seus
A = (1, 2), B = (4, 5) signes són diferents, tenim que l’equació correspon a una hi-
pèrbola.
Com que busquem l’equació d’una circumferència, la condi-
ció que s’ha de complir és que, si C (x, y) és el centre de la 65. Comencem buscant els focus de l’el·lipse. A partir de l’equa-
circumferència, d (C, A) d (C, B) d (C, P). ció, veiem que a 2 25 a 5 i b2 9 b 3. De la relació
a 2 b 2 c resulta que c
2 2 25 9 16 c 4. Aleshores,
(x 1)2 + (y + 2)2 = (x 4)2 + (y + 5)2 tenint en compte que els focus estan situats sobre l’eix OX,
(x 4)2 + (y + 5)2 = (x + 3)2 + (y + 2)2 els focus de l’el·lipse són: F (c, 0) (4, 0) i F ( c, 0)
( 4, 0).
x = 1, y = 7
C = ( 1, 7) D’altra banda, la circumferència passa per aquests focus i té
centre en l’origen de coordenades; aleshores, el radi és la
El radi de la circumferència és la distància entre el centre C i distància entre el centre i un dels focus:
2 2
el punt P r = d(C,P) = ( 1 + 3 ) + ( 7 + 2)2 = 29 . r = d(C,F ) = ( 0 4 ) + (0 0)2 = 4
Per tant, l’equació de la circumferència de radi r i centre C és
2
Per tant, l’equació de la circumferència amb centre en l’ori-
2 2
( x + 1) + ( y + 7 ) = ( 29 ) = 29. gen de coordenades i radi r és x 2 + y 2 = 42 = 16.

133
BLOC 2. GEOMETRIA > UNITAT 9. CÒNIQUES

Finalment, calculem els punts d’intersecció entre la circumfe- Aleshores, l’equació de centre C i radi r és:
rència i l’el·lipse: 2

x2 + y2 = 16
(x 2)2 + (y + 3)2 = ( 50 ) (x 2)2 (y 3)2 50
5 7 9 b) El radi de la circumferència és la distància entre els punts
x2 y2 x =± ,y =±
+ =1 4 4 M i N dividida entre dos. És a dir:
25 9
2
5 7 9 d(M,N) ( 3 3 ) + ( 3 3)2 72
P = ± ,± r = = = =3 2
4 4 2 2 2

El centre de la circumferència és el punt mitjà entre els


16y 2 x2 y2 punts M i N:
66. a) C1: 9x 2 16y 2 144 x2 + = 16 + =1
9 16 9
1 3+3 3+3
Observem que l’equació correspon a una el·lipse de centre C = (M + N ) = , = (0, 0)
2 2 2
C (0, 0), en què a 2 16 a 4 i b2 9 b 3. De la
relació a 2 b 2 c Figura
2 resulta
03
que c 2 16 9 7 I resulta que l’equació de centre C i radi r és:
c 7. 2

Y
A
x2 + y 2 = 3 2 ( ) x2 y2 18

c
3 c) El radi de la circumferència és la distància entre els punts
2 P i Q dividida entre dos. És a dir:
2
1
d(P,Q) ( 6 2 ) + (6 0)2 10
F F
r = = = =5
2 2 2
–4 –3 –2 –1 0 1 2 3 4 X

–1 El centre de la circumferència és el punt mitjà entre els


punts P i Q:
–2

1 6+2 6+0
–3 C = (P + Q ) = , = ( 2, 3)
2 2 2
16y 2 x2 y2 I resulta que l’equació de centre C i radi r és:
C 2: 9x 2 16y 2 144 x2 = 16 =1
9 16 9
(x 2)2 (y 3)2 52 (x 2)2 (y 3)2 25
Observem que l’equacióFigura
correspon
04 a una hipèrbola de centre
C (0, 0), en què a 2 16 a 4 i b2 9 b 3. De la d) Calculem el punt d’intersecció entre les rectes:
relació c a b resulta que c 2 16 9 25 c 5.
2 2 2
2x + 5y = 10
Y x = 10, y = 2 C = (10, 2)
x 8y = 26
c
5
Per tant, l’equació de centre C i radi 2 és:
F F (x 10)2 (y 2)2 22 (x 10)2 (y 2)2 4
–10 –5 0 5 10 X
2. El centre és a la recta x 2y 0; aleshores, serà de la forma
–5 C ( 2y, y). Com que la circumferència passa pels punts P i
Q, s’ha de complir que d (C, P) d (C, Q).
b) Els vèrtexs de l’el·lipse C1 són V1 (a, 0) (4, 0), V2 ( 2y 4)2 + (y 3)2 = ( 2y 0)2 + (y 1)2
( a, 0) ( 4, 0), V3 (0, b) (0, 3) i V4 (0, b) (0, 3).
y = 2 x =4 C = (4, 2)
Com que la paràbola passa per tres d’aquests vèrtexs i és
oberta cap a la dreta, els vèrtexs pels quals passarà són V2, V3 El radi és la distància entre el centre C i els punts P o Q.
i V4 i el vèrtex de la paràbola serà V2. Aleshores, l’equació
de la paràbola serà de la forma: r = d(C,P) = (4 4)2 + ( 2 3)2 = 5

y2 2p(x 4) I l’equació de la circumferència de radi 5 i centre C és:


Com que els punts V3 i V4 formen part d’aquesta paràbola, (x 4)2 (y 2)2 52 (x 4)2 (y 2)2 25
podem substituir un dels dos en l’equació i així obtenir el
valor de p 32 2p(0 4) p 9/8. Per tant, l’equació 3. Com que la circumferència que busquem és concèntrica a la
de la paràbola és: circumferència x 2 y 2 2x 10y 17 0, tindrà el mateix
centre que aquesta. El busquem:
y2 2p(x 4) y2 2 · 9/8 · (x 4) y2 9/4(x 4)
2 2a a 1; 10 2b b 5 C ( 1, 5)

Avaluació (pàg. 230)


I l’equació de la circumferència de centre C i radi 4 és:
(x 1)2 (y 5)2 42 (x 1)2 (y 5)2 16
1. a) Trobem el radi de la circumferència, en què r d (C, P).
2
4. Calculem les distàncies entre les rectes i el centre de les cir-
r = d(C,P) = ( 2 1) + ( 3 4)2 = 50 cumferències, i les comparem amb el radi d’aquestes.

134
BLOC 2. GEOMETRIA > UNITAT 9. CÒNIQUES

a) De les expressions de m, n i p obtenim: 7. Si observem l’equació de la hipèrbola, veiem que el centre és


4 2a a 2; 0 2b b 0 C (2, 0) C (1, 3); a 2 9 a 3 i b2 4 b 2. De la relació c 2
1 22 02 r2 r = 5 a2 b 2 tenim que c 2 9 4 13 c 13 .
2 2 1 0 4 0 Per tant, els focus són F (1 c, 3) (1
d(C, r ) = = =0< 5 =r 13 , 3) i F
22 + ( 1)2 5
(1  13 , 3). Els vèrtexs són A (1 a, 3) (1 3, 3)
La recta r és secant a la circumferència. (4, 3) i A (1 a, 3) ( 2, 3).

b) De les expressions de m, n i p obtenim: Finalment, calculem l’excentricitat: e c/a e 13 3


2 2a a 1; 3 2b b 3/2 C (1, 3/2)
8. a) El vèrtex és (2, 2) (a, b) i la directriu y 5, per tant, la
2 12 ( 3/2)2 r2 r = 5 2 directriu està situada per sota del vèrtex. Aleshores, a par-
tir de la directriu, tenim que y b p/2 5 2 p/2
2 1 + 1 ( 3 2)
3 52 5 p 6. L’equació reduïda de la paràbola és:
d(C, r ) = = = =r La
22 + 12 5 2
(x 2)2 2 · 6 · (y 2) (x 2)2 12(y 2)
recta r és tangent a la circumferència.
b) Tenint en compte que el focus és a la dreta del vèrtex i, en
5. a) Aquesta el·lipse està centrada en el punt C (1, 3) i, com conseqüència, la directriu és paral·lela a l’eix OY, V (2, 1)
que el denominador de x 2 és més petit que el de y 2, els (a, b) i F (6, 1) (a p/2, b). Per tant:
focus estan situats sobre la recta x 1 a2 9 a 3i
6 a p/2 6 2 p/2 p 8
b2 4 b 2 c2 a2 b2 2 c 5 .
I l’equació de la paràbola és:
Per tant, els focus són F (1, 5 ) i F (1, 5 ); els vèr-
texs són A (1, 3), A (1, 3), B (2, 3) i B ( 2, 3) i (y 1)2 2 · 8 · (x 2) (y 1)2 16(x 2)

l’excentricitat és e c/a 5 3. 9. Calculem els punts d’intersecció entre la paràbola i la hipèr-


x2 y2 bola resolent el sistema que formen les seves equacions:
b) 2x 2 y2 4 + =1 Com que el denominador
2 4 y 2 = 9x
x = 16, 94; y = 12, 35
de x 2 és més petit que el de y 2, els focus estan situats so- x2 y2
bre l’eix OY a2 4 a 2 i b2 2 b 2 c 2  =1 x = 16, 94; y = 12, 35
16 9
a2 b2 2 c 2.
A = (16, 94; 12, 35 )
Per tant, els focus són F (0, 2)iF (0, 2 ); els vèr-
B = (16, 94;12, 35 )
texs són A (0, 2), A (0, 2), B ( 2 , 0) i B ( 2 , 0)
i l’excentricitat és e c/a 2 2.
10. a) 9x 2 4y 2 36 0 Com que A 9 4 B i tenen el
mateix signe, aleshores la cònica correspon a una el·lipse.
6. a) El valor de c és la distància entre el centre i el focus.
c = d(C,F ) = ( 3 2)2 + (0 0)2 = 5 x2 y2
9x 2 4y 2 36 0 + =1
4 9
A partir de l’excentricitat, tenim que:
Si observem l’equació, veiem que el centre és C (0, 0) i,
e c/a 5/4 5/a a 4 a2 16. com que el denominador de x 2 és més petit que el de y 2,
De la relació c 2 a2 b 2 tenim que b 2 c2 a2 25 tenim que a 2 9 a 3, b 2 4 b 2. Finalment,
de la relació a 2 b 2 c 2 resulta que c 2 4 9 13
(x + 3)2 y2
16 9. I l’equació de la hipèrbola és = 1. c 13 .
16 9
b) (x 2)2 (y 3)2 1 x 2 y 2 4x 6y 12 0
b) La hipèrbola està centrada en el punt mitjà dels dos vèr-
Aquesta equació correspon a una circumferència, ja
texs:
que, si desenvolupem l’equació, A 1 B i tenen el mateix
1 6 2 2+2 signe.
C = ( A + A ') = , = ( 2, 2 )
2 2 2 En calculem els elements característics, que són el radi i
el centre. Si observem l’equació, veiem que el centre és
La distància focal és 10; aleshores, 10 2c c 5.
C (2, 3) i el radi és r 2 1 r 1.
D’altra banda, el valor de a correspon a la condició se-
c) y 3x 2 Com que A 3 i B 0, aleshores l’equació cor-
güent: 2a d (A, A ).
respon a una paràbola.
2a = (6 + 2)2 + (2 2)2 2a = 8 a=4 a 2 = 16
y 3x 2 x 2 y/3 Si observem l’equació, veiem que el
I de la relació tenim que
c2 a2 b2 b2 c2 a2 centre és C (0, 0), en què 1/3 2p p 1/6. Aleshores,
25 16 9. Per tant, l’equació de la hipèrbola resulta la la directriu és d: y p/2 1/18.
(x 2)2 (y 2)2
següent: =1 11. a) Com que el vèrtex és V (11, 121/24), l’altura és de
16 9 121/24 metres.

135
BLOC 2. GEOMETRIA > UNITAT 9. CÒNIQUES

b) La pilota tocarà el terra quan y sigui 0. Ara, apliquem el teorema de Pitàgores per obtenir el semieix
x1 = 0 menor b = a 2 c 2 = 4,482 060 UA b = 6,71 108 km
121
( x 11)2 = 24 y = 121 en què 1 UA 149 597 870,7 km.
24 x 2 = 22
— Antenes parabòliques
c) Com que la porteria és a 20 metres del jugador, substituïm
t Resposta suggerida:
aquest valor per x en l’equació de la paràbola per trobar el
valor de l’altura a la qual es troba la pilota. Els cables dels ponts penjants tenen forma parabòlica; els
telescopis, els detectors de radar i els reflectors lluminosos
(20 11)2 24(y 121/24) y 5/3 1,67 metres
són parabòlics; alguns marcs fotogràfics tenen forma el-
Aleshores, la pilota entrarà a la porteria, ja que 1,67 < 2,40. líptica.
— Ovals, ovoides i el·lipses
Zona (pàg. 231) t Resposta suggerida:
— La papiroflèxia i les còniques Oval: corba tancada i plana formada per quatre arcs de cir-
cumferència tangents entre si i iguals dos a dos.
t Resposta suggerida:
Ovoide: corba tancada i plana formada per quatre arcs de cir-
Hipèrbola:
cumferència tangents entre si, dos d’iguals i dos de desiguals.
http://links.edebe.com/u29nas
Es poden relacionar amb una el·lipse.
El·lipse (mètode del jardiner):
t Resposta suggerida:
— Mark Twain i el cometa Halley
Coincideixen que són corbes tancades i planes formades per
t Calculem en primer lloc la semidistància focal, c, a partir de arcs de circumferència tangents entre si. Es diferencien en el
l’excentricitat, e, i el semieix major, a: fet que l’oval està compost per un nombre parell d’arcs de cir-
cumferència enllaçats entre si i simètrics respecte als seus ei-
c xos major i menor normals entre si; i l’ovoide està compost per
e = c = e a = 0,967 990 17,857 619 =
a dos arcs del mateix radi, i dos més de radi diferent, i té un eix
= 17,285 997UA de simetria que conté els centres dels arcs desiguals.

136
BLOC 3. ANÀLISI

10# Funcions de variable real

En context (pàg. 237) b) f ( x ) = e x +1


a) La catenària és la forma que té un cable subjecte de dos L’equació té valor per a qualsevol valor de x. Per tant:
punts, com per exemple els cables elèctrics; l’equació que D (f ) = !
la descriu és:
Una exponencial de base positiva sempre donarà un valor
1 positiu; per tant, el recorregut és: D (f ) = (0, + )
y = cos h ( ( 2x a ) ) cos h ( a )
2 1
3. a) h g = h ( g ( x ) ) =
En canvi, la fórmula de la paràbola és: e x 2
1 1
y = x2 b) h f g = h (f ( g ( x ) ) ) = =
(e 2x
) 2 2 e 2x 4
b) Gaudí també dissenyava amb formes helicoïdals, el·lipses,
cercles, etc. c) f g = f ( g ( x )) = e 2x 2
2
c) Resposta oberta a manera de reflexió personal. d) f g h = f ( g (h ( x ) ) ) = e x 2 2

4. a) N = N 0 · 2kt 6 = 24k log 6 = 4k log 2


Llenguatge matemàtic (pàg. 238)
log 6
Resposta oberta a manera de reflexió personal. k = = 0,65
4 log 2

b) N (Bacteris)
Fixa-t’hi (pàg. 239) 37 N (t )
Per poder-la considerar una funció, per a cada valor de x no-
més hauria d’existir un valor de f (x), cosa que no passa, per
exemple, en x 4.
15

Problemes resolts (pàgs. 252 i 253) 6

O 2 4 6 8 t (hores)
1. a) V (m3)
4,8
4,6

Exercicis i problemes (pàgs. 254 a 258)

2,8

2 1 CONCEPTE DE FUNCIÓ Pàg. 254

5. a) f (x) x3 4x 6

O 800 1000 2000 2500 h (m)


f (0) 6 f ( 2) 6 f (3) 21
b) f ( x ) = x 3
D (f) [0, 2 500]
f (0) N.E f ( 2) N.E. f (3) 0
R(f) [2; 4,8]
c) f (x) ln(x 1)
Creix en (0, 800) i (1 000, 2 000), experimenta un màxim
f (0) 0 f ( 2) N.E. f (3) 1,39
en (2 000; 4,8) i decreix en (2 000, 2 500).
x 1
b) V (h 400) 2,4m3 d) f ( x ) =
x +2
2
2. a) f (t ) = f (0) 0,5 f ( 2) N.E. f (3) 0,4
t 1
Busquem el punt on s’anul·la el denominador: 6. a) f (x) 3x 2
t 1 0 t 1
x f (x)
Per tant, el domini és: D (f ) = ! {1} 1 1
El recorregut és: 0 2

R (f ) = R { 0} 1 5

137
BLOC 3. ANÀLISI > UNITAT 10. FUNCIONS DE VARIABLE REAL

Figura 3
Figura 86
Y c) V t) 9,8 · t
3
V
10
2 f (x )
8
1 6

4
–2 –1 O 1 2 X
2
–1

–6 –4 –2 O 2 4 6 8 t
–2 –2

–3 –4

–6

b) f (x) x2 4
5
11. a) f ( x ) =
x f (x) x 3
Busquem el punt on s’anul·la el denominador:
4 12
x 3 0 x 3
2 0
Per tant: D (f ) = ! { 3}
0 4
Com que el numerador mai no pot ser 0, tampoc no ho
2 0 podrà ser f (x). Per tant:
4 12 R (f ) = ! { 0}
Figura 4
b) f ( x ) = x 2
Y
2 El radicand no pot ser negatiu, així que:
f (x )
1 x 2≥0 x≥2

O
D (f ) = [ 2, + )
–3 –2 –1 1 2 3 4 X
–1
Com que és una arrel, f (x) només pot prendre com a valors
–2 0 o valors positius, R (f ) = [ 0, + ) .
–3
2
12. f ( x ) =
–4 x 4
a) f (2) 1 f (4) N.E. f (6) 1
7. a) f (x) 2,5 0,2x b) f (x) 0,25
b) f (t ) = N 0 · e t
2
= 0, 25 8=x 4 x = 12
x 4
8. f (x) 0 no existeix.
f (x) 1 s’esdevé quan x 0. f (x) 1

f (x) 1 s’esdevé quan x 1. 2


= 1 2=x 4 x =2
x 4
9. Considerem els punts de tall de la recta amb els eixos de co-
ordenades; són ( 6,0) i (0, 3), de manera que: f (x) 0,5
2
x x1 y y1 x +6 y = 0, 5 4=x 4 x =8
= = y = 0,5x + 3 x 4
x2 x1 y2 y1 6 3
Figura 85
13. a) f ( x ) = ( x 3) ( x + 3)
10. a) f (x) 3 0,8x
Sabem que el radicand no pot ser negatiu i que els punts
Y que cal estudiar són 3 i 3.
4
3
x
2
4 3 0 3 4
1
x 3 0
–6 –5 –4 –3 – 2 –1 O 1 2 3 X
–1 x 3 0
–2
(x 3)(x 3) 0 0

b) No és una funció. D (f ) = ( , 3] [ 3, + )

138
BLOC 3. ANÀLISI > UNITAT 10. FUNCIONS DE VARIABLE REAL

Figura 6
x 16. a) f (x) e 2x
b) f ( x ) = cos x D (f ) = !
2 Y
x 3
c) f ( x ) = D (f ) = ! {k ,k !}
sin x 2

4x
d) f ( x ) = 1

x3 x
6 X
Busquem els punts on s’anul·la el denominador: –3 –2 –1 O 1 2 3 4 5
–1

x1 = 1 –2

x3 x =0 x2 = 0 –3

x3 = 1 Figura 7
b) f (x) x3 2x 3
D (f ) = ! { 1, 0,1} Y
5

4
14. a) f ( x ) = x 2 + 2x + 1
3
El radicand no pot ser negatiu; per tant: 2

x2 2x 1 0 (x 1)2 0 1

2 1 2 –5 –4 –3 –1 O 1 2 3 4 X
–1
x 1 0 –2
Figura 8
(x 1)2 0
x2 2x 1 0 c) f (x) sin(2x)
Y
D (f ) = ! 3
2
Com que és una arrel, f (x) només pot prendre com a valors
1
0 o valors positius: R (f ) = [ 0, + )
x +2 –3 –2 –1 O 1 2 3 4 5 X
b) f ( x ) = –1
4 x2 –2
Busquem els punts on s’anul·la el denominador: Figura 9
x1 = 2 d) f (x) cos(x 2)
x2 4=0
x2 = 2 Y
2
D (f ) = ! { 2, 2} 1

Per trobar el recorregut, representem gràficament la fun-


Figura 5
ció: –5 –4 –3 –2 –1 O 1 2 3 4 5 X
Y –1
6
f (x )
–2
4
Figura 10
2
e) f (x) ln (x 2 1)
O

– 10 –8 –6 –4 –2 2 4 6 8 X Y
3
–2
2
–4 1

–6
–6 –5 –4 –3 –2 –1 O 1 2 3 4 5 X
–1
–8
–2
Figura 11
–3
R (f ) = R { 0}
f) f (x) x2 1
15. Veiem que es tracta d’una paràbola. Tindrà la forma f (x) Y
ax 2 bx c; a més a més, sabem que passa per ( 2, 0), 5

(0, 2) i (2, 0). 4

3
2
a ( 2 ) + b( 2) + c = 0 2

a · 02 + b · 0 + c = 2 f (x) = x 2 4
2 –3 –2 –1 O 1 2 3 X
a ( 2 ) + b(2) + c = 0

139
BLOC 3. ANÀLISI > UNITAT 10. FUNCIONS DE VARIABLE REAL

17. Les gràfiques de la dreta no corresponen a funcions, ja que 22. Consisteix a intercanviar la f (x) per la x i, posteriorment, aïllar
per a alguns valors de x tenen assignats més d’un valor de la nova f (x), que serà la funció inversa.
f (x); ho podem comprovar amb x 4 en la gràfica superior x
a) f (x) 2x 6 f 1 ( x ) = +3
dreta, i amb x 3 en la gràfica inferior dreta. En la gràfica su- 2
perior esquerra: D (f ) = R i R (f ) = [ 0, + ) . En la gràfica infe- b) f (x) x f (x)1 x
rior esquerra: D (f ) = R i R (f ) = [ 1,1] . c) f ( x ) = x2 16 f 1
( x ) = ± x 2 + 16
2x 5x
d) f ( x ) = f 1
(x ) =
x 5 x 2
2 OPERACIONS AMB FUNCIONS Pàg. 255
x 4 r3
18. f ( x ) = x + 2 g (x ) = x =4 23. V ( r ) =
x2 8 3
Figura 12
a) g (x) f (x) 0,5 6 5,5 3r
V 1
(r ) = 3
4
b) f (x) · g (x) 6 · 0,5 3
4 V V (r )
c) (f g ) (x ) = + 2 = 2,5
42 8 1,2

6
d) ( g f ) ( x ) = = 0,21 1,0
62 8
0,8

2 0,6
19. f ( x ) = x 2 1 g ( x ) = 2x + 3 h (x ) = V – 1 (r )
x +1 0,4
a) f (x) g (x) x2 1 2x 3 x2 2x 2
0,2

2 x3 + x2 x +1
b) f ( x ) + h ( x ) = x2 1+ = O
x +1 x +1 0,2 0,4 0,6 0,8 1,0 1,2 1,4 r

2 2x 2 5x 1
c) h ( x ) g (x ) = 2x 3=
x +1 x +1
2 2x 2 + 5x + 1
3 TRANSFORMACIÓ DE FUNCIONS Pàg. 255
d) g ( x ) h ( x ) = 2x + 3 =
x +1 x +1 24. f (x) x 2
e) f ( x ) · g ( x ) = ( x 2 1) ( 2x + 3 ) = 2x 3 + 3x 2 2x 3 a) f (x) 2
2 4x + 6 b) f (x 4)
f) g ( x ) · h ( x ) = ( 2x + 3 ) · =
x +1 x +1 Figura 13
c) f (2x)
f (x ) x2 1
g) = d) f (0,5x)
g (x ) 2x + 3
g (x ) 2x + 3 Y f (2x )
h) = 5
f (x ) x2 1 f (x )
4

3
x2 4
20. f ( x ) = x2 4 g (x ) = 2
x f (x + 4) f (0,5 x)
x2 4 1
a) f ( x ) g (x ) = x2 4
x –7 –6 –5 –4 –3 –2 –1 O 1 2 3 4 5 X
3
( x2 4) 2 –1
b) f ( x ) · g ( x ) = f (x ) – 2
x –2

Figura 17
2

c) (f g ) (x ) =
(x2 4) 4x 2
=
x4 12x 2 + 16 25. f (t) 5 3t 4,9t 2
x2 x2
a) Y
x2 8
d) ( g f ) ( x ) = 5
x f (t )
4

3
21. f ( x ) = cos x g (x ) = x 2
2
a) (f g ) ( x ) = cos ( x 2 ) 1

b) ( g f ) ( x ) = cos x 2
–2 –1 O 1 2 3 t
c) ( g g ) (x ) = x 2 2 –1

d) (f g f ) ( x ) = cos ( cos x 2 ) –2

140
BLOC 3. ANÀLISI > UNITAT 10. FUNCIONS DE VARIABLE REAL
Figura 18

Figura 22
b) Y b) f (x 3)
5 Y
4
f (t + 2 ) 4

3
3

2
2

1
1

–5 –4 –3 –2 –1 O
1 X
–4 –3 –2 –1 O 1 t
–1
–1
Figura 19
–2 Figura 23
c) f ( x)
c) Y Y
4
5
f (–t ) 3
4
2
3
1
2

1 –2 –1 O 1 2 X
–1

–2 –1 O 1 2 t
Figura
–1
20
Figura 24
d) f (x)
–2
Y
1
d) Y
– f (t )
–2 –1 O 1 2 X
1
–1

–2 –1 O 1 2 t –2

–1
–3

–2
–4

–3

–4

–5

4 CARACTERÍSTIQUES
D’UNA FUNCIÓ Pàgs. 255 i 256
26. Vermella f (x) x2
28. a) f (x) 4x
Marró f (x) 4x 2 Busquem el valor de f (x) quan x 0:
Groga f (x) (x 1)2 f (0) 0

Negra f (x) x2 2 Per tant, el punt de tall és (0, 0).


b) f (x) 3x 2
Blava f (x) (x 2)2 2
Figura 21
Calculem el valor de f (x) quan x 0:
27. La funció de la gràfica és f (x) 3x 2 1.
f (0) 2
a) f (x) 2
Per tant, el punt de tall és (0, 2). Busquem ara el valor de
Y x quan f (x) 0:
3

2
3x 2 0 x 2/3
1 Per tant, el punt de tall és (2/3, 0).

–3 –2 –1 O 1 2 3 X
c) f (x) x2 3x 2
–1
Calculem el valor de f (x) quan x 0:
–2

–3
f (0) 2
–4 Per tant, el punt de tall és (0, 2).

141
BLOC 3. ANÀLISI > UNITAT 10. FUNCIONS DE VARIABLE REAL

Figura 28

Busquem el valor de x quan f (x) 0: b) f (x) x2 4


Y
x1 = 1
x2 3x + 2 = 0 x = 8
x2 = 2 6

4
Per tant, els punts de tall són (1, 0) i (2, 0).
2

d) f (x) x2 16 –6 –4 –2 O 2 4 6 8 X
–2
Calculem el valor de f (x) quan x 0: –4

f (0) 16 Figura 29
Mínim en (0, 4)
Per tant, el punt de tall és (0, 16).
Busquem el valor de x quan f (x) 0:
31. Y
4
x1 = 4 f (x )
x2 16 = 0 x =
x2 = 4 3

2
Per tant, els punts de tall són ( 4, 0) i (4, 0).
1

29. f (x) ex
Figura 25 –2
Figura
–1 O 30 1 2 3 4 5 6 7 X
a) f ( x)
–1
Y
10

8 32. Y
4
6 3 f (x )
4 2
1

–4 –3 – 2 –1 O 1 2 3 4 5 6 7 8 X
–4 –2 O 2 4 6 8 10 X –1
–2 –2

Figura 26
b) f (x) 33. a) La funció és creixent en tot el seu domini.
Y Els punts de tall són ( 0, 0 ) , ( , 0 ) , ( 2 , 0 ); no té màxims ni
mínims.
2
3
O b) La funció creix entre 2 , ,0 ( 0, 2 ), i
– 10 –8 –6 –4 –2 2 4 X
2 2
–2 3
decreix en , ( 2, + ). Els punts de tall són
–4
2 2
( 2 , 0), ( , 0), (0, 1) i (5, 0).
–6

–8
34. a) f (x) 4x 2 16: funció parella.

– 10
b) f (x) sin2x: funció imparella.
c) f (x) 2x 3 x: funció imparella.
2
Figura 27 d) : funció parella.
30. a) f (x) x 2 3x 2 x2 1
Figura 311
Y 35. a) f ( x ) = 2 +
10 x2
8 Y
10
6
8

2 4

–4 –2 O 2 4 6 8 X
–2 – 10 –8 –6 – 4 –2 O 2 4 6 8 10 X
–2

Mínim en (1,5, 0,25) Asímptotes x 0, y 2.

142
BLOC 3. ANÀLISI > UNITAT 10. FUNCIONS DE VARIABLE REAL

4x 3 + 1 38. a) f (x) x 2 5x 4
b) f ( x ) =
Figura 32 x2 Com que és una funció polinòmica D(f ) = !. El vèrtex de
b 5
Y la paràbola es troba en xv = = i correspon a un
2a 2
8 mínim absolut, ja que a 0. La funció decreix en
6 ( ∞, 2,5) i creix en (2,5, ∞). La funció talla els eixos en
4
(0, 4), (1, 0) Figura
i (4, 0).35
2 Y
O 4
f (x )
–4 –2 2 4 X 3

–1 O X
Asímptotes x 0, y 4x –1
1 2 3 4 5 6 7

–2
3
c) f ( x ) =
x +2 Figura 33
b) f ( x ) = 2x 8
Y
10 El radicand ha de ser positiu o nul:
8
2x 8 0 x 4
6

4
Per tant, D(f ) = [ 4, + ) . El recorregut d’una arrel quadra-
da simple és: R(f ) = [ 0, + ) .
–12 –10 –8 –6
–4 –2 O 2 4 6 8 10 X
La funció no presenta extrems relatius ni absoluts, i pre-
–2 senta punts de tall en (4, 0).
Figura 36
–4 Y
4
–6
3
Figura 84
f (x )
2
Asímptotes x 2, y 0
1

36. a) Preu (€)


–1 O 1 2 3 4 5 6 7 8 X
260
–1
250

240
x +1
230 c) f ( x ) =
Oferta B 2
220

210 Oferta A
Com que és una funció polinòmica, sabem que D(f ) = R i
200
R(f ) = R. La funció és decreixent en tot el seu domini i no
1 2 3 4 5 6 7 8 9 10 11 12 presenta extrems absoluts ni relatius; talla els eixos en els
Mesos Figura 37
punts (0, 1/2) i (1, 0).
b) L’oferta A és mes favorable els 5 primers mesos. A partir de Y
6
llavors és més favorable l’oferta B. f (x )
4
c) La diferència és màxima en el moment de comprar l’ordi-
nador i és de 20 €.Figura 34 2

–10 –8 –6 –4 –2 O 4 6 X
2
37. a) X (km) –2

120

d) f ( x ) = ln x + x

60 El contingut d’un logaritme mai no pot ser negatiu ni nul;


per tant, D(f ) = ( 0, + ) i R(f ) = !. La funció sempre és
30
creixent i no presenta extrems
Figura 38 absoluts ni relatius; talla
amb els eixos de coordenades en (0,567; 0).
Punt de 60 80 130 160 315 t (min)
partida Y
3
f (x )
b) D (f) [0, 5 h 15 min] R(f) [0, 60] 2

1
c) Els punts de tall són (0, 0) i (5 h 15 min, 0).
d) Creix en (0, 1), (1 h 20 min, 2 h 10 min) i decreix en (2 h –1 O 1 2 3 4 X
–1
40 min, 5 h 15 min).
–2

e) No té màxims ni mínims relatius.

143
BLOC 3. ANÀLISI > UNITAT 10. FUNCIONS DE VARIABLE REAL

41. En el cas lineal, sabem que passa per (0, 0) i (1, 4):
5 TIPUS DE FUNCIONS Pàgs. 256 i 257
x x1 y y1 x 0 f (x) 0
Figura 39 = =
39. a) f (x) 2x x2 x1 y2 y1 1 0 4 0
Y
f ( x ) = 4x
3
En el cas de la interpolació quadràtica, sabem que el vèrtex es
2 troba en (0, 0). La posició del vèrtex és determinada per
1 b
xv = ; per tant, b ha de ser 0. Així:
2a
–3 –2 –1 O 1 2 3 X
a · 02 + c = 0
h ( x ) = ax 2 + cFigura 44 h ( x ) = 4x 2
–2 a · 12 + c = 4
–3
Y
5 f (x )
b) f (x) 3x 6 Figura 40 4

3
Y
2
6 h (x )
1
4
–2 –1 O 1 2 3 X
2 –1

–2
–4 –2 O 2 4 6 X
–2
Les expressions algèbriques són y 4x i y 4x 2.
–4
Figura 41 42. Sabem que la funció passa pels punts (0, 3) i (3/2, 0); així:
c) f (x) 5x 2 6x 7 x x1 y y1 x 0 f (x) 3
= =
x2 x1 y2 y1 3 0 0 3
Y 2
12
f ( x ) = 2x + 3

10 Figura 45
43. a) f (x) ex
8
Y
5
6
4
4
3
2
2

1
–2 O X
Figura 242 4

–6 –5 –4 –3 –2 –1 O 1 2 3 4 X
d) f (x) 0,5x 2 2 –1

Y Figura 46
b) f (x) ln x
12
Y
10
4
8
2
6

4
–2 O 2 4 6 X
–2

–8 –6 –4 –2 O 2 4 6 8 X –4
Figura 43

40. A = r 2
44. a) f (x) cos(2x)
A
12
f ( /2 ) = 1 f ( 3 /2 ) = 1 f( )=1
10

8 b) f (x) sin2x
6
f ( /2 ) = 1 f ( 3 /2 ) = 1 f( )=0
4

2 c) f (x) tg2x

O 2 4 6 8 r f ( /2 ) N.E . f ( 3 /2 ) N.E . f( )=0

144
BLOC 3. ANÀLISI > UNITAT 10. FUNCIONS DE VARIABLE REAL

Figura 51
d) f (x) cosec x 49. a) f (x) log x
f ( /2 ) = 1 f ( 3 /2 ) = 1 f( ) N.E . Y
2

1
4 r3
45. V ( r ) = Figura
. Com que47
el radi és una distància, no pot ser O
3 –5 – 4 –3 –2 –1 1 2 3 4 X
–1
negatiu; això implica que D (f ) = [ 0, + ).
–2
V
–3
2 Figura 52
V (r )
1,5
b) f ( x ) = 2x 1
1 Y
3
0,5
2

O 1
– 1,5 –1 – 0,5 0,5 1 1,5 r

–4 –3 –2 –1 O 1 2 3 4 5 X
–1
x si x 0
Figura 53
46. a) f ( x ) = 2 si 0 < x 2
c) f (x) x 3
x +1 si x 3
Y
5
b) Decreix en ( , 0 ] i creix en [ 3, + ) . La funció no té mà-
4
xims ni mínims i talla els eixos en el punt (0, 0). 3
Figura 48 2
47. a) f ( x ) = sin ( x ) 1

Y –1 O 1 2 3 4 5 6 7 8 X
–1
2
Figura 54
1
d) f (x) x 3
–1 O 1 2 3 4 5 6 7 8 X Y
–1 5

–2 4

3
–3
2
Figura 49
1
b) f (x) tg (2x)
– 8 –7 – 6 –5 – 4 –3 –2 –1 O 1 X
Y –1
Figura 55
2

1 50. a) a (c ) = c 2 + 9
–1 O 1 2 3 4 5 6 7 X a
8

–2 6

4
a (c )
–3

x +2 si x 1 –6 –4 –2 O 2 4 6 8 10 c

48. f ( x ) = 3 Figurasi50
1< x 3 Figura 56
2x 3 si x > 3 b) c ( a ) = a2 9
c
Y 5

5 4
c (a )
3
4
2
3
1
2

1 –1 O 1 2 3 4 5 6 7 a
–1

–4 –3 –2 –1 O 1 2 3 4 5 X
–1
c) [3, + )

145
BLOC 3. ANÀLISI > UNITAT 10. FUNCIONS DE VARIABLE REAL

51. a) Any Sou 52. a) E 6,62 · 10 Figura 59representació corresponent tindrà un


34 · f. La

aspecte semblant a:
0 20 000
E
1 20 600 (10 -19 J)
6
2 21 218 5 E (f )

4
3 21 854,54
3
4 22 510,17 2

5 23 185,48 1

O 8 f (1014 Hz)
6 23 881,04 1 2 3 4 5 6 7

7 24 597,47 b) N = N 0 · e ktFigura 60
. La representació corresponent tindrà un as-
8 25 335,40 pecte semblant a:
Figura 57
N
N0
b) Sou
(milers d'€)
26

25
N (t )
24 N0/2

23

22

21 O t t
20
Figura 61corresponent tindrà un as-
c) T = k R 3 . La representació
O 1 2 3 4 5 6 7 8 t (anys)
pecte semblant a:
c) 200 000 · x 8 = 25 000 x = 1,0283 i = 2,83 % T
5

4
d) Calculem el sou de cada any en aquest cas.
3
T (R )
2
Any Sou
1
0 20 000
–1 O 1 2 3 4 5 R
–1
1 20 650

2 21 300 k
d) F = . En el cas de la Terra i la Lluna, el valor de k és de
3 21 950 R2 Figura 62
2,93 · 1037. La representació corresponent tindrà un as-
4 22 600
pecte semblant a:
5 23 250 F

6 23 900
F (R )
7 24 550

8 Figura 5825 200

Sou R
(milers d'€)
26

25
53. Descriu l’oscil·lació d’un pèndol al llarg del temps. L’amplitud
24
d’oscil·lació és de 3 cm.
23

22

21

20 6 INTERPOLACIÓ I
O 1 2 3 4 5 6 7 8 t (anys)
EXTRAPOLACIÓ Pàgs. 257 i 258

x x1 y y1 3
e) Els sis primers anys haurà cobrat més amb la segona ofer- 54. = f (x ) = x
x2 x1 y2 y1 2
ta.
a) f (3) 1,5 · 3 4,5
f) Sumant els sous de les dues opcions, comprovem que al
treballador li hauria interessat més la segona opció i a l’em- b) f (4,5) 1,5 · 4,5 6,75
presa, la primera. c) f (5) 1,5 · 5 7,5

146
BLOC 3. ANÀLISI > UNITAT 10. FUNCIONS DE VARIABLE REAL

55. a) f (0) 1,5 · 0 0 x 3 Figura 89 x 1


b) AB y = + BC y = +
b) f (20) 1,5 · 20 30 2 2 2 2
Y
56. Una interpolació cúbica tindrà la forma y ax 3 bx 2 cx 3

d. Hem de trobar, per tant, 4 incògnites (a, b, c i d). Per a A C


2
això, necessitem 4 equacions que seran proporcionades per
4 punts. 1
B
x x1 y y1 x 15 y 6
57. a) = = –3 –2 –1 O 1 2 3 4 X
x2 x1 y2 y1 35 15 14 6 –1

2x
y = Figura 63
5 AB y x 1 BC 65
Figura y 3

b) Y Y
4 3 C
B
y=2x/5 A 2
2

1
–4 –2 O 2 4 6 8 10 X
–2
–2 –1 O 1 2 3 X
–1
2 5
c) y = x x = y F = 21,25 N
5 2
m m 60. a) y 1 = ax12 + bx1 + c 5=c
d) a ( 50 N) = 20 a ( 80 N) = 32
s2 s2 y 2 = ax 22 + bx 2 + c 4 = a +b +c
y 3 = ax 32 + bx 3 + c 7 = 4a + 2b + c
58. y 1 = ax12 + bx1 + c 3=c f (x) = 2x 2 3x + 5
y 2 = ax 22 + bx 2 + c 1= a +b +c
b) f (3) 14
y 3 = ax 32 + bx 3 + c 9 = 9a + 3b + c
c) g (x) és la funció lineal que passa per (0, 5) i (1, 4).
f (x) = 2x 2 4x + 3
x 0 g (x) 5
a) f (0,5) 1,5 = g (x ) = x + 5
1 0 4 5
b) f (4) 19
h (x) és la funció lineal que passa per (1, 4) i (2, 7).

59. a) 2 = a b + c Figura 64 x 1 h(x) 4


x2 x 5 = h ( x ) = 3x + 1
1= a +b +c f (x) = + 2 1 7 4
4 2 4
2 = 9a + 3b + c
d) g (5)Figura
0 66h (5) 16
Y
5 e) Y
10
4
9
3 8
2 7 C
1 6
A
5

–4 –3 –2 –1 O 1 2 3 4 5 X 4
–1 3 B
2
1
2=a b +c
x2 2x –2 –1 O Figura
1 2 367
4 X
3=c f (x) = + +3
Figura
3 883
3 = 4a + 2b + c Y
16
Y 14
4
12
3
10
2
8
1
6

–4 –3 – 2 –1 O 1 2 3 4 5 X 4
–1
2
–2
–6 –4 –2 O 2 4 6 8 X

147
BLOC 3. ANÀLISI > UNITAT 10. FUNCIONS DE VARIABLE REAL

62. a) Si la funció és contínua i f (a) 0 i f (b) 0, significa que la


SÍNTESI Pàg. 258
funció passa de tenir valor positiu a valor negatiu. Per tant,
en efectuar aquesta transició ha de tallar l’eix d’abscisses
61. a) D (f ) = ! R (f ) = [ 25, + ) en un punt anomenat c. Així, f (c) 0.
Figura
D 68
(g ) = ! R (f ) = ( 0, + )
b) Si la funció és contínua i no constant, perquè f (a)
b) Y f (b) la funció ha de decréixer i després créixer, o vicever-
15
sa; per tant, existeix algun extrem relatiu.
10

5 f (x )
63. a) f ( 2 ) = 0 f ( 4 ) = 1, 41
–15 – 10 – 5 O 5 10 15 X
–5
g ( 2 ) = 1,098 g ( 5 ) = 1,609
– 10

– 15
b) Estudi de f (x):
– 20Figura 69
–25
D (f ) = [ 2, + ) R (f ) = [ 0, + )

Y És creixent en tot el seu domini i no presenta extrems. Talla


60
els eixos en (2, 0). No té simetria ni periodicitat.
40
g (x ) Estudi de g (x):
20

D ( g ) = ( 1, + ) R (g ) = !
–60 –40 –20 O 20 40 X
–20
És creixent en tot el
Figura 72seu domini i no presenta extrems. Talla
els eixos en (0, 0). No té simetria ni periodicitat.
c) (f e 2x 25
g ) = Figura 70 (g f ) = ex 2 25

c) Y
d) Y 5
15
4
10
3 f (x )
( f º g ) (x )
5
2

– 20 –15 –10 –5 O 5 10 15 X 1
–5

–10 –2 –1 O 1 2 3 4 5 6 7 8 9 X
–1
Figura 73
–15

–20
Figura 71
Y
g (x )
2

Y –6 –4 –2 O 2 4 6 8 10 X
20 –2
( g º f ) (x )
15 –4

10 –6

–8
5

–10 –5 O 5 10 15 20 X d) x 2 = ln ( x + 1) x = 5,51 ( 5,51; 1,87 )


e) La funció f (g (x)): e) (f g) = ln ( x + 1) 2

D (f g) = ! R (f g ) = [ 25, + ) f) Estudiem la funció (f g ) :


El radicand no pot ser negatiu; per tant:
No presenta màxims ni mínims i és creixent en tot el seu
domini. ln ( x + 1) 2 0 x 6,389

La funció g (f (x)): D (f g ) = [ 6,389, + ) R (f g ) = [ 0, + )


D (g f ) = ! R ( g f ) = ( 0, + ) La funció és creixent en tot el seu domini i no presenta ex-
trems. Talla l’eix d’abscisses en (6,389; 0).
Decreix en ( , 0 ) , presenta un mínim en (0, 0) i creix en
( 0, + ). 5
64. f ( x ) =
f) Resposta suggerida: 3x 2

Escollim dos punts qualssevol de f (x), com ( 5, 0) i (5, 0). a) f ( 2) 0,625; f (1) 5; f (2) 1,25

x ( 5 ) = y 01 b) f (x) 1 x 2,33; f (x) 5 x 1


f (x ) = 0
5 ( 5) 0 0 c) D (f ) = ! { 2/ 3} R (f ) = ! { 0}

148
BLOC 3. ANÀLISI > UNITAT 10. FUNCIONS DE VARIABLE REAL

Figura 74

d) Y És una funció creixent en ( 3, 2), assoleix un màxim en (2, 6)


8
f (x )
i decreix en (2, 5). Els punts de tall són ( 2, 0), (5, 0) i (0, 2),
6
i no presenta periodicitat.
4

2 b) D (f ) = ! R (f ) = [ 1,1]

–10 –8 –6 –4
–2
O
2 4 6 8 10 X La funció creix en ( /2 + k , + k ) , assoleix màxims en
–4
els punts ( + k ,1), decreix en ( + k , /2 + k ) i pre-
–6 senta mínims en ( /2 + k , 1). Els punts de tall amb els
–8 eixos es produeixen en ( 0,1) , ( / 4 + k /2, 0 ) . Es tracta
d’una funció periòdica de període .
Figura 76
e) La funció decreix en tot el seu domini i no presenta màxims
ni mínims. Té un punt de tall amb l’eix en (0, 2,5). No pre- 3. a) X (km)
42
senta periodicitat ni simetria. 41

34
f) Interpoladors lineals a trossos:
g (x) és el polinomi interpolador que passa per ( 2, 0,625)
i (1, 5): 18

x +2 g (x) + 0,625
= g (x) = 5,625x + 8,75
1+ 2 5 + 0,625 Figura 77
h (x) és el polinomi interpolador que passa per (1, 5) i O 1h 2h 215’’ 245’’ 3h t
255’’
(2, 1,25):
V (km/h)
x 1 h(x) 5
= h(x) = 3,75x + 8,75 18

2 1 1,25 5 16
14
Polinomi interpolador quadràtic: 12

y 1 = ax12 + bx1 + c 0,625 = 4a 2b + c


y 2 = ax 22 + bx 2 + c 5 = a +b +c
y 3 = ax 32 + bx 3 + c 1,25 = 4a + 2b + c O 1h 2h 215’’ 245’’ 3h t
255’’

f (x) = 1,406x 2 + 0,468x + 5,937


b) D (f ) = [ 0, 3 ] D ( g ) = [ 0, 3 ]
Figura 75
65. a) f ( 3 ) = 5 f ( 7 ) = 8,062 R (f ) = [ 0, 42 ] R ( g ) = [ 0,18 ]

b) D (f ) = ! R (f ) = [ 4, ) La funció f (x) és contínua en tot el seu domini i creix en els


intervals (0, 2 h), (2 h 15 min, 2 h 45 min) i (2 h 55 min,
c) Y 3 h).
10
f (x )
La funció g (x) és discontínua per als següents valors de x:
5 1 h, 2 h, 2 h 15 min, 2 h 45 min i 2 h 55 min. No hi ha inter-
vals de creixement ni decreixement, ja que la funció és cons-
–15 –10 –5 O 5 10 15 20 X tant en tot el seu recorregut excepte en les discontinuïtats.
–5
4
d) La funció decreix en ( , 0 ), assoleix un mínim en (0, 4), punt 4. F =
d2
que a més és l’únic tall amb els eixos. En ( 0, + ) creix. No
Figura 78
presenta periodicitat, però sí simetria respecte a x 0. a) D (f ) = ! { 0} R (f ) = ( 0, + )
e) f 1
(x ) = x2 16 b) F
10
f) El polinomi ha de passar per (3, 5) i (7; 8,062). F (d )
8

x 3 g (x) 5 6
= g (x) = 0,7655x + 2,7035 4
7 3 8,062 5
2

– 10 – 8 –6 – 4 – 2 O 2 4 6 8 10 d
Avaluació (pàg. 260) –2

4
c) F (d ) = 1 =1 d = ±2
1. a) D (f ) = ! d2
b) D (f ) = [ 3, 3 ] d) F ( 0 ) = N.E . F ( 1) = 4 F (1) = 4
F (100 ) = 0,000 4
c) D (f ) = ( 0, + )
d) D (f ) = ! { 0,1, 4} 5. f ( x ) = e x +1 y = e x +1
Per a efectuar la inversa, on abans teníem y hi posem x, i vice-
2. a) D (f ) = [ 3, 5 ] R (f ) = [ 1, 6 ] versa:
x = e y +1 ln x = y + 1 y = 1 + ln x

149
BLOC 3. ANÀLISI > UNITAT 10. FUNCIONS DE VARIABLE REAL

y 1 = ax12 + bx1 + c 10 = c
6. a) (f g ) ( x ) = (2x 2 + 5)3 4(2x 2 + 5) + 6 =
10. a) y 2 = ax 22 + bx 2 + c 0 = 9a + 3b + c
= 8x 6 + 60x 4 + 142x 2 + 111 0 = 81a + 9b + c
y3 = ax 32 + bx 3 + c
b) (f g ) ( 2 ) = 2151 10 82
Figura 40
f (x) = 2 x x + 10
c) ( g f ) ( x ) = 2(x 3 4x + 6)2 + 5 = 27 9

= x6 16x 4 + 24x 3 + 322 96x + 77 b) Y


8
f (x + 2)
d) ( g f ) ( 2 ) = 77 6

7. a) f (x) e2x Figura 79 2

–4 –2 O 2 4 6 8 10 X
Y
–2
20 Figura 83
–4
15

10
c) Y
5 35

30
– 20 – 15 –10 –5 O 5 10 15 20 X 25
–5
20

15
Figura 80 f (– x )
2 10

b) f ( x ) = 5
3x
–15 –10 –5 O 5 10 15 X
Y
3

2
f (x )
Zona (pàg. 261)
1
–6 –5 –4 –3 –2 –1 — Gràfiques sense esforç
O 1 2 3 4 5 X
–1
t Resposta oberta a manera de reflexió personal.
–2

–3 t Resposta oberta a manera de reflexió personal.

— El risc d’extrapolar
8. a) f (t) serà el polinomi que passa per (3, 37) i (5, 25). x x1 y y1 x 1, 3 y 5,2
= =
t t1 f (t ) x1 t 3 f (t ) 37 x 2 x1 y 2 y1 1,7 7,3
= =
t2 t1 x2 x1 5 3 25 37 y = 4,29x 0,38 y = 4,29 · 10 0,38 = 42,52
f (t ) = 6t + 55 t Una extrapolació permet una predicció bastant fiable.

g (t) serà el polinomi que passa per (5, 25) i t No, ja que la capacitat de la molla per estirar-se no ha de ser
necessàriament constant.
t t1 g (t ) x1 t 5 f (t ) 25
= = — Baixa el preu del taxi?
t2 t1 x2 x1 8 5 23 25
g (t ) = 16t + 105 t f (x) 2,05 0,98x
g (x) 1,84 1,05x
37 = 9a + 3b + c
b) 25 = 25a + 5b + c x (t ) = 2t 2 + 10t + 25 0,98x + y = 2,05 x =3
23 = 64a + 8b + c 1,05x + y = 1,84 y = 4,99
c) x (6) 13 m Coincideixen al cap deFigura
3 km.87
t (0) Figura
6,8 s 81 Y
6
t (10) 6,2 s
5
A
9. Y 4
3 f (x)
3
2
2 g (x)
1
1
–4 –3 –2 –1 O 1 2 4 5 6 7 X
–1
O 1 2 3 4 5 X
–2

–3
t Resposta oberta a manera de reflexió personal.

150
BLOC 3. ANÀLISI

11# Successions i límits

En context (pàg. 263) 20t 2


a) — Sí que l’atraparà. Es tracta d’una progressió geomètrica. AO: m = lim t + 3 = 20t = 20
t ± t t +3
a a · r n+1
S = 20t 2 20t 2 20t 2 60t
1 r n = lim 20t = lim = 60
t ± t +3 t ± t +3
— L’experiència també ens diu que l’atraparà. No es va
tenir en compte que la sèrie que plantejava Zenó no L’asímptota obliqua és y 20x 60.
tendeix a infinit, de manera que, arribat a un cert punt,
20 · 12
la superarà. b) f (t ) = =5
1+ 3
b) Aquesta activitat pretén que l’alumne reflexioni sobre el
concepte de límit i el significat que té, en el context de mc 2
l’univers. 6. a) E (v ) =
v2
1
c2
Llenguatge matemàtic (pàg. 265) La funció està definida per a v < c .
Es tracta d’un nombre negatiu de magnitud superior a la de lim E (v ) = +
qualsevol nombre real. v ±c

Per tant, l’energia divergeix quan la velocitat s’apropa a


c.
Problemes resolts (pàgs. 277 i 278)
b) Quan la massa d’un objecte és més gran que 0, fa falta
1. a) an = 40 + 3 (n 1) una energia infinita perquè aquest objecte assoleixi la velo-
2n 1 citat de la llum.
an =
10 x +2
7. f (x ) =
b) an = 40 + 3 (12 1) = 73 x3 + ax 2 + 16x

212 1 Si f és discontínua en x 4, aleshores:


an = = 204,8
10 43 42 · a 16 · 4 0 a 8
x +2
2. a) C = Co (1 + i · t ) f (x ) =
x3 8x 2 + 16x
b) C = Co (1 + i )t
x1 = 0
c) Cs = 2 000 (1 + 0,02 · 10) = 2 400 AV: Q (x) 0 x3 8x 2 + 16x = 0 x =
x2 = 4
Cc = 2 000 (1 + 0,02)10 = 2 437,98 x +2
AH: lim =0
x + x3 8x 2 + 16x
3. a) f ( 0 ) = 2 · e 0 = 2
x +2
b) lim 2e t = 2e + = + lim =0
t +
x x3 8x 2 + 16x

4. a) f ( 0 ) = 25 + 50e 0 = 75 La funció presenta dues asímptotes verticals en x 0ix 4, i


tendeix a 0 en ±∞.
b) lim ( 25 + 50e t
) = 25 + 50e = 25
t +
3x 2 + 4x k 3x 2 + 4x k
20t 2 8. f ( x ) = =
5. f (t ) = 2x + 4 2 ( x + 2)
t +3
Veiem que el denominador s’anul·la quan x 2. Si volem
a) AV: Q (x) 0 t 3 0 t 3 que hi hagi una discontinuïtat no evitable en aquest punt, ne-
Considerem únicament valors positius del temps, de ma- cessitem que el numerador no s’anul·li també en x 2.
nera que no hi ha asímptotes verticals. 3 · ( 2)2 4 · ( 2) k 0 k 4
20t 2 Si k 4 no tindrem la discontinuïtat no evitable de salt infinit;
AH: No existeixen, ja que lim =+
t + t +3 per tant, la k que busquem és qualsevol excepte 4; k ≠ 4.

151
BLOC 3. ANÀLISI > UNITAT 11. SUCCESSIONS I LÍMITS

Exercicis i problemes (pàgs. 279 a 282)


16. a) lim
4n 8
= lim
4 8
=
4
n + 5n n + 5 5n 5

1000n 1000
1 SUCCESSIONS Pàg. 279 b) lim
n + n2 2n
= lim
n + n 2
=0

9. a) an = 1 + 3 (n 1) = 3n 4 c) lim ( 2 n) =
n +
n+1
b) an = n · ( 1)
5 + 2n n+6
d) lim =
3 7 13 n + 2n n+2
10. a) a1 = , a2 = , a3 = , a4 = , a5 = ,
2 4 8 16 5 + 2n n+6
27 53 107 = lim lim =1 1= 0
a6 = , a7 = , a8 = , n + 2n n + n+2
32 64 128
213 427 4n
a9 = , a10 = 1
256 512 17. a) lim 1 + = e4
n + n
b) a1 = 1, a2 = 1, a3 = 1, a4 = 1, a5 = 1, a6 = 1
2n 3 8
a7 = 1, a8 = 1, a9 = 1, a10 = 1 b) lim =1
n + 2n 3
1
11. a) Aritmètica, d 5. 1
n
1
4n 4 1

b) Geomètrica, r 3. c) lim 1 + = lim 1+ =e4


n + 4n n + 4n
c) Geomètrica, r 1; aritmètica, d 0.
2n 3 8
d) Cap de les dues. d) lim =0
n + 2n 4
e) Geomètrica, r 3.
f) Cap de les dues. 18. a) 21500 = a1 + d 23 000 = a1 + 4d
an = 21000 + 500n
1 + 2n 1
12. lim = lim +2 =2 b) a1 = 21000
n + n n + n
1+2n c) a10 = 21000 + 9 500 = 25 500
a) lim e n = e2 n
n + d) an = 21000 (1 + 0,023 )
1 + 2n a5 = 22 999,68
b) lim = 2
n + n e) an = 21000 · 1,023n 1

f) A partir del sisè any, ja que aleshores per a la primera ofer-


13. a) lim ( an + bn + cn ) = 2 ta a5 = 23 500 i per a la segona, a5 = 23 528,67
n +

b) lim ( an · bn ) = 24
n + 19. Com que és una progressió geomètrica:
bn 2
c) lim = an = a1 · r n 1
n + an 3
d) lim (bn an ) = 10 Considerant les condicions de l’enunciat:
n +
an 12 = a1 · r 3 12
e) lim (bn ) = 0,000 244 12r = 18
n +
a1 · r 2 a1 · r 4 = 18 r
cn
an 1
f) lim =1 r1 =
n + bn 2r 2 + 3r 2=0 2
r2 = 2
14. a) an = n
a10 = 10 a25 = 25 a1,1 = 96
12
a1 = 3
b) an = 3 + (n 1) · 2,5 r3 a1,2 =
2
a10 = 25,5 a25 = 63
3
an = 96 0,5n 1 o an = ( 2)n 1

15. a) a1 = 1; a2 = 2; a3 = 4; a4 = 8; a5 = 16; a6 = 32; 2


a7 = 64; a8 = 128; a9 = 256; a10 = 512.
20. a) És monòtona decreixent i el seu valor tendirà a 0 a mesura
b) an = 2n 1 que augmenti n; per tant, és convergent.
c) a64 = 263 = 9, 2 · 1018 an r a1 a1 a1
b) lim Sn = lim = =
d) Aquesta quantitat de grans correspon a unes 15 · to- 1012 n + n + r 1 r 1 1 r
nes de blat, les quals corresponen a més de 21 000 anys
on hem utilitzat que lim an = 0.
de la producció mundial de blat. n +

152
BLOC 3. ANÀLISI > UNITAT 11. SUCCESSIONS I LÍMITS

No existeix el límit.
2 SUCCESSIONS EN LA
MATEMÀTICA FINANCERA Pàgs. 279 i 280 b) f ( x ) =
2x
quan x 1.
+ 2x + 1 x2
21. C = 7 000 · (1 + 3 · 0,028 5 ) = 7 598,5 2x 2 1
lim = =
x 1 x 2 + 2x + 1 4 2
0,0275
12·4 2x 2 1
22. C = 15 000 · 1 + = 16 742,06 lim = =
x 1+ x 2 + 2x + 1 4 2
12
El límit és 1/2.
23. Interès simple:
2C 0 = C 0 · (1 + 0,05n ) n = 20 30. La variable a ha de complir que:
Interès compost: ax 2 8 4a 8 3
1 = lim = = 2a 4 a=
n x 2 x 2 2
2C 0 = C 0 · (1 + 0,05 ) n = 14,21

4
31. a) No existeix. c) 4 e) 0
0,08
24. a) 1 + = 1,082 4 TAE = 8,24 % b) 1 d) 1 f) 2
4
12
0,078
b) 1 + = 1,080 8 TAE = 8,08 %
12 4 CÀLCUL DE LÍMITS Pàgs. 280 i 281
Per tant, el primer cas genera més interessos. 0
n 32. a) lim x · 3x 1 = =0
c) 15 000 = 12 000 (1 + 0,808 ) n 35 mesos x 0 3
n ln ( x + 1) 0
25. 9 000 = 7 000 · (1 + 0,07 ) n = 3,7 b) lim = =0
x 0 2x + 2 2
6
26. 3 600 = 3 000 · (1 + i ) i = 3,08 % x 1 1
c) lim =
x 2 2 x +x 2

3 LÍMIT DE FUNCIONS Pàg. 280 d) lim


x3 + 5x 2 + 8x + 4
= lim
(x + 2) (x + 2) (x + 1)
=
1
x 2 x3 + x2 8x 12 x 2 (x + 2) (x + 2) (x 3) 5
27. a) lim 3 x 1
= 31 = 3
x 2 x 5 0
e) lim (x2 16 ) =9 2 =1
x 5
b) lim x 2 = 1 =1
x 3 4
x 2 x 2
c) lim sin x = 1 f) lim = ( 1) = 1
x 90º x 2 2 x
d) lim ln x = 1
x e
h (x ) 1 1
x
33. a) lim = lim =
28. a) lim =0
x 2 f (x ) x 2 (x2 9) · ( x 2 1) 15
x ± 3 x3
b) lim (f ( x ) + g ( x ) ) =
6 x x 4
b) lim =1
x + 2 x = lim ( x 2 1) + lim 2x 8 = 15 + 0 = 15
x 4 x 4
6 x
lim =1
x 2 x c) lim (f ( x ) g ( x )) =
x 2

6x 4 + 1 6 = lim ( x 2 1) + lim 2x 8 = 15 + 4
c) lim = =3 x 2 x 2
x ± 2x 4 +1 2
No existeix.
6x 4 + 1
d) lim = lim ( 3x ) =
x + 2x 3 + 1 x + d) lim (f ( x ) · j ( x ) ) =
x 1
6x 4 + 1 2

x
lim
2x 3 +1
= lim
x
( 3x ) = +
= lim
(x2 (
1) ( 2 + x ) 4 ) =0
x 1 x
x x2 1 35
29. a) f ( x ) = quan x 3. e) lim (f ( x ) · h ( x ) ) = lim =
x 3 x 6 x 6 x2 9 27
x 3 2
x2 1

lim = = f (x ) (2 + x ) 4
x 3 x 3 0 f) lim [ j ( x ) ] = lim =
x 2 x 2 x
x 3
lim+ = =+
x 3 x 3 0+ = 63 = 216

153
BLOC 3. ANÀLISI > UNITAT 11. SUCCESSIONS I LÍMITS

2
2x 3 + 4x 2 18 2 ( x + 2 ) = lim x 2 + 4x + 4 = 1
34. a) lim = lim =0 c) lim
x + x4 + 3x 6 x + x x ± 2x 2 3 2 x ± 2x 3 2
4x 2 5x 18 4 La funció f (x) té una AH en y 0,5.
b) lim = lim x =+
x + 3x 6 x + 3 5
d) lim 2 =2
3x 2 + 2x 3 x ± 2x + 1
c) lim =
x + 2x 2 2 La funció f (x) té una AH en y 2.

x 3 + 2x x 2x + 3
d) lim = lim =+ 38. a) f ( x ) =
x + 500x 2 x + 500 4x 8
x2 +1 x2 +1 AV: Q (x) 0 x 2
e) lim = lim =1
x + x x + x2 2x + 3 1
AH: lim = y = 0, 5
x ± 4x 8 2
2 x 1 2x 2 + 10
f) lim : = lim =2 x2
x + x x2 + 5 x + x2 x b) f ( x ) =
x 4
g) lim
x5 4x 3
= lim
(x2 4) x 3
= 4 AV: Q(x) 0 x 4 0 x 4
x 2 2x 2 8 x 2 (x2 4) 2
f (x ) x2
2
AO: m = lim = lim =1
x3 + 5x 2 + 8x + 4 ( x + 2 ) ( x + 1) = 1 x ± x x ± x 2 4x
h) lim = lim 2
x 2 x 2 + 4x + 4 x + ( x + 2) 4x
n = lim (f ( x ) mx ) = lim =4
x ± x ± x 4
x2 + a x2 a 2ax 2 + 2ax
35. lim = lim =6 y x 4
x + x a x +a x + x2 a2
2x
2a = 6 a=3 c) f ( x ) =
x 4
AV: Q(x) 0 x 4 0 x 4

5 LÍMITS I ASÍMPTOTES Pàg. 281 AH: lim


2x
=2 y =2
x ± x 4
2
36. a) f ( x ) = 2 x
x 2 d) f ( x ) =
x
Q(x) 0 x 2 0 x 2
AV: Q(x) 0 x 0
x
b) f ( x ) = 2 x
x2 16 AH: lim = 1 y = 1
x ± x
x1 = 4
Q (x ) = 0 x2 16 = 0 x =
x2 = 4 x1 = 2
39. a) AV: Q ( x ) = 0 x2 2=0 x =
x x2 = 2
c) f ( x ) =
( x + 2) ( x 3) x2 1
AH: lim =1 y =1
Q (x ) = 0 ( x + 2) ( x 3) = 0 x ± x2 2
x1 = 2 x1 = 0
x = b) AV: Q ( x ) = 0 x2 2x = 0 x =
x2 = 3 x2 = 2

4 5
d) f ( x ) = AH: lim =0
x3 4x x ± x2 2x
x1 = 0 c) AV: Q ( x ) = 0 2x 3 + 4 = 0 x = 1,259 9
Q (x ) = 0 x3 4x = 0 x = x2 = 2 f (x ) x4 1
x3 = 2 AO: m = lim = lim =
x ± x x ± 2x 4 + 4x 2

4x + 2 4 x 4 + 4x
n = lim (f ( x ) mx ) = lim =0
37. a) lim = =2 x ± x ± 4x 3 + 8
x ± 2x 3 2
y 0,5x
La funció f (x) té una AH en y 2.
4x + 2 d) AV: Q ( x ) = 0 x 1= 0 x =1
b) lim =0
x ± 2x 2 3 f (x ) x2
AO: m = lim = lim =1
La funció f (x) té una AH en y 0. x ± x x ± x 2 x

154
BLOC 3. ANÀLISI > UNITAT 11. SUCCESSIONS I LÍMITS

x lim f (x) = 2
n = lim (f ( x ) mx ) = lim =1 x 2
x ± x ± x 1 lim+ f (x) = 4
x 2
y x 1
Els límits no coincideixen, de manera que la funció no és con-
tínua en x 2, on presenta una discontinuïtat evitable.
40. a) AH: lim (10 + 24 000 · e 3t
) = 10
x +

b) La població Figura
disminuirà
01 a mesura que passi el temps; el 46. a) La funció presenta una discontinuïtat de salt infinit en x 2
nombre d’animals s’anirà aproximant a 10 exemplars. ix 2, i una discontinuïtat evitable en x 0.
lim f (x) no existeix.
41. a) Q ( x ) = 0 cos x = 0 x = 180º · (k + 90º ) x 0
lim+ f (x) = +
x 2
b) Y
3 lim f (x) =
f(x) = tg(x) x 2
2

1
lim + f (x) =
x 2
3 – 0 3 X
–2 2

2

2 –1 2 2 lim f (x) = +
x 2
–2

–3
b) La funció presenta una discontinuïtat de salt infinit en x 0
i una de salt finit en x 2, i és contínua en x 2.
lim f (x) no existeix.
x 0
6 CONTINUÏTAT DE Figura 02
x
lim f (x) no existeix.
2
FUNCIONS Pàgs. 281 i 282
lim f (x) = f (–2).
x 2

42. a) Contínua.
b) Discontinuïtat evitable.
47. Y
3

c) Discontínua de salt infinit. 2


1
d) Discontinuïtat essencial.
–3 –2 –1 0 1 2 3 4 5 6 X
43. a) f (x ) x 2 4x 2 –1

–2
Com que és una funció polinòmica, no presenta cap dis-
continuïtat.
x 1 kx + m si x 3
b) f ( x ) =
x +5 2k
48. f ( x ) = si 0 < x < 3
La funció presenta una discontinuïtat en el punt en el qual x 1
s’anul·la el denominador: kx 2 m + 5x si x > 3
Q (x) 0 x 5 2k
lim (kx + m ) = lim+ m = 2k
x 0 x 0 x 1
44. a) La funció presenta una discontinuïtat de salt finit en el
2k
quilòmetre 300. lim = lim+ (kx 2 m + 5x ) k = 9k m + 15
x 3 x 1 x 3

b) No tindria sentit, ja que no seria lògic tenir un preu infinit. m = 2k 3


2k = 8k + 15 k = m=3
m = 8k + 15 2
2x + 4 si x 2
45. f ( x ) = x si 2<x 2
3x + 2 si x > 2
SÍNTESI Pàg. 282
Per comprovar la continuïtat de la funció, estudiem els límits
laterals en els punts x 2 i x 2. x2
49. a) lim =+
x 1 2x 2
Estudiem x 2:
x2
lim f (x) = 0 lim+ =+
x 2 x 1 2x 2
lim + f (x) = 2
x 2 x2
b) lim =+
La funció no és contínua en x 2, on presenta una disconti- x 2x 2
nuïtat evitable. x2
c) lim =+
Ara estudiem x 2: x 2x 2

155
BLOC 3. ANÀLISI > UNITAT 11. SUCCESSIONS I LÍMITS

Figura 03

x2 d) Y
d) lim =0 4
x 0 2x 2 3

x2 2
lim+ =0 f(x)
x 0 2x 2 1
0
–4 –3 –2 –1 1 2 3 4 5 6 X
x2 –1
e) lim =2 Figura 04
x 2 2x 2 –2

–3

x2 2
f) lim =
x 2 2x 2 3
53. Y
10

x 2 + 2x a 8 a 0 9
50. a) lim = =
x 2 x2 bx + 2 6 2b 0 8

8 a=0 a=8 6
5
6 2b = 0 b =3
4

3
b) lim
x 2 + 2x 8
= lim
( x + 4) ( x 2)
= lim
x+4
=6 2
x 2 x2 3x + 2 x 2 ( x 1) ( x 2 ) x 2 x 1
1

51. a) La funció presenta en x 1 i en x 4 una discontinuïtat de –3 –2 –1 0 1 2 3 4 5 6 7 8 9 10 X


–1
salt finit, i en x 2 una discontinuïtat evitable.

b) lim 2x = 2 54. a) Sí, per exemple la funció tangent.


x 1

b) No, una funció pot tenir com a màxim 2 asímptotes horit-


c) lim+ 4 = 4
x 1 zontals, una en menys infinit i una altra en més infinit.

d) lim f ( x ) = lim 4 = lim+ ( 2x + 8 ) = 4 c) No, en ±∞ mai no tendiran a un valor finit.


x 2 x 2 x 2
d) No, només en pot tenir una.
e) lim ( 2x + 8 ) = 0
x 4

2
Avaluació (pàg. 284)
f) lim ( x 5) = 1
x+ 4
1. La suma d’angles interns d’un hexàgon és de 720º. La suma
g) lim ( 2x + 8 ) no existeix. dels 6 primers termes equival a:
x 4
(a1 + a6 ) n (40 + a6 ) 6
Sn = 720 = a6 = 200
h) lim f (x) no existeix. 2 2
x 0
a6 a1 200 40
2 d = = = 32
i) lim ( x 5) = 1 5 5
x 6
a1 = 40; a2 = 72; a3 = 104; a4 = 136; a5 = 168; a2 = 200
2x + 4 si x 0
1 18 6 1
52. f ( x ) = si 0 < x < 3 2. a) r = = =
x 2 54 18 3
n
ex 3 si x > 3 1
an = 54 ·
3
a) lim ( 2x + 4 ) = 4 1
8
1
10
x 0 b) a8 = 54 = 0,024 7 a10 = 54 = 0, 0027
1 3 3
lim+ = 0,5
x 0 x 2
1 15
lim =1 3. a) d = =5
x x
3 2 3
lim e x 3 = 1 a1 = 42 5 = 37
x 3+
b) an = 37 + 5n
b) La funció té una discontinuïtat de salt finit en x 0, una
I 9 000
discontinuïtat de salt infinit en x 2 i és contínua en x 3. 4. I = C 0 · i · t C0 = = = 135 000
i ·t 5
0,04 ·
c) No. 3

156
BLOC 3. ANÀLISI > UNITAT 11. SUCCESSIONS I LÍMITS

5. C = C 0 (1 + i )t 9. a) lim f ( x ) = 2
x

10
C1 = 5 000 (1 + 0,02 ) = 6 094,97 b) lim f ( x ) = 2
x +
C 6 094,97
i2 = 15 1= 15 1 = 0,028 5 = 2,85 %
C0 4 000 c) lim f ( x ) =
x 4

d) lim + f ( x ) = 0
6 4n 2 4 x 4
6. a) lim = = 2
n + 2n 2 2
e) lim f ( x ) no existeix.
x 4
4n 2 + 3n 2 2
b) lim = lim =0
n + 2n 3 4n n + n f) lim f ( x ) = 2
x 2

2n 3 4n n2
c) lim = lim =+ g) lim + f ( x ) = 4
n + 4n n + 2 x 2

3x 6 h) lim f ( x ) no existeix.
7. a) lim = = x 2
x 2 x2 4 0
i) lim f ( x ) =
x 3
3x 6
b) lim+ = =+
x 2 x2 4 0+ j) lim+ f ( x ) =
x 3
x +2 x 7 x2 5x 14
c) lim · = lim =1
x x +1 x 3 x x2 2x 3 k) lim f ( x ) =
x 3

8x 4 + 2 l) lim f ( x ) = 2
d) lim = lim ( x2) = x 0
x 2x 2 + 4 x

En x 4 hi ha una discontinuïtat de salt infinit; en x    2,


8. a) AV: Q ( x ) = 0 x =± 2 una discontinuïtat de salt finit; en x 0, una discontinuïtat
f (x ) 2x 3 3x evitable, i en x 3, una discontinuïtat de salt infinit.
AO: m = lim = lim = 0,4
x + x x + 5x 3 10x
17x 1
n = lim (f ( x ) mx ) = lim =0 si x < 0
x ± x ± 5x 2 10 2x
10. a) f ( x ) =
x +1 si 0 x <2
y 0,4x
5 x si x 2
b) AV: Q (x) 0 x 0
f (x ) 4x 2 5 1
AO: m = lim = lim =2 lim =
x + x x + 2x 2 x 02x
lim+ x + 1 = 1
5 x 0
n = lim (f ( x ) mx ) = lim =0
x ± x ± 2x La funció presenta una discontinuïtat de salt infinit en
y 2x x 0.

1 2x 2 + 4x 1 lim x + 1 = 3
c) f ( x ) = 2x = x 2
2 x x +2 lim+ 5 x =3
x 2
AV: Q (x) 0 x 2 lim 5 x =3
x 2
f (x ) 2x 2 + 4x 1
AO: m = lim = lim =2 La funció és contínua en x 2.
x + x x + x 2 + 2x
1 x2 1
n = lim (f ( x ) mx ) = lim =0 si x < 1
x ± x ± x +2 b) f ( x ) = x 1
2 si x 1
y 2x

4x 2 5 x2 1 (x + 1)(x 1)
d) f ( x ) = lim = lim = lim (x + 1) = 2
x2 + 2 x 1 x 1 x 1 x 1 x 1

AV: Q ( x ) 0
lim+ 2 = 2
x 1
4x 2 5
AH: lim =4 y =4
x ± x2 + 2 La funció és contínua.

157
BLOC 3. ANÀLISI > UNITAT 11. SUCCESSIONS I LÍMITS

ZONA (pàg. 285)


P(t ) =
5t + 8
és l’expressió que determina l’evolució del preu
— Velocitat de la grip t +1
del nostre component informàtic.
2
P(4) = = 1,782
1 + 3e 0,8 4 L’evolució a llarg termini dels preus dels productes no respon a
models senzills com el que hem vist; per exemple, el preu del
2 petroli varia substancialment quan hi ha inestabilitat política,
P( ) = =2
1 + 3e 0,8 guerres, crisi econòmica, sobreproducció, una forta competèn-
cia amb altres fonts energètiques, etc. Per a predir-ne l’evolució
En què els resultats estan expressats en milers de persones.
del preu, s’han d’elaborar models més complexos.
— L’evolució del preu d’un article

a 1+ 8
P(1) = = 6,5
1+b a=5
a 2+8 b =1
P(2) = =6
2+b

158
BLOC 3. ANÀLISI

12# Derivades

En context (pàg. 287) La funció està definida per a tots els valors de x; per tant,
D f (x ) R
a) En la gràfica de l’acceleració en funció del temps, veiem
que el valor de la màxima acceleració s’esdevé en l’instant Recorregut: Rf ( x ) = R .
inicial; és a dir, quan t 0. La funció no presenta cap tipus de simetria.
b) La gràfica de la distància en funció del temps té una forma Tall amb l’eix horitzontal: f (0) 3.
aproximadament lineal; podem dir, amb petites variacions,
que, com més temps passa, més lluny es troba del punt de La funció talla l’eix d’abscisses en el punt (0, 3).
partida. Respecte a la gràfica de la velocitat, Bolt comença Calculem la derivada de f (x ):
en repòs (v 0) i la seva velocitat augmenta durant els
primers segons; al final de la cursa, la seva velocitat expe- 1 si x 0
rimenta algunes variacions, i disminueix lleugerament.
Quant a l’acceleració, a l’inici de la cursa és molt alta, però, f ( x ) = 2x si 0 < x 2
a mesura que passa el temps i va adquirint velocitat, l’ac-
5
celeració disminueix; hi ha intervals del final de la cursa en 2
si x > 2
què l’acceleració és negativa i, per tant, desaccelera. ( x 1)
c) Resposta suggerida: Es tracta de prediccions; és a dir, Observem que la derivada mai no val 0. Per a estudiar el crei-
càlculs basats en un seguit de suposicions, de manera que xement o el decreixement de la funció, hem d’avaluar el valor
no s’ha comprovat. de la seva derivada en un punt de cada interval.
d) Resposta suggerida: Per a optimitzar formes geomètri- x 1 x 1 x 2
ques, en situacions de física aplicada, economia, enginye-
ria, tractament d’imatges, meteorologia, etc. f (x )
f (x )

Fixa-t’hi (pàg. 288) Asímptotes horitzontals: lim f ( x ) = 0; lim f ( x ) =


x + x
La funció f (x ) és una funció parabòlica que, en x 1, decreix.
En canvi, la funció g (x ) és una funció lineal que creix en tot el La funció no presenta asímptotes verticals ni obliqües.
seu domini.
Y

2x – 1 si x ≤ 1 5

Amplia (pàg. 292) f(x)= x 2 si 1 < x ≤ 3


4
3
x 3 + 2x – 1 si x > 2
2
1
f (x ) = x =x 2 1

1 –6 –5 –4 –3 –2 –1 0 1 2 3 4 5 6 X
1 1 –1
f (x ) = x 2 = –2
2 2 x
–3
–4
–5
Amplia (pàg. 294) –6

1 –7
h (x ) = f (x ) · g (x ) 1 –8
h (x ) = f (x ) · g (x ) 1
h ( xh) (=x )f (=xf) ·( xg)(·xg) ( x ) 1 + ( g ( x )) · f
2
(x ) · g (x ) 2 =
1
h (x ) = f (x ) · g (x ) 1 + ( g ( x )) · f (x ) · g (x ) 2 =
fh ((xx)) ·=gf( x( x) ) ·gg ((xx)) · f +( x( ) g ( x )) · f (x ) · g (x ) = 2. La funció és f ( r ) = r 2 . L’equació de la recta tangent és de-
= f ( x ) · g ( x ) g2 ( x ) · f ( x )
= f ( x ) · g ( xg) ( x )g2 ( x ) · f ( x ) terminada per y f (a) f’(a) (r a). Calculem la derivada de
= g ( x )2 f (x ):
g (x )
f (x ) = 2 r; f (2) = 4 ; f (2) = 4
Problemes resolts (pàgs. 298 i 299) Per tant:

y 4 =4 (r 2) y =4 ( r 1)
x 3 si x 0
x2 + 1 si 0 < x 2 3. x 2t 2 t 2
1. f ( x ) =
5 L’equació de la recta tangent és determinada per:
si x > 2
x 1
x f (a) f (a)(t a)

159
BLOC 3. ANÀLISI > UNITAT 12. DERIVADES

Calculem la derivada de la funció: lim f ( x ) = 2a


x 2
x (t ) 4t 1 2a = 2 a =1
lim+ f ( x ) = 2
x 2
x (2) 8
x (2) 7 Si a 1, la funció és contínua en x 2. Per tant, ara estudiem
la derivabilitat quan x 2.
L’equació de la recta tangent és:
x 8 7 (t 2) x 7t 6 2 x 0
L’equació normal és determinada per:
f (x ) = 1 0<x 2
1 1
x f (a ) = (t a) x >2
f (a ) 2 x +2

Si substituïm pels valors obtinguts anteriorment: Vegem com es comporta la derivada entorn de x 2
1 1 1
x 8= (t 2 ) x = (t 58 ) f (2 ) = xlim2 1 = 1 f ( 2+ ) = lim+ x = 0,25
7 7 x 2 2 x +2

Com que els valors laterals no coincideixen, la funció no és


2x 1 si x 1 derivable.
4. f (x ) = x2 si 1 < x 3
Ara, estudiem la continuïtat en el punt x 0.
x 3 + 2x 1 si x > 3
lim f ( x ) = 2x + 1 = 1
x 0
Com que es tracta de funcions polinòmiques, podem reduir
l’estudi als punts x 1 i x 3. Comprovem si la funció f (x ) és lim+ f ( x ) = ax = 0
x 0
contínua en x 1 i en x 3.
Com que els límits no coincideixen, la funció no és contínua
lim f ( x ) = lim 2x 1=1 en x 0 i, en conseqüència, tampoc no és derivable.
x 1 x 1
lim+ f ( x ) = lim+ x 2 = 1
x 1 x 1
f (1) = 1 Exercicis i problemes (pàgs. 300 a 306)

Com que els valors coincideixen, la funció és contínua en x 1.

lim f ( x ) = lim x 2 = 9 1 TAXA DE VARIACIÓ Pàg. 300


x 3 x 3
lim+ f ( x ) = lim+ x 3 + 2x 1 = 32 6. a) f (x ) x3 4x 6 en [0,2]
x 3 x 3
f ( 3) = 9 f (0) 6 f (2) 6
6 6
Atès que els valors la funció no coincideixen, no és contínua TVM [ 0,2 ]f ( x ) = =0
2 0
en x 3, com la funció no és contínua, tampoc no és deriva-
ble en x 3. b) f ( x ) = x 3 en [3,7]
Derivem la funció:
f (3) 0 f (7) 2
2 si x 1 2 0
f ( x ) = 2x si 1 < x 3 TVM [ 3,7 ]f ( x ) = = 0,5
7 3
3x 2 + 2 si x > 3
c) f (x ) (x 1)2 en [ 3,0]
Vegem com es comporta la derivada a l’entorn de x 1: f ( 3) 16 f (0) 1

f (1 ) = xlim1 2=2 f ( ) = xlim1


1+ 2x = 2 1 16
+ TVM [ 3.0 ]f (x ) = = 5
0+3
Coincideixen; per tant, la funció és derivable en x 1. x 1
d) f ( x ) = en [ 1,2]
x +2
2x + 1 x 0 f ( 1) 2 f (2) 0,25
5. f ( x ) = ax 0<x 2
0,25 + 2
x +2 x >2 TVM [ 1.2 ]f (x ) = = 0,75
2+1
Podem reduir l’estudi de la funció als punts x 0ix 2.
2 0
7. a) TVM [ 0,2 ]f ( x ) = =1
Perquè la funció sigui contínua en x 2 s’ha de complir la 2 0
condició que:
9 25
TVM [ 0,2 ] g ( x ) = = 8
lim f ( x ) = lim+ f ( x ) = f ( 2 ) 2 0
x 2 x 2

160
BLOC 3. ANÀLISI > UNITAT 12. DERIVADES

2 2 c) f (x ) x2 1 en x 0; f (0) 1
b) TVM[2,4]f ( x ) = =0
4 2 x2 1+1
TVI 0f ( x ) = lim = lim x = 0
1 9 x 0 x 0 x 0
TVM[2,4]g ( x ) = = 4
4 2 d) f (x ) x3 2x 4 en x 1; f (1) 1
0 2 1
c) TVM[4,10]f ( x ) = = x3 + 2x 4+1
10 4 3 TVI1f ( x ) = lim =
x x 1 1
25 1
TVM[4,10]g ( x ) = =4 (x 1) ( x 2 + x + 3 )
10 4 = lim =5
x 1 x 1
5 0
8. a) TVM[0,5]f ( x ) = = 0,25 km/min 11. a) f (x ) x f (1) 1 f (1 h) 1 h
20 0
1+h 1
5 0 TVM[1,1 + h]f ( x ) = =1
TVM[0,5]g ( x ) = = 0,25 km/min 1+h 1
20 0
10 5 b) f (x ) x2 f (1) 1 f (1 h) h2 2h 1
b) TVM[20,45]f ( x ) = = 0,2 km/min
45 20 h2 + 2h + 1 1
TVM[1,1 + h]f ( x ) = =h+2
10 5 1+h 1
TVM[20,40]g ( x ) = = 0,25 km/min
40 20
c) f ( x ) = x f (1) 1 f (1 + h ) = 1+h
10 0
c) TVM[0,45]f ( x ) = = 0,222 km/min 1+h 1 1+h 1
45 0 TVM[1,1 + h]f ( x ) = =
1+h 1 h
10 0
TVM[0,40]g ( x ) = = 0,25 km/min
40 0 1 1
d) f ( x ) = f (1) 1 f (1 + h ) =
x 1+h
d) Corresponen a la velocitat mitjana.
1 h
1
9. a) Calculem: 1
TVM[1,1 + h]f ( x ) = 1 + h = 1+h =
1+h 1 h 1+h
f (0) 3 f (2) 15,778
15,778 3 12. La funció que representa l’àrea del cercle és: A ( r ) = r 2
TVM[0,2]f ( x ) = = 6,39
2 0
16 4
a) TVM[2,4]A ( r ) = = 18,85
b) Calculem: 4 2
f (3) 41,17 f (6) 807,85 100 0
b) TVM[0,10]A ( r ) = = 31,41
10 0
807,85 41,17
TVM[3,6]f ( x ) = = 255,56
6 3 r2 4
c) TVI2A ( r ) = lim =
r 2 r 2
c) Podem escriure la fórmula de la taxa de variació instantà-
= lim
( r + 2 ) ( r 2 ) = 12,56
nia de la manera següent:
r 2 r 2
f (a + h ) f (a )
TVIaf ( x ) = lim r2 100
h 0 h d) TVI10 A ( r ) = lim =
r 10 r 10
Per tant: ( r + 10 ) ( r 10 ) = 62,8
= lim
1+ 2e 6 +h (1 + 2e 6 ) 2e 6 ( eh 1) r 10 r 10
TVI6f ( x ) = lim = lim =
h 0 h h 0 h r2 10 000
e) TVI100 A ( r ) = lim =
= 2e 6 lim
eh 1
= 2e 6 lim
(1 + h ) 1 = 2e 6 = 806,86 r 100 r 100
h 0 h h 0 h ( r + 100 ) ( r 100 )
= lim = 628,3
r 2 r 100
10. a) f (x ) 2x 3 en x 2; f (2) 7
2x + 3 7 2 (x 2)
TVI2f ( x ) = lim
x 2 x 2
= lim
x 2 x 2
=2 2 DERIVADA D’UNA FUNCIÓ Pàg. 301

1 13. a) f (x ) x 6
b) f ( x ) = en x 2; f (2) 1/2
x f ( x ) f (a )
f ( a ) = lim
1 1 ( x 2) x x a a

1 x +6 5
TVI2f ( x ) = lim x 2 = lim 2x = f (1) = lim = 1
x 2 x 2 x 2 x 2 4 x 1 x 1

161
BLOC 3. ANÀLISI > UNITAT 12. DERIVADES

x y f (a) f (a)(x a)
b) f ( x ) = 1
2 Calculem la derivada de la funció en el punt x 1
x 1 1
1+ ( x 1) 4 4
f (1) = lim 2 2 = lim 2 = 0,5 (r + h )
3
r3
x 1 x 1 x 1 x 1 V (1) = lim 3 3 =
h 0 h
c) f (x ) x2 2x 1 4 4
( r 3 + 3r 2h + 3rh 2 + h 3 ) r3
x 2 + 2x + 1 2 = lim 3 3 =
f (1) = lim = h 0 h
x 1 x 1
= 4 (1)2
x 2 + 2x 1 V (1) = 12,57
= lim = lim ( x + 1) = 0
x 1 x 1 x 1

Per tant, l’equació de la recta tangent és:


2
d) f ( x ) = 4
x y = 12,57 ( r 1) y = 12,57r 8,38
3
2 2 2x 2 ( x 1)
2
x x x
f (1) = lim
x 1 x
= lim = lim = 17. a) f (x ) 3x 2 4x 5
1 x 1 x 1 x 1 x 1
2 3x 2 4x + 5 3a 2 + 4a 5
= lim = 2 f ( a ) = lim =
x 1 x x a x a
3x 2 3a 2 4x + 4a
= lim + lim =
14. f (x ) x 2 3x 5 en x 2; f (2) 3 x a x a x a x a
3 (x + a) (x a) 4 (x a)
L’equació de la recta tangent és determinada per: = lim + lim =
x a x a x a x a
y f (a) f (a)(x a) = 6a 4
Calculem la derivada de la funció en el punt x 2
b) f (x ) 3x 80
x2 3x + 5 3 3x + 80 3a 80
f ( 2 ) = lim = f ( a ) = lim =
x 2 x 2 x a x a
x2 3x + 2 3 (x a)
= lim = = lim =3
x 2 x 2 x a x a
= lim
(x 2 ) ( x 1)
=1
x 2 x 2 18. De la imatge, deduïm que:
L’equació de la recta tangent és: x +2 x 2
f (x ) =
y 3 (x 2) y x 1 x 2 x >2

15. f (x ) 3x 2 4x 6 N’estudiem la continuïtat:

3x 2 4x + 6 10 lim ( x + 2 ) = 0 = xlim2+ ( x 2 ) = 0 = f ( 2 )
a) f ( 2 ) = lim = x 2
x 2 x 2
Com que els valors coincideixen, la funció és contínua. N’es-
3x 2 4x 4 tudiem la derivabilitat:
= lim =
x 2 x 2
( 3x + 2 ) ( x 2) 1 x 2
= lim =8 f (x ) =
x 2 x 2 1 x >2

3x 2 4x + 6 13 lim ( 1) = 1 lim (1) = 1


b) f ( 1) = xlim1 = x 2 x 2+
x +1
Com que els valors no coincideixen, la funció no és derivable
3x 2 4x 7 en x 2.
= lim =
x 1 x +1
3x 2 4x 7 x a si x < 0
= lim =
x 1 x +1 19. f ( x ) = x 2 si 0 x 2
= lim
( x + 1) ( 3x 7)
= 10 2x si x > 2
x 1 x +1
Atès que la funció que s’ha d’estudiar està composta per fun-
cions polinòmiques, l’estudi de la funció es redueix als punts
4 4 x 0 i x 2.
16. V ( r ) = r 3 ; V (1) =
3 3 Estudiem el punt x 0 i comprovem si la funció és contínua
L’equació de la recta tangent és determinada per: en aquest punt:

162
BLOC 3. ANÀLISI > UNITAT 12. DERIVADES

lim f ( x ) = 1 3 2
5 3 3
x 0 e) f ( x ) = x3 = x 5 f (x ) = x 5 =
lim+ f ( x ) = 0 5 5 x25
x 0
1 3
f (0) = 0 1 1 1
f) f ( x ) = =x 2 f (x ) = x 2 =
x 2 2 x3
Com que no coincideixen, la funció no és contínua en x 0 i,
per tant, tampoc no és derivable en aquest punt.
22. a) f (x ) 3x 5 g (x ) cos x
Estudiem la continuïtat de la funció f (x ) en el punt x 2:
f (x ) 15x 4 g (x ) sin x
lim f ( x ) = 4
x 2 h (x ) f (x ) g (x ) 15x 4 sin x
lim f ( x ) = 4
x 2+ j (x ) f (x ) g (x ) 15x 4 sin x
f (2) = 4
b) f ( x ) = x g (x ) lnx
Els valors coincideixen; per tant, la funció f (x ) és contínua en
1 1
x 2. f (x ) = g (x ) =
2 x x
Derivem la funció f (x ):
1 1
h (x ) = f (x ) + g (x ) = +
1 si x < 0 2 x x
f ( x ) = 2x si 0 x 2
1 1
2 si x > 2 j (x ) = f (x ) g (x ) =
2 x x

Per saber si és derivable en x 2, observem com es comporta 1


c) f (x ) 2x 3 g (x ) =
al voltant d’aquest punt: x
f (2 ) 4 f (2 ) 2 1
f (x ) 6x 2 g (x ) =
No coincideixen; per tant, la funció f (x ) no és derivable en x2
x 2. 1
h ( x ) = f ( x ) + g ( x ) = 6x 2
x2
20. a) f (x ) 2x 3 1
1
3 j (x ) = f (x ) g ( x ) = 6x 2 +
2 (x + h) + 1 2x 3 1 x2
f ( x ) = lim =
h 0 h
d) f (x ) sin x g (x ) 5ex
6x 2h + 6xh 2 + h 3
= lim = 6x 2 f (x ) cos x g (x ) 5ex
h 0 h
h (x ) f (x ) g (x ) cos x 5e x
b) f ( x ) = x +1
j (x ) f (x ) g (x ) cos x 5e x
x +h +1 x +1
f ( x ) = lim = e) f (x ) log3x g (x ) 3x
h 0 h
( x +h +1 x +1 )( x +h +1 + x +1 ) 1 1
= lim = f (x ) = · g ( x ) = ln 3 · 3x
x ln 3
h 0 h ( x +h +1 + x +1 )
1 1
= lim
( x + h + 1 x 1) = h (x ) = f (x ) + g (x ) = · + ln 3 · 3x
x ln 3
h 0 h ( x +h +1 + x +1 ) 1 1
h h (x ) = f (x ) g (x ) = · ln 3 · 3x
= lim = x ln 3
h 0
( x +h +1 + x +1 h )
1 23. a) f (x ) x 6 · sin x f (x ) 6x 5 · sin x cos x · x 6
=
2 x +1 x2
b) f ( x ) = x 2 x f ( x ) = 2x x +
2 x
c) f ( x ) = x 2 ln x f ( x ) = 2x ln x + x
3 CÀLCUL DE DERIVADES Pàgs. 301 a 304
d) f ( x ) = ( x 3 2x ) e x f (x ) =
21. a) f (x ) x 5 f (x ) 5x 4
= ( 3x 2 2) e x + ( x 3 2x ) e x =
b) f (x ) x 4 f (x ) 4x 5

1 3 = e x ( x 3 + 3x 2 2x 2)
c) f ( x ) = =x 3 f ( x ) = 3x 4 =
x3 x4 cos x
1 3
e) f ( x ) = ln x · cos x f (x ) = sin x · ln x
4 1 1 x
d) f ( x ) = x =x 4 f (x ) = x 4 =
4 44 x 3 f) f ( x ) = sin x · cos x f ( x ) = cos2 x sin2 x

163
BLOC 3. ANÀLISI > UNITAT 12. DERIVADES

sin x f (x ) 3(4x 4)(2x 2 4x 3)2


g) f ( x ) = x · sin x f (x ) = + cos x x
2 x b) f (x ) ln(cos x )
h) f ( x ) = 3x · x f ( x ) = ln 3 · 3x · x + 3x = f (x ) és composició de g (x ) ln x i h (x ) cos x; per tant:
= 3x (ln 3 · x + 1) f (x ) g (h (x )) g (cos x ) ln(cos x )

sin x cos x · x sin x 1


24. a) f ( x ) = f (x ) = Com que g ' ( x ) = i h (x ) sin x, aplicant la regla de
x x2 x
la cadena:
2x 3 4x
b) f ( x ) = 1
3x 2 f (x ) = ( sin x ) = tg x
cos x
f (x ) =
( 6x 2 4 ) ( 3x 2) 3 · ( 2x 3 4x )
=
2
( 3x 2 ) c) f ( x ) = 3x 2 + 6x
12x 3 12x 2 +8 f (x ) és composició de g ( x ) = x i h (x ) 3x 2 6x; per
= 2
( 3x 2 ) tant:
x f ( x ) = g (h ( x ) ) = g ( 3x 2 + 6x ) = 3x 2 + 6x
2 x
2 x x
c) f ( x ) = f (x ) = =
x x2 1
Com que g ' ( x ) = i h (x ) 6x 6, aplicant la regla
2 x
x 2 x x de la cadena:
= =
x2 x2
6x + 6 3x + 3
x f (x ) = =
ln x 2 3x 2 + 6x 3x 2 + 6x
x x ln x 1
d) f ( x ) = f (x ) = =
ln x ln2 x ln2 x d) f ( x ) = e x 2 +2x
sin x
cos x · ln x f (x ) és composició de g (x ) e x i h (x ) x2 2x; per tant:
ln x x
e) f ( x ) = f (x ) = =
sin x sin2 x f ( x ) = g (h ( x ) ) = g ( x 2 + 2x ) = e x 2 +2x
1 cos x · ln x
= Com que g (x ) ex i h (x ) 2x 2, aplicant la regla de la
x · sin x sin2 x
cadena:
ex
ex x f ( x ) = e x 2 +2x ( 2x + 2 )
ex 2 x
f) f ( x ) = f (x ) = =
x x
e) f ( x ) = ex
1
ex x f (x ) és composició de g ( x ) = x i h (x ) e x; per tant:
= 2 x
x
f ( x ) = g (h ( x ) ) = g (e x ) = ex .
25. f (x ) 2x 2 4x 6 g (x ) x2 2x 1
Com que g ( x ) = i h (x ) e x, aplicant la regla de
f (x ) 4x 4 g (x ) 2x 2 2 x
la cadena:
a) h (x ) f (x ) g (x ) h (x ) 2x 2
ex
b) i (x ) g (x ) f (x ) i (x ) 6x 6 f (x ) =
2 ex
c) j (x ) f (x ) · g (x ) j (x ) (4x 4)( x 2 2x )
f) f (x ) sin(cos x )
(2x 2 4x 6)( 2x 2) 8x 3 24x 2 28x 12
f (x ) és composició de g (x ) sin x i h (x ) cos x; per tant:
f (x )
d) k ( x ) =
g (x ) f (x ) g (h (x )) g (cos x ) sin(cos x )
( 4x 4) ( x2 + 2x ) ( 2x 2 4x + 6 ) ( 2x + 2 ) Com que g (x ) cos x i h (x ) sin x, aplicant la regla de
k (x ) = 2
=
( x 2 + 2x ) la cadena:
12x 12 f (x ) sin x · cos(cos x )
= 2
( x 2 + 2x ) g) f (x ) cos(ln x )
f (x ) és composició de g (x ) cos x i h (x ) ln x; aleshores:
26. a) f (x ) (2x 2 4x 3)3
f (x ) g (h (x )) g (ln x ) cos(ln x )
f (x ) és composició de g (x ) x 3 i h (x ) 2x 2 4x 3; ales- 1
hores: Com que g (x ) sin x i h ( x ) = , aplicant la regla de
la cadena: x
f (x ) g (h (x )) g (2x 2 4x 3) (2x 2 4x 3)3
sin (ln x )
Com que g (x ) 3x 2 i h (x ) 4x 4, aplicant la regla de la f (x ) =
cadena: x

164
BLOC 3. ANÀLISI > UNITAT 12. DERIVADES

x +1 sin x sin x
h) f ( x ) = f ( x ) = g (h ( x ) ) = g = ln
x x x
x +1
f (x ) és composició de g ( x ) = x i h (x ) = ; per 1 x cos x sin x
x Com que g ( x ) = i h (x ) = , aplicant la
tant: x x2
regla de la cadena:
x +1 x +1
f ( x ) = g (h ( x ) ) = g =
x x x cos x sin x
x2 x cos x sin x
1 1 f (x ) = = =
sin x x sin x
Com que g ( x ) = i h (x) = , aplicant la regla de
2 x x2 x
la cadena: 1
1 = cotg ( x )
f (x ) = x
x +1
2 · x2
x f) f (x ) sin (ln x )
f (x ) és composició de g (x ) sin x i h (x ) ln x; aleshores:
27. a) f ( x ) = sin x
f (x ) g (h (x )) g (ln x ) sin(ln x )
f (x ) és composició de g ( x ) = x i h (x ) sin x; per tant: 1
Com que g (x ) cos x i h ( x ) = , aplicant la regla de la
f ( x ) = g (h ( x ) ) = g ( sin x ) = sin x cadena: x

1 cos (ln x )
Com que g ( x ) = i h (x ) cos x, aplicant la regla f (x ) =
2 x x
de la cadena:
cos x 2x si x 1
f (x ) =
2 sin x 28. a) f ( x ) =
4x si x > 1

(
b) f ( x ) = ln cos x ) 3x 2 3 si x 2
f (x ) és composició de g (x ) ln x i h ( x ) = cos x , i, al b) f ( x ) = 1
si x > 2
seu torn h (x ) és composició de i (x ) cos x i j ( x ) = x 2 x 2
per tant:
sin x si x 0
f ( x ) = g (h ( x ) )
f ( x ) = g (i ( j ( x ) ) ) c) f ( x ) = e x si 0 < x < 1
g ( x ) = i ( j ( x )) e x ( x + 1) si x > 1
1 1
Com que g ( x ) = , i (x ) sin x i j ( x ) = , apli-
x 2 x 29. a) f (x ) 2x 6
cant la regla de la cadena:
f (x ) 2
sin x tg x
f (x ) = = 2 (x + h) 6 ( 2x 6 ) = lim 2 = 2
2 · cos x · x 2 x f ( x ) = lim
h 0 h h 0
c) f (x ) a tg x
b) f (x ) x
f (x ) és composició de g (x ) a x i h (x ) tg x; per tant:
f (x ) 1
f (x ) g (h (x )) g (tg x ) a tg x
1
f ( x ) = lim
( x + h ) ( x ) = lim 1 = 1
Com que g (x ) ln(a) · a x i h ' ( x ) = , aplicant la
cos2 x h 0 h h 0

regla de la cadena:
c) f ( x ) = x +1
ln a · a tg x
f (x ) = 1
cos2 x f (x ) =
2 x +1
d) f (x ) cos(x 2)
x +h +1 x +1
f (x ) és composició de g (x ) cos x i h (x ) x 2: f ( x ) = lim ·
h 0 h
f (x ) g (h (x )) g (x 2) cos(x 2) x +h +1 + x +1
· =
Com que g (x ) sin x i h (x ) 2x, aplicant la regla de la x +h +1 + x +1
cadena: h 1
= lim =
f (x ) 2x sin(x 2)
h 0
h ( x +h +1 + x +1 ) 2 x +1

sin x 2x
e) f ( x ) = ln d) f ( x ) =
x x 5
sin x 2 ( x 5) 2x 10
f (x ) és composició de g (x ) lnx i h ( x ) = , per tant: f (x ) = 2
= 2
x ( x 5) ( x 5)

165
BLOC 3. ANÀLISI > UNITAT 12. DERIVADES

2 (x + h) 2x c) f (x ) 2ex en x 0

f ( x ) = lim
(x + h) 5 x 5
=
L’equació de la recta tangent és determinada per:
h 0 h y f (a) f (a)(x a)
2 (x + h) (x 5) 2x ( x + h 5 ) + 2x ( x 5)
= lim = Calculem la derivada de la funció:
h 0 h
= lim
( x 5 ) · 2h lim
2xh
= f (x ) 2e x
h 0 h (x
5) ( x + h 5) h 0 h (x 5) ( x + h 5)
f (0) 2
= lim
( x 5) ·
h 0 (x
f (0) 2
5) ( x + h 5)
2h 2x h L’equació de la recta tangent és:
· lim lim · lim =
h 0 h h 0 ( x 5) ( x + h 5) h 0 h y 2 2(x 0) y 2x 2
2 (x 5) 2x 10 L’equació normal és determinada per:
= 2 2
= 2
( x 5) ( x 5) ( x 5) 1
y f (a ) = (x a)
f (a )
30. a) f (x ) ln(x 1) en x 2
Si substituïm pels valors obtinguts anteriorment:
L’equació de la recta tangent és determinada per:
1 x
y f (a) f (a)(x a) y 2= ( x 0) y = +2
2 2
Calculem la derivada de la funció:
d) f (x ) ln (x 1) en x 0
1
f (x ) =
x +1 L’equació de la recta tangent és determinada per:

f (2) 0,333 y f (a) f (a)(x a)

f (2) 1,098 Calculem la derivada de la funció:


L’equació de la recta tangent és: 1
f (x ) =
y 1,098 0,333(x 2) y 0,333x 1,764 x +1
f (0) 1
L’equació normal és determinada per:
1 f (0) 0
y f (a ) = (x a)
f (a ) L’equació de la recta tangent és:

Si substituïm pels valors obtinguts anteriorment: y 0 1(x 0) y x

1 L’equació normal és determinada per:


y 1,098 = ( x 2) y = 3x + 7,098
0,333 1
y f (a ) = (x a)
f (a )
b) f ( x ) = x 2 en x 6
L’equació de la recta tangent és determinada per: Si substituïm pels valors obtinguts anteriorment:

y f (a) f (a)(x a) 1
y 0= ( x 0) y = x
1
Calculem la derivada de la funció:
1 31. f (x ) x 3 3x 2 x 5 en el punt A (3, 2)
f (x ) =
2 x 2
1 L’equació de la recta tangent és determinada per:
f (6) =
4 y f (a) f (a)(x a)
f (6) = 2 Calculem la derivada de la funció:

L’equació de la recta tangent és: f (x ) 3x 2 6x 1


x 6 x +2 f (3) 8
y 2= y =
4 4 f (3) 2
L’equació normal és determinada per: L’equació de la recta tangent és:
1 y 2 8(x 3) y 8x 22
y f (a ) = (x a)
f (a )
L’equació normal és determinada per:
Si substituïm pels valors obtinguts anteriorment: 1
y f (a ) = (x a)
y 2 4(x 6) y 4x 26 f (a )

166
BLOC 3. ANÀLISI > UNITAT 12. DERIVADES

Si substituïm pels valors obtinguts anteriorment: 34. a) f ( x ) = 2 x


1 x 19 1
y 2= ( x 3) y = + El pendent es correspon amb ; per tant:
8 8 8 f (x )
1
32. La recta que compleix aquestes condicions és la recta normal =1 f (x ) = 1
f (x )
de f (x ) en x 6; per tant:
f (x ) 2x 1 f ( 6) 13 f ( 6) 36 Derivem la funció per trobar el punt x pel qual ha de passar
la recta normal:
1
y f (a ) = (x a)
f (a ) 1
f (x ) = f (x) = 1 x =1
1 x + 474 x
y 36 = ( x + 6) y =
13 13 f (1) 2
Per tant, l’equació de la recta tangent de f (x ) en x 1 és:
33. a) f (x ) x 2 4
1
El pendent es correspon amb f (x ); per tant: y +2= ( x 1) y =x 3
1
f (x ) 2x
b) f (x ) x3 11x
f (x ) 2 2x 2 x 1
1
f (1) 3 El pendent es correspon amb ; per tant:
f (x )
Així, l’equació de la recta tangent de f (x ) en x 1 és: 1
=1 f (x ) = 1
y 3 2(x 1) y 2x 5 f (x )
2
b) f ( x ) = Derivem la funció per trobar el punt x pel qual ha de passar
x la recta normal:
El pendent es correspon amb f (x ); per tant:
f (x ) 3x 2 11
2
f (x ) = f (x ) 1 3x 2 11 1 x ±2
x2
f ( 2) 14; f (2) 14
2
f (x ) = 2 =2 x = ±1
x2 Per tant, l’equació de la recta normal de f (x ) en x 2 és:
f ( 1) = 2; f (1) = 2 1
y + 14 = ( x + 2) y =x 12
1
Així, l’equació de la recta tangent de f (x ) en x 1 és:
I l’equació de la recta normal de f (x ) en x 2 és:
y 2 2(x 1) y 2x 4
1
I l’equació de la recta tangent de f (x ) en x 1 és: y 14 = ( x 2) y = x + 12
1
y 2 2(x 1) y 2x 4
c) f (x ) x2 7x 7
c) f (x ) x2 4x
1
El pendent es correspon amb ; per tant:
El pendent es correspon amb f (x ); per tant: f (x )
f (x ) 2x 4 1
=1 f (x ) = 1
f (x )
f (x ) 2 2x 4 2 x 3
f (3) 3 Derivem la funció per trobar el punt x pel qual ha de passar
la recta normal:
Així, l’equació de la recta tangent de f (x ) en x 3 és:
f (x ) 2x 7
y 3 2(x 3) y 2x 9
f (x ) 1 2x 7 1 x 3
d) f (x ) ln x
f (3) 5
El pendent es correspon amb f (x ); per tant:
Per tant, l’equació de la recta normal de f (x ) en x 3 és:
1 1
f (x ) = y +5 = ( x 3) y =x 8
x 1
1
f (x ) = 2 =2 x = 0,5 d) f (x ) 0,5x 2 2
x
1
f (0,5) 0,69 El pendent es correspon amb ; per tant:
f (x )
Així l’equació de la recta tangent de f (x ) en x 0,5 és: 1
=1 f (x ) = 1
y 0,69 2(x 0,5) y 2x 1,69 f (x )

167
BLOC 3. ANÀLISI > UNITAT 12. DERIVADES

Derivem la funció per trobar el punt x pel qual ha de passar Com que els valors coincideixen, la funció és contínua en
la recta normal: x 0. Ara, n’estudiarem la derivabilitat observant com es
comporta la funció derivada al voltant de x 1.
f (x ) x
f (x ) 1 x 1 1 si x < 1
f (x ) =
1 si x 1
f ( 1) 2,5
Per tant, l’equació de la recta normal de f (x ) en x 2,5 f (1 ) 1 f (1 ) 1
és: Els valors entorn del punt no coincideixen; per tant, la fun-
1 ció no és derivable en x 1.
y 2,5 = ( x + 1) y = x + 3,5
1 1
b) f ( x ) = 2
( x 1)
35. Perquè sigui horitzontal, el pendent ha de ser nul; és a dir, Igualem el denominador a 0 per trobar els punts en què
f (x ) 0. Per tant: aquest s’anul·la:
2 (x 1)2 0 x 1
x1 =
f ’ ( x ) = 3x 2 8x + 4 = 0 3 Podem reduir l’estudi de la funció f (x ) en el punt x 1:
x2 = 2
lim f ( x ) = +
x 1
Així doncs, calculem el valor de f (x ) en aquests punts:
lim+ f ( x ) = +
x 1
f ( 2 ) = 10
2 238 Per tant, la funció f (x ) no és contínua en x 1; en conse-
f = qüència, tampoc no és derivable en aquest punt.
3 27
x 1 si x < 0
2 238 c) f ( x ) =
Els punts són , i (2, 10). cos x si x 0
3 27
Podem reduir l’estudi de la funció f (x ) en el punt x 0:

x 2 lim f ( x ) = lim x 1= 1
36. f ( x ) = x 0 x 0
x +1 lim+ f ( x ) = lim+ cos x = 1
x 0 x 0
L’equació de la recta tangent és determinada per:
f (0) = 1
y f (a) f (a)(x a)
Calculem la derivada de la funció: Com que els valors de tots dos límits coincideixen amb el
valor de la funció en x 0, podem assegurar que la funció
3 1
f (x ) = 2
f (2) = f (2) = 0 és contínua. Ara, estudiarem la derivabilitat de la funció:
( x + 1) 3
1 si x < 0
f (x ) =
L’equació de la recta tangent és: sin x si x 0

1 x 2
y 0= ( x 2) y = Vegem com es comporta la derivada al voltant de x 0:
3 3 3
f (0 ) 1 f (0 ) 0
L’equació normal, si substituïm pels valors obtinguts anterior-
Els valors entorn del punt no coincideixen; per tant, la fun-
ment, és:
ció no és derivable en x 0.
1
y 0= (x 2) y = 3x + 6 d) E (x ) [x]
1
3 E (x ) és la funció que li assigna a un nombre la seva part
entera.
37. a) f ( x ) = x 1 Per tant, si representem gràficament la funció:
f (x ) és equivalent a: Y
5
x +1 si x < 1 E(x) = [x]
f (x ) = 4
x 1 si x 1
3

Podem reduir l’estudi de la funció f (x ) en el punt 2

x 1, ja que en la resta del domini és contínua i derivable 1


com en tots els polinomis:
–4 –3 –2 –1 0 1 2 3 4 5 6 7 X
lim f ( x ) = lim ( x + 1) = 0 –1
x 1 x 1
–2
lim+ f ( x ) = lim+ ( x 1) = 0
x 1 x 1 –3
f (0) = 0

168
BLOC 3. ANÀLISI > UNITAT 12. DERIVADES

Podem observar clarament que la funció no és contínua ni, Vegem com es comporta la derivada al voltant de
per tant, derivable quan x té un valor enter. x 1:
2 f (1 ) 2 f (1 ) 0,25
e) f ( x ) =
x +1
Els valors entorn del punt no coincideixen; per tant, la fun-
Igualem el denominador a 0 per trobar els punts en què ció no és derivable en x 1.
aquest s’anul·la:
h) f ( x ) = sin x
x 1 0 x 1
Estudiarem l’interval en què x va entre 0 i 2 :
Podem reduir l’estudi de la funció f (x ) en el punt x 1:
lim f sin x si 0 < x
(( xx )) == f (x ) =
x lim1 f
x 1 sin x si <x <2
lim + f (( xx )) == ++
x lim1+ f
x 1
Podem reduir l’estudi de la funció f (x ) al punt x = :
Per tant, la funció f (x ) no és contínua en x 1, ni per tant
derivable en aquest punt. lim f ( x ) = lim sin x = 0
x x

f) f ( x ) = 2x + 4 lim+ f ( x ) = limx sin x = 0


x x

f (x ) és equivalent a: f( )=0

2x 4 si x < 2
f (x ) = Com que els valors de tots dos límits coincideixen amb
2x + 4 si x 2 el valor de la funció en x = , podem assegurar que la
funció és contínua. Ara, estudiarem la derivabilitat de
Podem reduir l’estudi de la funció f (x ) en el punt x 2: la funció:
lim f ( x ) = lim 2x 4 = 0 cos x si 0 < x
x
lim2 f ( x ) = x lim2 2x 4 = 0 f (x ) =
x lim 2 ff
lim ((xx))== x lim
lim
2 2x 2x + 44==00 cos x si <x <2
x x 22+
lim + f ( x ) = x x lim
22+
2x +4=0
x lim2 f ( x )f = 0x lim
x 2+
( x ) =22++02x + 4 = 0 Vegem com es comporta la derivada al voltant de x = :
f (0) = 0
f (0) = 0
f ( )=1
Com que els valors de tots dos límits coincideixen amb el f ( +) = 1
valor de la funció en x 2, podem assegurar que la fun-
ció és contínua. Ara, estudiarem la derivabilitat de la Els valors entorn del punt no coincideixen; per tant, la fun-
funció: ció no és derivable en x = .
2 x < 2 Es tracta d’una funció contínua, però no derivable per a
f (x ) =
2 x 2 tots els valors de x que són múltiples de .

Vegem com es comporta la derivada al voltant de x 2:


ax 2 + b si x < 1
38. f ( x ) =
f ( 2) 2 f ( 2) 2 bx + 3 si x 1

Els valors entorn del punt no coincideixen; per tant, la fun- Perquè f (x ) sigui derivable en x 1, que és el punt que cal
ció no és derivable en x 2. estudiar, s’ha de complir en primer lloc la condició que la
funció f (x ) sigui contínua en aquest punt; per tant:
x2 + 1 si x < 1
g) f ( x ) = lim f ( x ) = lim f ( x )
xlim1 f x = xlim 1+ f x
x+3 si x 1 ( ) ( )
limxlim1 2
ax f +x b == xlim
( ) 1+
lim fbxx + 3 ( )
xlimx1 ax
1 2 + b = xxlim 11++ bx + 3
Podem reduir l’estudi de la funció f (x ) en el punt x 1:
x a1+ b 2= b + 3 x 1 a = 3
lim ax + b = lim+ bx + 3
+

lim f ( x ) = lim x 22 + 1 = 2 x a1+ b = b + 3 x 1 a = 3


xlim1 f ( x ) = xlim 1 x2 + 1 = 2 a +b = b +3 a=3
xlim1 f ( x ) = xlim 1 x +1 = 2
lim
x 1f ( x ) = lim x 1 x+3 =2
xlim 1+ f ( x ) = xlim 1+ x+3 =2 Una vegada es compleix que la funció és contínua, ha de ser
xlim 1+ f ( x ) = xlim 1+ x+3 =2
x 1+ f (x1) 1=+ 2 derivable en x 1. Perquè ho sigui, els valors de la derivada
f (1) = 2
f (1) = 2 entorn del punt han de ser iguals:

Com que els valors de tots dos límits coincideixen amb 2ax x <1
el valor de la funció en x 1, podem assegurar que la f (x ) =
b x 1
funció és contínua. Ara, estudiarem la derivabilitat de la
funció: f (1 ) = f (1+ )
2x si x < 1 2a = b b =6
f (x ) = 1
si x 1 Per tant, els valors perquè f (x ) sigui derivable són a 3 i
2 x+3 b 6.

169
BLOC 3. ANÀLISI > UNITAT 12. DERIVADES

39. a) f (x ) 4x f (x ) 4 f (x ) 0 41. a) f (x ) 4x 3 5x 2 f (x ) 12x 2 10x f (1) 22


b) f (x ) 3x 2 f (x ) 3 f (x ) 0 f (x ) 24x 10 f (1) 34
c) f (x ) x2 3x 2 f (x ) 2x 3 f (x ) 2 f 3(x ) 24 f 3(1) 24
d) f (x ) x2 16 f (x ) 2x f (x ) 2 1
b) f ( x ) =
x
e) f (x ) x3 3x 2 f (x ) 3x 2 3 f (x ) 6x
1
f (x ) = f (1) = 1
1 1 x2
f) f ( x ) = x 1 f (x ) = f (x ) =
2 x 1 4 ( x 1)
3
2
f (x ) = f (1) = 2
x3
g) f (x ) x4 2x f (x ) 4x 3 2 f (x ) 12x 2
6
f 3 (x ) = f 3 (1) = 6
h) f (x ) (x 2)3 f (x ) 3(x 2)2 f (x ) 6(x 2) x4

40. a) f (x ) 4x 3 2x 2 3x 80 f (x ) 12x 2 4x 3 c) f ( x ) = 2x + 2

f (x ) 24x 4 f 3(x ) 24 1 1
f (x ) = f (1) =
2x + 2 2
f 4(x ) 0 3 1
f (x ) = ( 2x + 2 ) 2 f (1) =
b) f (x ) ex f 4(x ) ex 8
1
c) f ( x ) = ln x f (x ) = 3 5 3
x f 3 (x ) = ( 2x + 2 ) 2 f 3 (1) =
2 32
1 2
f (x ) = f 3 (x ) =
x2 x3 d) f (x ) e 2x
6 f (x ) 2e 2x f (1) 2e 2
f 4 (x ) = = 6x 4
x4
f (x ) 4e 2x f (1) 4e 2
d) f (x ) sin x f (x ) cos x
f 3(x ) 8e 2x f 3(1) 8e 2
f (x ) sin x f 3(x ) cos x
e) f (x ) x · ln x
f 4(x ) sin x
f (x ) ln x 1 f (1) 1
e) f (x ) cos x f (x ) sin x
1
f (x ) cos x f 3(x ) sin x f (x ) = f (1) = 1
x
f 4(x ) cos x 1
f 3 (x ) = f 3 (1) = 1
1 x2
f) f ( x ) = f (x ) (x 2) 2
x 2 f) f (x ) x2
f (x ) 2(x 2) 3 f 3(x ) 6(x 2) 4
f (x ) 2x f (1) 2
f 4(x ) 24(x 2) 5
f (x ) 2 f (1) 2
g) f (x ) cos(2x ) f (x ) 2 sin(2x ) f 3(x ) 0 f 3(1) 0
f (x ) 4 cos(2x ) f 3(x ) 8 sin(2x )
42. a) v (0) f (0) 3 m/s
f 4(x ) 16 cos(2x )
b) v (2) 16,6 m/s
h) f (x ) 3x 4 70x f (x ) 12x 3 70
c) v (t ) 3 9,8 t
f (x ) 36x 2 f 3(x ) 72x
d) a (0) v (0) 9,8 m/s2
f 4(x ) 72
3 2 1 e) a (t ) v (t ) 9,8 m/s2
i) f ( x ) = 2x 3 f (x ) = x 2
2 f) f (x ) 3 9,8 t
1 3
3 2 3 2
f (x ) = x 2 f (x ) =
3 x 2 f (x ) 9,8
4 8
5 f n(x ) 0
9 2
f 4 (x ) = x 2
16 43. f (x ) x · ex
j) f (x ) 40x 3 2x 2 f (x ) 120x 2 4x f (x ) ex x · ex ex(x 1)
f (x ) 240x 4 f 3(x ) 240 f (x ) x· ex 2ex ex(x 2)
f 4(x ) 0 f n(x ) (x n)ex

170
BLOC 3. ANÀLISI > UNITAT 12. DERIVADES

44. a) f (x ) (2x 4 4x 2 3)5 b) f (x ) sin x

f (x ) 5(8x 3 8x )(2x 4 4x 2 3)4


f (x) = cos x = sin +x
b) f (x ) ln(cos(sin x )) 2
f ( x ) = sin x = [ sin ( + x ) ] =
sin ( sin x ) · cos x
f (x ) =
cos ( sin x ) = sin 2 +x
2
c) f (x ) ln(arc cos2 x )
f 3 (x ) = cos x = sin +x =
2
2
f (x ) =
arc cos x · 1 x2 = sin 3 +x
2
2x 3 x2 +1
d) f ( x ) = f n ( x ) = sin n +x
3x 4 2 2
6x 6 + 6x 5 12x 3 12x 2 + 4x c) f (x ) x12 x4 2
f (x ) = 2
( 3x 4 2)
La derivada enèsima de la funció polinòmica per a n 12
cos x 2 + sin x és:
e) f ( x ) =
ex
f n(x ) 0
2x 3 · sin x 2 + cos x cos x 2 sin x
f (x ) = d) f (x ) ex e x
ex
x f (x ) ex e x
f) f ( x ) =
3 x f (x ) ex e x

3
f (x ) = 2
f n(x ) ex ( 1)n 1· e x

2 x · 3 ( x ) e) f (x ) e 2x
g) f (x ) e cos(3x ) f (x ) 2e 2x
f (x ) 3 sin (3x ) · e cos(3x ) f (x ) 4e 2x

h) f ( x ) = tg x f n(x ) 2n · e 2x

1 f) f (x ) cos 2x
f (x ) =
2 cos2 x tg x
f ( x ) = 2 sin ( 2x ) = 2 cos 2x +
2
i) f ( x ) = tg 2x
f ( x ) = 4 cos ( 2x ) = 4 cos 2x + 2
1 2
f (x ) =
cos2 2x · 2x
f n ( x ) = 2n cos 2x + n
1 2
j) f ( x ) = sin cos
x
g) f (x ) x 1
1 1
sin cos cos f (x ) x 2
x x
f (x ) =
x2 f (x ) 2x 3

tg x f 3(x ) 6x 4
k) f ( x ) =
x
f n(x ) ( 1)n · n! · x (n 1)
tg x
x 1
f (x ) = 2x 46. a) f ( x ) = A cos ( t + )
cos2 x · x
b) f ( x ) = A 2 sin
( t+ )
l) f ( x ) = 7 1+x 2
c) f ( x ) = A
3 3 cos ( t + )
2x · ln 7 · 71+x 2 f 4 (x ) = A 4 sin
( t+ )
f (x ) =
2 71+x 2
si n 2k
k
45. a) f (x ) ln x f (x ) x 1 f (x ) x 2 f n (x ) = ( 1) A n sin ( t )
f 3(x ) 2x 3 f 4(x ) 6x 4 si n 2k 1
k
f n(x ) ( 1)n 1 · (n 1)! · x n f n (x ) = ( 1) A n cos ( t + )

171
BLOC 3. ANÀLISI > UNITAT 12. DERIVADES

47. Per a poder formar el triangle, primerament hem de trobar la Y


recta normal:
8
f (x ) cos(2x ) 6
y = 4x – 24
f (x ) 2 sin(2x ) y = x2 – 6x + 1
4

2
f =0 f = 2
4 4
–6 –4 –2 0 2 4 6 8 X
–2
Per tant, la recta normal és:
–4

1 x –5
y 0= x y =
2 4 2 8 –8

Busquem els punts de tall de la recta amb els eixos de coor-


50. f (x ) x2 1
denades quan x = 0 y = i quan y = 0 x = .
8 4 f (x ) 2x
Així doncs, la gràfica resultant és: f (1) 2 f (1) 0
Per tant, l’equació de la recta tangent és: y 2x 2
Y
b ·h 1·2
A= = =1
2 2

O
X
51. f (x ) (x 2 4) en x 1ix 1
4
Busquem les dues rectes tangents:

8
f (x ) 2x f ( 1) 2 f (1) 2

x f (1) f ( 1) 3
y
2 8 Quan x 1, la recta tangent de f (x ) és:

Per tant, el valor de l’àrea és: y 3 2(x 1) y 2x 5


Quan x 1, la recta tangent de f (x ) és:
·
b·a y 3 2(x 1) y 2x 5
A= = 8 4 = 0,15
2 2
Y

48. f (x ) x2 x 2 5

Els punts de tall de la funció amb l’eix d’abscisses verifiquen 4

que: 3

x1 = 2 2
f (x ) = 0 x2 + x 2=0 x =
x2 = 1 1

Per tant, procedim a buscar la recta tangent en els punts en –3 –2 –1


0
1 2 3 4 X
els quals la funció f (x ) talla l’eix d’abscisses: –1

f (x ) 2x 1 f ( 2) 3 f (1) 3 –2

Quan x 1, la recta tangent de f (x ) és:


Per tant, l’àrea és:
y 0 3 (x 1) y 3x 3
Quan x 2, la recta tangent de f (x ) és: b·a 5·5
A= = = 12,5
2 2
y 0 3(x 2) y 3x 6

49. f (x ) x 2 6x 1 52. a) f (x ) x 2 4x 1
Si la recta tangent de f (x ) ha de ser paral·lela a la recta Si la recta tangent de f (x ) ha de ser paral·lela a la recta
y 4x, la condició perquè siguin paral·leles és que tinguin el y 4x, la condició perquè siguin paral·leles és que tinguin
mateix pendent; per tant, s’ha de complir que: el mateix pendent; per tant, s’ha de complir que

f (x ) 4 2x 6 4 x 5 f (x ) 4 2x 4 4 x 4
Així doncs, busquem l’equació de la recta tangent de f (x ) en Així doncs, busquem l’equació de la recta tangent de f (x )
x 5: en x 4:
y 4 4(x 5) y 4x 24 y 1 4(x 4) y 4x 15

172
BLOC 3. ANÀLISI > UNITAT 12. DERIVADES

b) La recta normal, a partir dels valors obtinguts en l’apartat


anterior, és: 4 APLICACIÓ DE
LES DERIVADES Pàgs. 304 i 305
1 x
y 1= ( x 4) y = +2
4 4 55. a) f (x ) x 2 3x 2
c) En l’apartat a hem obtingut que la recta és tangent a la f (x ) 2x 3 f (x ) 0 x 3/2
funció en el punt x 4; per tant, f (4) 1 i, aleshores, el
punt és (4, 1). x 0 x 2

f (x )
53. Sabem que f i f 1 compleixen que:
f (x )
f (f 1(x) ) = x
3
Per tant, f (x ) és decreixent en , i creixent en
Si derivem aquesta expressió, aplicant la regla de la cadena 2
en el primer terme, obtenim que: 3
,+ .
2
1
f (f 1(x) ) (f 1(x)
) =1 (f 1(x)
) =
f (f 1(x) ) b) f (x ) x2 4
f (x ) 2x f (x ) 0 x 0
54. a) Com hem deduït en l’exercici anterior:
x 1 x 1
1
(f 1(x)
) = f (x )
f (f 1(x) )
f (x )
f ( x ) = arc sin ( x ) ;
f ( x ) = arc sin ( x ) ; Per tant, f (x ) és decreixent en , 0 ) i creixent en ( 0, +
ff (1x()f = ( xarc
) ) =sinsin( x(f) (; x ) ) ( )
f (1x()f = ( xarc
) =sin sin( xf) (; x ) .
1 f x ) = sin (f x )
f (1x() = ( arc
) ) sin ( x(1 ) (; ) ) 1
fcos((ff ((xx)))) == sin(ff (1f (((xxx))) ) == 1 c) f (x ) x3
fcos 1 (ff (xx ) ) == sin
( f x (arc 1
sin(x))
cos((f ( x ))) = (f (11 ( x ) )) = (arc sin(x))
( ) ( )
1
cos (f ( x ) ) = (f ( x ) ) =1 (arc sin(x)) 1 f (x ) 3x 2 f (x ) 0 x 0
arc(fsin
(cos ( x()x) )=) (=f 1 ( x ) ) =1 (arc sin(x)) =
( arc sin ( x ) ) = cos ( arc1sin (arc xsin(x))
=
( arc sin ( x ) ) = cos ( arc1sin ( x )) )) = ( x 1 x 1
( arc sin ( x ) ) 1= cos ( arc1sin ( x ) ) 1=
=( arc sin ( x ) ) 11= cos ( arc sin== ( x ) ) 11= 2 f (x )
= 1 sin2 ( arc cos
sin ( xarc ) sin ( x )1) x
= 1 sin2 ( arc 1 sin ( x ) )) = 1 1 x 22 f (x )
= 1 sin2 ( arc 1 sin ( x ) ) = 1 1 x
= 1 sin2 ( arc sin ( x ) ) = 1 x 2
1 sin2 ( arc sin ( x ) ) 1 x2 Per tant, f (x ) és creixent per a tota x.
d) f (x ) x4
b) f ( x ) = arc cos ( x ) ; f (x ) 4x 3 f (x ) 0 x 0

f 11 (f ( x ) ) = cos (f ( x ) ) x 1 x 1
f 1 (f ( x ) ) = cos (f ( x ) )
f (f ( x ) ) = cos (f ( x ) ) 1
f sin
1 f x
( (f( ( x) ) )==cos
(f (1f ((xx)) ) = 1 f (x )
sin (f ( x ) ) = (f 11 ( x ) ) = ( arc cos 1 ( x ))
sin (f ( x ) ) = (f ( x ) ) = ( arc cos 1 ( x )) f (x )
sin (f ( x ) ) = (f 1 ( x ) ) = 1( arc cos ( x ) )
arc cos ( x ) ) =
( arc 1( arc cos (=x ) )
( arc cos ( x )) = ( arc1cos ( x ) ) =
( cos ( x ) ) = sin sin ( arc1cos ( x ) ) =
f (x ) és decreixent en ( , 0 ) i creixent en ( 0, + ).
( arc cos ( x ) ) =1 sin ( arc cos ( x ) ) = 1 e) f (x ) x3 12x
= 1 sin ( arc cos=( x ) ) 1
= 1 cos22 ( arc 1 cos ( x ) ) = 1 1 x 22
= 1 cos2 ( arc 1 cos ( x ) ) = 1 1 x2 x1 = 2
= 1 cos ( arc cos ( x ) ) = 1 x f ( x ) = 3x 2 12 f (x ) = 0 x =
1 cos2 ( arc cos ( x ) ) 1 x2 x2 = 2

x 3 x 0 x 3
c) f ( x ) = arc tg ( x ) ;
f (x )
f ((ff (( xx )) )) == tg
1
(f ( x ) )
f 1 tg (f ( x ) ) f (x )
f 1
(f (1x ) ) = tg (f ( x1 ) ) 1
1 = (f 1(x) ) = 1
cos22 (1f ( x ) ) = (f 1(x) ) = ( arc tg 1 ( x )) f (x ) és decreixent en ( 2, 2) i creixent en ( , 2) ( 2, + )
cos (f ( x ) ) = (f (x) ) = ( arc tg ( x ) )
cos2 (f ( x ) ) ( arc tg ( x ) )1 ( , 2) ( 2, + ) .
arc tg x = cos2 f x = 1
(( arc tg (( x )) )) = cos22 ((f (( x )) )) = 1 + tg22 ( arc
=
1 tg ( x ) ) = f) f (x ) 4x 80
( arc tg ( x ) ) = cos (f ( x ) ) = 1 + tg2 ( arc tg ( x ) ) =
1 1 + tg ( arc tg ( x ) ) f (x ) 4
= 1
= 1 +1x 2
= 1 + x2 f (x ) 0 per a tot D (f ); per tant, la funció és creixent en tot
1 + x2 D (f ).

173
BLOC 3. ANÀLISI > UNITAT 12. DERIVADES

g) f (x ) 2 · ln x x 1 x 1
2
f (x ) = f (x )
x
f (x )
El domini d’una funció logarítmica amb base més gran que
1 és D (f ) ( 0,+ ) . En tot aquest interval, la seva derivada La funció passa de decréixer a créixer en x 0, que, per
és positiva, de manera que la funció és creixent per a tot el tant, és un mínim.
seu domini.
f) f (x ) 3x 20
h) f (x ) x5
f (x ) 3
f (x ) 5x 4 f (x ) 0 x 0
Com que f (x ) 0 per a tot D (f), la funció no presenta cap
x 1 x 1 extrem.
f (x )
f (x ) 57. a) a 2 = b 2 + c 2 a= b2 + c2
c
a (c ) = 9 + c2 a (c ) =
Per tant, f (x ) és creixent per a tot x. 9 + c2
c
56. a) f (x ) x 2 3x 2 9 + c2 ·c
b) a (c ) = 9 + c2 9
f (x ) 2x 3 f (x ) 0 x 3/2 2
=
3

x 0 x 2
( 9+ c2 ) (9 + c2 )
c) a (c) 0 c 0
f (x )
f (x ) D (f ) = [ 0, + ) a (1) 0; per tant, la funció és creixent en
tot D (f).
La funció passa de decréixer a créixer en x 3/2; per tant,
és un mínim. 58. a) f (t ) 2t 2 5t 1
b) f (x ) x2 4 f (t ) 4t 5 f (t ) 0 t 5/4
f (x ) 2x f (x ) 0 x 0
x 0 x 2
x 1 x 1 f (x )
f (x ) f (x )
f (x )
5 5
La funció passa de decréixer a créixer en x 0; per tant, és Decreixent en , i creixent en ,+ .
4 4
un mínim.
1 1
c) f (x ) 2x 3 3x 2 2 g (x ) = x4 x2
4 2
x1 = 0
f ( x ) = 6x 2 6x f (x ) = 0 x = x1 = 0
x2 = 1
g (x) = x 3 x g (x ) = 0 t = x2 = 1
x 0,5 x 0,5 x 2 x3 = 1

f (x )
x 2 x 0,5 x 0,5 x 2
f (x )
f (x )
La funció passa de créixer a decréixer en x 0 i, per tant, f (x )
és un màxim, i de decréixer a créixer en x 1, que és un
mínim. Creixent en ( 1, 0 ) (1, + ) i decreixent en ( , 1) ( 0,1)
d) f (x ) x3 12x 1 ( , 1) ( 0,1).
x1 = 2 b) En la funció f (x ), el punt x 5/4 és un mínim.
f ( x ) = 3x 2 12 f (x ) = 0 x =
x2 = 2 En la funció g (x ), el punt t 1 és un mínim.

x 3 x 0 x 3 59. a) f (x ) x 3 2x 2 x 1
f (x ) f ( x ) = 3x 2 + 4x + 1 f (x ) = 0
f (x ) x1 = 1
x =
La funció passa de créixer a decréixer en x 2 i, per tant, x 2 = 1/ 3
és un màxim, i de decréixer a créixer en x 2, que és un
mínim. x 2 x 1/2 x 0

e) f (x ) x2 2 f (x )
f (x )
f (x ) 2x f (x ) 0 x 0

174
BLOC 3. ANÀLISI > UNITAT 12. DERIVADES

La funció f (x ) és creixent en ( , 1) ( 1 / 3, + ) i de- c) Serà positiva en (0, 2), ja que és creixent en aquest interval
creixent en ( 1, 1/3), i presenta un màxim en x 1 i un i nul·la en (2, 5) perquè es manté constant.
mínim en x 1/3.
d) Serà positiva en ( 0, + ), ja que és creixent en aquest inter-
b) f (x ) sin x (2x ), x [ 0, ] val.
f (x ) 2cos (2x ) f (x ) 0 x1 = / 4; x 2 = 3 / 4
61. f (x ) x · e x
x = /8 x = /2 x =7 /8 a) f (x ) e x(x 1) f (x ) 0 x 1
f (x )
x 2 x 0
f (x )
f (x )

La funció f (x ) és creixent en ( 0, / 4 ) ( 3 / 4, ) i de- f (x )


creixent en ( / 4, 3 / 4 ) , i presenta un màxim en
x = / 4 i un mínim en x = 3 / 4. f (x ) és decreixent en ( , 1) i creixent en ( 1, + ).
c) f ( x ) = e x 2 1 b) f ( 1) 0,37.

f (x) = 2x · e x 2 1 f (x ) = 0 x =0 f (x ) presenta un mínim en ( 1, 0,37).

x 1 x 1 62. f (x ) x 4 2x 2 1
x1 = 0
f (x )
a) f ( x ) = 4x 3 4x f (x ) = 0 x = x2 = 1
f (x )
x3 = 1
La funció f (x ) és creixent en ( 0, + ) i decreixent en
x 2 x 1/2 x 1/2 x 2
( , 0 ), i presenta un mínim en x 0.
f (x )
d) f (x ) (x 2)3
f (x )
f (x ) 3(x 2)2
f (x ) 0 per a qualsevol valor de x; per tant, la funció és Creix en ( 1, 0 ) (1, + ) i decreix en ( , 1) ( 0,1) .
creixent en tot D (f). Malgrat que la derivada de f (x ) val 0 b) La funció f (x ) presenta un màxim en x 0 que correspon al
per a x 2, no presenta cap màxim ni mínim en aquest punt (0, 1), i mínims en x 1 i x 1 que corresponen als
punt, sinó un punt d’inflexió. punts ( 1, 0) i (1, 0) respectivament.
e) f (x ) 0,5x 2 2
63. Si el producte de totes dues variables és 128 significa que:
f (x ) x f (x ) 0 x 0
x·y 128
x 1 x 1 La suma de la quarta part del primer més la meitat del segon
f (x ) és:
x y
f (x ) P (x ) = +
4 2
La funció f (x ) és creixent en ( 0, + ) i decreixent en Substituïm la primera expressió en la segona perquè depen-
( , 0 ) , i presenta un mínim en x 0. gui únicament de x:
f) f (x ) ln (x 2 1) x 2 + 256
P (x ) =
2x 4x
f (x ) = f (x ) = 0 x =0
x2 +1 Derivem la funció i la igualem a 0 per buscar el seu mínim:
4x 2 1 024
x 1 x 1 P (x ) = =0 x = 16
16x 2
f (x )
Per tant:
f (x )
y 8.
La funció f (x ) és creixent en ( 0, + ) i decreixent en
64. f (x ) x 3 2x 2 x 4
( , 0 ) , i presenta un mínim en x 0. x1 = 1/ 3
a) f (x ) 3x 2 4x 1 f (x ) 0 x =
60. a) Serà positiva en ( ,1) ( 5, + ) , ja que la funció és crei- x2 = 1
xent en aquest interval. Serà negativa en (1, 5), on la fun-
ció és decreixent. x 1/4 x 1/2 x 2
f (x )
b) Serà positiva en (4n ( )
1) 4 , (4n + 1) 4 , ja que la funció
f (x )
és creixent en aquest interval. Serà negativa en (4n ( 3) 4 , (4n 1) 4 )
Per tant, el mínim es troba en x 1 any.
((4n 3) 4 , (4n )
1) 4 , ja que és decreixent en aquest interval.
b) f (1) 4 4 000 euros

175
BLOC 3. ANÀLISI > UNITAT 12. DERIVADES

65. Sabem que c 2 i que: f (x ) 12x 2; f (1,077) 0


a·b·c 18 a·b 9 La funció té un mínim en 1,077.
També sabem que l’àrea formada per les cares de l’ortoedre f) f ( x ) = 2x 8
és:
1
S = 2ab + 2ac + 2bc = 2ab + 4a + 4b f (x ) = f (x ) 0
2x 8
36
S ( a ) = 18 + 4a +
a La derivada no pot tenir mai valor nul; per tant, no té cap
extrem. És creixent per a tot D (f), ja que f (x ) 0 per a tot
Com que busquem que l’àrea sigui tan petita com es pugui,
D (f).
derivem la funció S (a) i la igualem a 0:
36 9 g) f (x ) x3 12x 6
S (a ) = 4 =0 a=3 b = =3
a2 a x1 = 2
f ( x ) = 3x 2 12 f (x ) = 0 x =
Per tant, a 3 m, b 3 m, c 2 m. x2 = 2

66. a) f (x ) x 2 5x 4 x 3 x 0 x 3

f (x ) 2x 5 f (x ) 0 x 5/2 f (x )

x 0 x 3 f (x )

f (x ) La funció creix en ( , 2) ( 2, + ) i decreix en ( 2, 2).


f (x ) f ( 2) 12 0 x 2 és un màxim.

5 5 f (2) 12 0 x 2 és un mínim.
La funció decreix en , i creix en ,+ .
2 2 h) f (x ) x4 1
f (5/2) 2 0 Hi ha un mínim en x 5/2. f (x ) 4x 3 f (x ) 0 x 0
3x 4
b) f ( x ) = x 1 x 1
2
3 f (x )
f (x ) =
2 f (x )
La funció és creixent per a qualsevol valor de x.
La funció decreix en ( , 0 ) i creix en ( 0, + ).
4
c) f ( x ) = f (0) 0 No podem determinar si hi ha un màxim o un
x
mínim mitjançant la segona derivada.
4
f (x ) = f (x ) 0
x2 i) f ( x ) = e x 3 3x +1

La derivada no pot tenir mai valor nul; per tant, no té cap x1 = 1


extrem. És decreixent, ja que f (x ) 0 per a tot x. f ( x ) = ( 3x 2 3) e x 3 3x +1 f (x ) = 0 x =
x2 = 1
1
d) f ( x ) =
x2 + 1 x 2 x 0 x 2
2x f (x )
f (x ) = 2
( x 2 + 1) f (x )
f (x ) 0 x 0
La funció decreix en ( 1, 1) i creix en ( , 1) (1, + ).
x 1 x 0 x 1 f ( 1) 0 x 1 és un màxim.
f (x ) 0 f (1) 0 x 1 és un mínim.
f (x ) Màx.
j) f (x ) ln (x 2 2x 3)
La funció creix en ( , 0) i decreix en (0, + ) . 2x 2
f (x ) = f (x ) = 0 x =1
f (0) 2 0 Hi ha un màxim en x 0. x2 2x + 3

e) f (x ) x4 5x 1
x 0 x 2
f (x ) 4x 3 5 f (x ) 0 x 1,077
f (x )
x 0 x 2 f (x )
f (x )
La funció decreix en ( ,1) i creix en ( ,1).
f (x )
f (1) 1 0 x 1 és un mínim.

176
BLOC 3. ANÀLISI > UNITAT 12. DERIVADES

x +2 si x 1 Y
b) 7
g(x)
67. a) f ( x ) = 3 si 1 < x 3 6

2x 3 si x > 3 5

4
Derivem la funció: 3
2
1
1 si x 1
f (x ) = 0 si 1 < x 3 – 4 –3 –2 – 1 0
–1
1 2 3 4 X

2 si x > 3
Y
c) h(x)
Per tant, la funció no té cap extrem; és creixent en l’interval 3
( ,1] ( 3, ) i constant en (1, 3]. 2

2 1
( x 1) si x 0
b) f ( x ) =
x si x > 0 –4 –3 –2 –1 0 1 2 3 4 X
–1

Derivem la funció: –2

2x + 2 si x 0 Y
d) i(x)
f (x ) = 1 4
si x > 0
2 x 3

2
Analitzem la funció per parts:
1
— f (x ) 2x 2 si x 0; en aquest interval f (x ) és
sempre positiva, de manera que la funció serà creixent. –4 –3 –2 –1 0 1 2 3 4 X
1 –1
— f (x ) = si x 0, per a qualsevol valor f (x ) serà
2 x
positiva i, per tant, f (x ) serà creixent. e) Y
8

La funció és creixent i no presenta extrems. 6


j(x)
4

2
10x
68. f ( x ) =
0
–8 –6 – 4 –2 2 4 6 8 10 X
x2 + 4 –2

–4
a) Per trobar el màxim, busquem el punt en el qual la deriva- –6
da s’anul·la: –8

10x 2 + 40 x1 = 2
f (x ) = =0 x = Y
x2 + 4)
2 x2 = 2 f)
( 3

2
k(x)
Descartem x 2, ja que no hi ha mesos negatius, i compro- 1
vem que x 2 és un màxim:
–4 –3 –2 –1 0 1 2 3 4 X
f (1) 0 f (3) 0 –1

Per tant, el mes de febrer és el mes d’ingressos màxims. –2

b) La quantitat ingressada aquest mes és:


f (2) 2,5 2 500 euros
70. f (x ) ax 2 bx c
b
x vèrtex =
SÍNTESI Pàgs. 305 i 306 2a

69. a) Y a) f (x ) x2 4
f(x)
4
0
3
x = =0
2·1
2
f (0) 4
1
f (2) 2 0
–4 –3 –2 –1 0 1 2 3 4 X
f (x ) té el vèrtex en (0, 4); és un mínim.

177
BLOC 3. ANÀLISI > UNITAT 12. DERIVADES

b) f (x ) x2 4x En el cas de les paràboles, quan el coeficient que multipli-


ca a x 2 és positiu, la funció té un mínim, i si el coeficient és
( 4) negatiu, té un màxim.
x = =2
2·1
f (2) 4 71. Atès que ens diuen que té el vèrtex en (2, 1), podem deduir
que:
f (2) 2 0
f (2) 0 b 4
f (x ) té el vèrtex en (2, 4); és un mínim.
f (2) 1 c 5
c) f (x ) x2 4x 4
72. Sabem que x y 50 i que la funció que ens proporciona el
x =
( 4) = 2 valor del material és V x 2 y 2; per tant, farem que la funció
2·1 depengui únicament de la variable x:

f (2) 0 V (x ) 2x 2 100x 502


Derivem la funció i igualem la derivada a 0 per trobar el seu
f (2) 2 0
mínim:
f (x ) té el vèrtex en (2, 0); és un mínim. V (x ) 4x 100 0 x 25
d) f (x ) (x 3)2 Comprovem que el valor obtingut és un mínim:
6 V (0) 0 V (50) 0
x = = 3
2·1 Una vegada comprovat que el valor de x és un mínim, substi-
tuïm per obtenir el valor de y:
f ( 3) 0
y 50 x 50 25 25
f ( 3) 2 0.
f (x ) té el vèrtex en ( 3, 0); és un mínim. 73. f (x ) x 2 2x 4
7 4
e) f (x ) 4x 2 16 a) TVM [ 2,3 ]f ( x ) = =3
3 2
0 2
x = =0
b) TVI1f ( x ) = lim
x2 2x + 4 3 ( x 1)
2·4 = lim =0
x 1 x 1 x 1 x 1
f (0) 16 c) f (x ) 2x 2

f (0) 8 0 f (0) 2

f (x ) té el vèrtex en (0, 16); és un mínim. f (0) 4


Per tant, l’equació de la recta tangent és:
f) f (x ) x2 x 6
y 4 2(x 0) y 2x 4
1 1
x = =
2·1 2 d) A partir dels valors obtinguts en l’apartat anterior, l’equació
de la recta normal és:
f ( 1/2) 25/4 (x 0) x
y 4= y = +4
f ( 1/2) 2 0 2 2
f (x ) té el vèrtex en ( 1/2, 25/4); és un mínim. e) f (x ) 2x 2
g) f (x ) 2(x 3)2 8 2x 2 12x 26 f (x ) 2

x =
( 12 ) = 3 f n(x ) 0 per a qualsevol n 2.
2 · ( 2) f) Per trobar els intervals de creixement i decreixement, bus-
quem els punts on s’anul·la la derivada i estudiem com es
f ( 3) 8 comporta al voltant d’ells:
f ( 3) 4 0 f (x ) 0 x 1
f (x ) té el vèrtex en ( 3, 8); és un màxim.
x 0 x 2
h) f (x ) x2 3(x 2) 1 x2 3x 5 f (x )
3 3 f (x )
x = =
2· ( 1) 2
La funció f (x ) creix en (1, + ) i decreix en ( ,1).
f (3/2) 29/4 g) f (1) 2 0 x 1 és un mínim.
f (3/2) 2 0 f (1) 3
f (x ) té el vèrtex en (3/2, 25/4); és un màxim. La funció té un mínim en (1,3).

178
BLOC 3. ANÀLISI > UNITAT 12. DERIVADES

74. f (x ) (x 5)2 g (x ) e x 5
5
4 25 c) TVI1f ( x ) = lim 3x 2
a) TVM [ 0,3 ]f ( x ) = = 7 =
3 0 x 1 x 1
e3 1 15 ( x 1)
TVM [ 0,3 ] g ( x ) = = 6, 36 = lim = 15
3 0 x 1 ( 3x 2 ) ( x 1)
b) TVI1f (x) = f (1) = 2 (1 5) = 8
5 5
TVI1g (x) = g '(1) = e1 = 2, 71 +
d) TVI1f ( x ) = lim 3x 2 2 =
c) L’equació de la recta tangent de f (x ) en x 0 és: x 0 x 0
f (x ) 2(x 5) 15x
f (0) 10 2 ( 3x 2 ) 15
= lim = lim = 3,75
x 0 x x 0 2 ( 3x 2)
f (0) 25
y 25 10(x 0) y 10x 25 e) La recta tangent de f (x ) en x 0 serà determinada per:

L’equació de la recta tangent de g (x ) en x 0 és: 15


f (x ) = 2
g (x ) ex ( 3x 2 )
g (0) 1 15
f (0) =
g (0) 1 4

y 1 1(x 0) y x 1 5
f (0) =
2
d) h (x ) f (x ) g (x ) h (x ) f (x ) g (x ) ex 2(x 5)
5 15 15 5
e) h (x ) f (x )·g (x ) h (x ) 2(x 5)ex ex(x 5)2 y + = ( x 0) y = x
2 2 4 4 2
f (x ) 2 (x 5) ex ex ( x 5) =
f) h ( x ) = h (x ) =
g (x ) e 2x f) A partir dels valors obtinguts en l’apartat anterior, podem
definir que l’equació de la recta normal de f (x ) en x 0 és:
x2 + 12x 35
= 5 4 4 5
ex y + = ( x 0) y = x
2 15 15 2
g) Estudiem el comportament de la funció f (x ):
g) Per estudiar la continuïtat de f (x ) busquem els punts en
f (x ) 0 x 5
els quals el denominador s’anul·la:
x 0 x 10
3x 2 0 x 2/3
f (x )
Per tant, en el punt x 2/3 la funció f (x ) no és contínua ni
f (x ) derivable.

La funció f (x ) creix en ( 5, + ) i decreix en ( , 5 ). 90


h) f ( x ) = 3
Estudiem el comportament de la funció g (x ): ( 3x 2 )
g (x ) 0 i) Podem observar que per a qualsevol valor de x sempre
f (x ) 0; per tant, la funció és decreixent per a tot D (f).
El valor de la derivada és sempre positiu, de manera que la
funció és sempre creixent. 0 0
76. a) TVM[ 0, ]f ( x ) = =0
0
h) Comprovem si l’extrem obtingut de f (x ) és un màxim o un
mínim: 0 0
TVM [ 0, ] g ( x ) = =0
0
f (5) 2 0 x 5 és un mínim.
f (5) 0 b) TVI f (x) = f = 4 cos 2 = 4
2 2 2
La funció f (x ) té un mínim en (5, 0).
La funció g (x ) no presenta cap extrem. TVI g ( x ) = f = 0, 5 (–sin 0) = 0
2 2

5 c) L’equació de la recta tangent de f (x ) en x 0 és:


75. f ( x ) =
3x 2
5 f (x ) 4cos(2x ); f (0) 4; f (0) 0
5
15
a) TVM [1,2 ]f ( x ) = 4 = y 0 4(x 0) y 4x
2 1 4
L’equació de la recta normal de g (x ) en x 0 és:
1 5
+
3 1
b) TVM [ 0,4 ]f ( x ) = 2 2 = g (x ) = sin x
4 0 4 2 2

179
BLOC 3. ANÀLISI > UNITAT 12. DERIVADES

1 3
g (0) = f >0
2 4
1 x 3
g (0) 0 y 0= ( x 0) y = La funció f (x ) té un mínim en x = .
2 2 4
d) L’equació de la recta normal de f (x ) en x 0 és: 3
f <0
1 x 4
y 0= ( x 0) y =
4 4 3
La funció f (x ) té un màxim en x = .
4
L’equació de la recta normal de g (x ) en x 0 és:
1
1 g (x ) = cos x g <0
y 0= ( x 0) y = 2x 2 2 2
1
2 La funció g (x ) té un màxim en x = .
2
e) h ( x ) = f ( x ) + g ( x )
1 g >0
h ( x ) = 4 cos ( 2x ) sin x 2
2 2
f) h ( x ) = f ( x ) · g ( x ) La funció g (x ) té un mínim en x = .
2
h ( x ) = 2 cos ( 2x ) cos x f (x )
2 i) h ( x ) =
g (x )
sin ( 2x ) sin x
2 4 2 cos ( 2x ) cos x + sin ( 2x ) sin x
2 2
h (x ) =
g) Per trobar els intervals de creixement i decreixement, bus-
quem els punts en els quals s’anul·la la derivada i estudi- cos2 x
2
em com es comporta aquesta al voltant d’ells:
g (x )
3 3 j) h ( x ) =
f (x ) 0 x1 = ; x2 = ; x3 = ; x4 = f (x )
4 4 4 4
3 2 sin x sin ( 2x ) 2 cos x cos ( 2x )
x = x = x = x = x = h (x ) = 2 2
5 5 4
4sin2 ( 2x )
f (x )
f (x ) 77. Estudiant el valor de la derivada segona d’una funció donada,
al voltant dels punts d’inflexió, podem definir-ne la concavitat
3 3
La funció f (x ) creix en , , , o la convexitat.
4 4 4 4
a) f (x ) x3
3 3 3 3
, , i decreix en , , .
4 4 4 4 4 4 4 4 f (x ) 0 6x 0 x 0
f (0) 0
g (x ) 0 2 El punt d’inflexió de f (x ) es (0, 0).
x =
b) f (x ) sin x cos x
2
3
x1 =
x = 2 x = x =2 4
3 3 3 f ( x ) = sin x cos x = 0 x =
7
f (x ) x2 =
4
f (x )
3 7
f =0 f =0
La funció g (x ) creix en , i decreix en 4 4
2 2
, , . 3 7
2 2 Els punts d’inflexió de f (x ) són ,0 i ,0 .
4 4
c) f (x ) x· ex
h) f (x ) 8 sin(2x ) f <0
4 f (x ) e x(x 2) 0 x 2
La funció f (x ) té un màxim en x = . f ( 2) 2e 2
4
El punt d’inflexió de f (x ) és ( 2, 2e 2).
f >0
4
78. Tenim que la derivada de la funció en x a és 0.
La funció f (x ) té un mínim en x = .
4 f (a) 0.

180
BLOC 3. ANÀLISI > UNITAT 12. DERIVADES

Sabem que la segona derivada és la derivada de la derivada; Per tant, l’equació de la recta tangent en x 1 de f (x ) és:
per tant:
y 2 4(x 1) y 4x 2
— Si f (a) 0 significa que f (x ) és creixent en x a.
d) f (x ) ln x en x 1
f (a h) f (a) f (a h).
1
Com que f (a) és 0, per a valors una mica més petits que f (x ) = f (1) 0 f (1) 1
a, la funció és decreixent (la seva derivada és negativa), i x
per a valors una mica més grans que a és creixent (la seva Per tant, l’equació de la recta tangent en x 1 de f (x ) és:
derivada és positiva); per tant, es tracta d’un mínim.
y 0 1(x 1) y x 1
— Si f (a) 0 significa que f (x ) és decreixent en x a.
e) f (x ) cos x en x =
f (a h) f (a) f (a h).
f (x ) sin x f( )= 1 f ( )=0
Com que f (a) és 0, per a valors una mica més petits que
a, la funció és creixent (la seva derivada és positiva), i per Per tant, l’equació de la recta tangent en x = de f (x ) és:
a valors una mica més grans que a és decreixent (la seva
derivada és negativa); per tant, es tracta d’un màxim. y 1 0 y 1

4. a) f (x ) 3x 2 sin x
Avaluació (pàg. 308 )
f (x ) 6x cos x
1. a) f (x ) 4x 2 2x 18
b) f (x ) ln x · (x 2 2x 1)
74 20
TVM [1,4 ]f ( x ) = = 18 x 2 + 2x + 1
4 1 f (x ) = + ( 2x + 2 ) ln x
x
b) f ( x ) = x2 + 1 x cos x
c) f ( x ) =
17 2 x +1
TVM [1,4 ]f ( x ) = = 0,9 x cos x
4 1 ( cos x x sin x ) x + 1
f (x ) = 2 x +1
c) f (x ) 4ln x
x +1
5,55 0 cos x
TVM [1,4 ]f ( x ) = = 1,85 d) f ( x ) =
4 1 x +1
12 cos x
d) f ( x ) = sin x x + 1
5x + 4 f (x) = 2 x +1
1 4 x +1

TVM [1,4 ]f ( x ) = 2 3 = 0,28


x +1 si x 1
4 1
5. f ( x ) = 0,5x 2 + 1,5 si 1 < x 5
2. x (t ) 5t 2 2t 2 2x 1 si x > 5
a) v (t ) x (t ) 10t 2 v (0) 2 m/s Els punts que cal estudiar de la funció són x 1 i x  5. Co-
b) v (3) 32 m/s mencem estudiant la continuïtat en el punt x 1:

c) a (t ) v (t ) 10 a (0) 10 m/s2 lim f ( x ) = lim x + 1 = 2


x 1 x 1

d) a (10) 10 m/s2 lim+ f ( x ) = lim+ 0,5x 2 + 1,5 = 2


x 1 x 1

2
3. a) f ( x ) = en x 1 Els valors dels límits laterals coincideixen; per tant, f (x ) és
x contínua en x 1.
2
f (x ) = f (1) 2 f (1) 2 1 si x 1
x2
f (x ) = x si 1 < x 5
Per tant, l’equació de la recta tangent en x 1 de f (x ) és:
2 si x > 5
y 2 2(x 1) y 2x 4
Comprovem com es comporta la derivada al voltant de x 1
b) f (x ) x2 2x 3 en x 3
f (1 ) 1 f (1 ) 1
f (x ) 2x 2 f (3) 6 f (3) 4 Com que els valors coincideixen, la funció és derivable en
x 1.
Per tant, l’equació de la recta tangent en x 3 de f (x ) és:
Estudiem la continuïtat en el punt x 5:
y 6 4(x 1) y 4x 10
lim f ( x ) = lim 0,5x 2 + 1,5 = 14
x 5 x 5
c) f (x ) x2 2x 3 en x 1
lim+ f ( x ) = lim+ 2x 1= 9
x 5 x 5
f (x ) 2x 2 f ( 1) 2 f ( 1) 4

181
BLOC 3. ANÀLISI > UNITAT 12. DERIVADES

Els valors dels límits laterals no coincideixen; per tant, f (x ) no b) f ( x ) = sin ( x )


és contínua en x 5. Al no ser contínua tampoc és derivable
en x 5. 1 ± 2k .
f (x ) = cos ( x ) f (x ) = 0 x =
2
6. a) f ( x ) = cos ( x ) Els valors d’aquests extrems són 1 o 1 alternativament.
f (x ) = sin ( x ) ; f ( x ) = 0 x =k 1 ± 4k
La funció té extrems infinits en ,1 i en
2
f (x ) és creixent en (2k 1, 2k 2) i decreixent en 1 ± 4k
(2k, 2k 1). , 1 .
2
b) f (x ) x3 1
1
c) f ( x ) =
f (x ) 3x 2 f (x ) 0 x 0 x2 + x + 1
x 1 x 1
f (x ) =
( 2x + 1) f (x ) = 0 x =
1
2
f (x ) ( x 2 + x + 1) 2
f (x )
f ( 1/2) 4/3
La funció és creixent en ( , 0) ( 0, + ) . La funció té un extrem en ( 1/2, 4/3).
c) f (x ) x2 3x 2 d) f (x ) x · ln x
f (x ) 2x 3 f (x ) 0 x 3/2 f (x ) ln x 1 f (x ) 0 x 1/e

x 1 x 2 f (1/e) 1/e
f (x ) La funció té un extrem en (1/e, 1/e).
f (x )
8. a) Es positiva en , 0)
( 3, ) i negativa en (0, 3).
(
3 3 b) La derivada és positiva en ( , ).
La funció creix en ,+ i decreix en , .
2 2
1 9. Sabem que x y 10, i que la condició és que
d) f ( x ) =
x2 +1 f (x,y) x2 6y f (x ) x2 6x 60
2x Per buscar el mínim, la derivada ha de ser nul·la:
f (x ) = 2
f (x ) = 0 x =0
( x 2 + 1) f (x ) 2x 6 0 x 3 y 7
Ens assegurem que el valor de x 3 sigui un mínim.
x 1 x 2
f (3) 2 0 x 3 és un mínim.
f (x )
f (x ) 10. El punt de la recta més proper al punt (4,1), el punt d’encre-
uament entre la recta y 3x 2 i una recta perpendicular a
La funció decreix en ( 0, + ) i creix en ( , 0). aquesta que passi pel punt (4, 1). Sabem que el pendent de
e) f (x ) ex 1
la nova recta ha de ser m = i sabem que la fórmula de la
3
f (x ) ex
x 7
recta és y y p = m ( x x 0 ) y = + .
f (x ) 0 per a qualsevol valor de x i, per tant, f (x ) és sem- 3 3
pre creixent. Per tant, hem de buscar el punt d’encreuament de totes dues
rectes:
f) f (x ) ln x x
1 y = 3x 2
f (x ) = 1 f (x ) = 0 x =1 13 19
x 7 x = ;y =
x y = + 10 10
3 3
x 0,5 x 2 13 19
Així doncs, el punt és , .
f (x ) 10 10
f (x )
Zona (pàg. 309)
La funció decreix en (1, + ) i creix en ( 0,1). — La marató
7. a) f (x ) x2 3x 2 t Resposta suggerida:
f (x ) 2x 3 f (x ) 0 x 3/2 L’objectiu de l’enllaç proposat és veure l’aplicació de les deri-
vades al càlcul de la variació de la velocitat en cada instant de
f (3/2) 1/4
temps d’un corredor de marató en cursa, i així saber els can-
f (x ) té un extrem en (3/2, 1/4). vis que es produeixen en el seu rendiment.

182
BLOC 3. ANÀLISI > UNITAT 12. DERIVADES

t Resposta suggerida: x 30 x 38,73 x 40


Mitjançant aquests altres enllaços, els alumnes veuran altres c (x ) 0
aplicacions de la derivada en el camp dels esports, relaciona-
c (x ) Mín.
des també en aquest cas amb la velocitat.
— Economia i derivades Per tant, el nivell de producció amb un cost mínim és de 39
unitats. Per a aquest nivell de producció, el cost-mitjana c (x )
t En primer lloc, determinem la funció cost-mitjana (cost per és de 17,49 euros i el cost total C (x ) de 677,46 euros.
unitat produïda):
t Aquesta informació és útil perquè l’empresari sàpiga el nom-
C(x) 1 x2 x 300 bre d’unitats amb el qual s’obté un cost mínim, de manera
c(x) = = + 2x + 300 = +2+ que pugui optimitzar els seus beneficis.
x x 5 5 x
C(x) 1 x2 x 300 — El pont matemàtic
c(x) = = + 2x + 300 = +2+
x x 5 5 x t Com s’explica en la pàgina web proposada:
A primera vista, el pont fa honor al nom, ja que en la seva
Funció que derivem per a determinar el mínim:
estructura s’observen diverses tangents a la corba sobre el
riu, i ofereix una bona plasmació visual de com, d’esquerra a
1 300
c (x ) = dreta, els pendents disminueixen: quan el pont puja (deriva-
5 x2 des positives) i quan baixa (derivades negatives), en què zero
és en el punt més alt (màxim).
1 300 1 300
c (x ) = 0 1 =3000 = 1 300
c ( x ) = 50 x2 = 50 x2 = t Les dues principals contradiccions sobre el pont tenen a veu-
5 x2 5 x2 re amb la impossibilitat que el construís Newton, i que els
x = 1500
2 x = 38,73 39
x = 1500
2 x = 38,73 39 estudiants el poguessin desmuntar i tornar a muntar.

183
BLOC 4. ESTADÍSTICA I PROBABILITAT

13# Estadística bidimensional

En context (pàg. 315) Observem que les rectes resultants són paral·leles als ei-
xos de coordenades i, en conseqüència, perpendiculars
Resposta oberta a manera de reflexió, que pot servir d’intro- entre elles.
ducció a l’estadística bidimensional. Les preguntes es poden
contestar observant el gràfic i analitzant la notícia que s’expli- — Veiem que, si r ±1 les rectes de correlació són coinci-
ca. dents, mentre que, si r 0 les rectes de correlació són
perpendiculars entre si. Podem observar que, a mesura
que augmenti el valor de r (o disminueixi entre 0 i 1) les
dues rectes de regressió s’aniran apropant.
Llenguatge matemàtic (pàg. 321)
— El coeficient de correlació de Pearson no ens ha de do-
nar informació sobre la mida de les nostres variables, Calculadora (pàg. 323)
sinó sobre el grau en què s’assemblen, per això és im-
— Resposta suggerida per a una calculadora del tipus Casio
portant que no siguin proporcionals a les variables ni
fx-82ES. En el vídeo següent s’explica com es fa servir
tinguin unitats.
aquesta calculadora per a resoldre problemes d’estadísti-
ca bidimensional:

Amplía (pàg. 322) http://links.edebe.com/gbxruc

— Si les variables tenen dependència funcional, r ±1.

XY
Problemes resolts (pàgs. 324 a 326)
r = = ±1 XY = ±( X Y )
X Y 1. Calculem les marginals de X i de Y, en què Y edat i
Ara, a partir de les rectes de regressió tenim: X temps. En aquestes taules, calcularem també les marques
de classe.
XY
y Y = (x X ) Marginal de X:
2
X

XY 0,5 1,5 2,5 3,5 4,5


x X = (y Y )
2
Y X 0-1 1-2 2-3 3-4 4-5
±( X Y ) fi 31 30 22 11 6
y Y =
2
(x X )
X
Marginal de Y:
±( X Y )
x X = (y Y )
2
Y 7,5 17,5 22,5 27,5 32,5 37,5 57,5
± Y Y 0-15 15-20 20-25 25-30 30-35 35-40 40-75
y Y = (x X )
X fi 15 14 14 14 14 14 15
± X
x X = (y Y ) Ara, calculem la covariància:
Y
n
(y Y =±
) (x X) 1
X Y X = x i fi =
N i =1
Y (x X) = ± X (y Y)
=
( 0, 5 31 + 1, 5 30 + 2, 5 22 + 3, 5 11 + 4, 5 6 ) =
Observem que les dues rectes coincideixen. 100
— Si les variables són independents, r 0: 181
= = 1,81
100
XY
r = =0 XY =0 m
X Y 1
Y = y j fj =
M
Ara, a partir de les rectes de regressió tenim: j =1

7,5 15 + 17,5 14 + 22,5 14 + 27,5 14 +


XY =
y Y = ( x X ) 100
2
X
y Y =0 (x X)
+ 32,5 14 + 37,5 14 + 57,5 15 2 900
XY x X =0 (y Y) = = 29
x X = (y Y ) 100 100
2
Y
n m
y =Y 1
XY = x i y j fij X Y =
x =X N i j

184
BLOC 4. ESTADÍSTICA I PROBABILITAT > UNITAT 13. ESTADÍSTICA BIDIMENSIONAL

1 c) Sigui y 5; introduïm aquesta dada en l’equació resultant


= ( 0, 5 7, 5 7 + 0, 5 17, 5 5 + ... + 4, 5 57, 5 0 ) de l’apartat b).
100
1,81 29 = 0,115 x 1,24 0,14 · 5 1,94

La covariància és negativa i de valor petit. Podem concloure Per tant, si una persona fa 5 anys que treballa a l’empresa
que els grups amb més edat dediquen menys temps a l’ordi- el seu salari serà de 1 940 €.
nador.
Exercicis i problemes (pàgs. 327 a 332)
2. a) Les rectes de regressió es tallen en el centre de gravetat
d’una distribució, és a dir, en el punt X ,Y . Plantegem el ( )
sistema següent i el resolem: 1 VARIABLE ESTADÍSTICA
y = 1,5x 1
BIDIMENSIONAL Pàg. 327
x = 0,36; y = 0,45
x = 0,3y + 0,5 4. a) El pes i l’alçada d’una persona són variables quantitatives i
contínues.
Per tant, la solució del sistema anterior coincideix amb les
mitjanes de X i de Y. És a dir: X = 0,36; Y = 0,45. b) L’hora del dia i la temperatura en aquesta hora són varia-
bles quantitatives i contínues.
b) Suposem que la recta r és la regressió de Y sobre X ja que
tenim aïllada la y. D’altra banda, suposem que s és la re- c) Les variables estadístiques província i color de cotxe més
gressió de X sobre Y ja que en aquesta equació està aïllada venut són variables qualitatives.
la x. Si això és cert, tindrem que: d) El nombre d’habitants i de televisors en una casa són vari-
XY XY
ables quantitatives i discretes.
= 1, 5; = 0,3
2 2
X Y
5. a) La taula de doble entrada és la següent:
El coeficient de Pearson és:
Y 0 1 2 3 4 5
2
X
XY XY XY XY
r = r2 = r = 0 0 0 2 2 1 1
2 2 2 2
X Y X Y X Y
1 0 1 1 3 0 1
Substituïm les dades que hem obtingut: 2 2 1 1 2 1 1

XY XY
3 0 2 1 0 0 0
r = = 1,5 0,3 = 0,67 < 1
2 2 4 0 3 1 1 0 0
X Y

5 1 3 1 0 2 1
Com que aquest valor és més petit que 1, aleshores el que
havíem suposat és cert. És a dir, la recta r és la regressió b) Y
de Y sobre X i la recta s és la regressió de X sobre Y.
6
c) El coeficient de correlació és el valor que hem calculat en 5

l’apartat b), així que, r 0,67. Com que aquest valor és 4

més proper a 1 que a 0, podem concloure que la regressió 3

és bona. 2
1

3. a) Coeficient de correlació: O 1 2 3 4 5 6 X

1 10,5 1 22
X = Xi = = 1,75; Y = Yi = = 3,6 6. a) Calculem les marques de classe:
N i 6 M i 6
1 19,75 0 + 10 10 + 20
2 = X i2 X =
2
1, 752 = 0,22916 x1 = = 5; x 2 = = 15;
X
N 6 2 2
i
20 + 30
1 2 138 2 x3 = = 25
2
Y = Yi 2 Y = 3,6 = 9,5 2
N i 6 0+3 3+6
n y1 = = 1,5; y 2 = = 4,5;
1 46,5 2 2
XY = X i Yi X Y = 1,75 3,6 = 1,3
N i 6 6+9
y3 = = 7,5
XY 1,3 2
r = = = 0,9 b) X
2 2
X Y 0,22916 9,5
8

b) Recta de regressió de X sobre Y: 6

XY 1,3 4
x X =
2
(y Y ) x 1,75 =
9,5
(y 3,6 )
Y 2

x 1,75 = 0,14 (y 3,6 ) x = 1,24 + 0,14y


O 5 15 25
Marques de classe

185
BLOC 4. ESTADÍSTICA I PROBABILITAT > UNITAT 13. ESTADÍSTICA BIDIMENSIONAL

Y 1
8
= (5 12 + 7 32 + 8 52 + 11 72 + 3 92 ) 52 =
34
6 = 2,42
4
8. a) La taula de doble entrada corresponent és:
2
Figura 3
Y 2 3 4 5 6 7 8 9 10
O 1,5 4,5 7,5 X
Marques de classe
3 4 0 0 0 0 0 0 0 0
c) Y 4 0 0 0 6 0 0 0 0 0
10 5 0 0 0 12 0 0 0 0 0
7,5 6 0 0 0 0 4 5 0 0 0
5,0 7 0 0 0 0 4 2 0 0 0
2,5 8 0 0 0 0 0 0 0 1 0
9 0 0 0 0 0 0 0 0 0
O 7,5 15 22,5 30 X
Figura 4
10 0Figura05 0 0 0 0 0 0 2
7. a) Y
b) Y
10
10
8
8
6
6
4
4
2
2

O 2 4 6 8 10 X
O 2 4 6 8 10 X

b) Marginal de X: c) A partir del diagrama de dispersió, observem que les varia-


bles són de dependència lineal positiva forta.
X 0 2 4 6 8
fi 2 7 9 11 5

Marginal de Y: 2 RELACIÓ ENTRE VARIABLES


ESTADÍSTIQUES: CORRELACIÓ Pàgs. 327 i 328
Y 1 3 5 7 9
fj 5 7 8 11 3 9. a) El pes i l’altura són variables amb dependència estadísti-
ca.
c) Mitjana de X:
b) El radi i la longitud d’una circumferència són variables
n
1 amb dependència funcional.
X = x i fi =
N i =1 c) La velocitat d’un cotxe i la despesa de la gasolina són vari-
1 ables amb dependència estadística.
= ( 0 2 + 2 7 + 4 9 + 6 11 + 8 5 ) = 4,588
34 d) La quantitat de pluja i la renda anual són variables inde-
Mitjana de Y: pendents, ja que no tenen res en comú.
m
1 10. Efectuem els càlculs en la taula que necessitem per a les
Y = y j fj =
M j =1 qüestions següents:
1
= (1 5 + 3 7 + 5 8 + 7 11 + 9 3 ) = 5 Y 1 1,5 2 3 fi Xi · fi Xi 2 · fi
34 X
12 12 3 1 0 16 192 2 304
d) Desviació típica de X:
15 6 4 4 1 15 225 3 375
1 2
X = fi x i2 X = 20 2 2 4 4 12 240 4 800
N i
25 0 1 1 5 7 175 4 375
1
= ( 2 02 + 7 22 + 9 42 + 11 62 + 5 82 ) 4,5882 = fi 20 10 10 10 50 832 14 854
34
Yj·fj 20 15 20 30 85
= 2,25
Yj2·fj 20 22,5 40 90 172,5
Desviació típica de Y:
1 2 1 n
832
Y = f j y 2j Y = a) Mitjana de X: X = x i fi = = 16,64
M j 50 50
i =1

186
BLOC 4. ESTADÍSTICA I PROBABILITAT > UNITAT 13. ESTADÍSTICA BIDIMENSIONAL

1 m
85 b) Per a calcular la covariància, necessitem uns càlculs pre-
Mitjana de Y: Y = y j fj = = 1,7 vis:
50 j =1 50
b) Variància de X: X Y X·Y

1 2 14 854 0 1 0
X
2 = fi x i2 X = 16,642 = 20,19
N i 50 1 2 2

Variància de Y: 2 2 4
3 3 9
1 2 172,5
Y
2 = f j y 2j Y = 1,72 = 0,56
M 50 4 3 12
j
5 3 15
c) La covariància entre les dues variables és la següent:
6 1 6
n m
1
XY = x i y j fij X Y = 7 2 14
N i j
8 4 32
1
= (12 1 12 + 12 1, 5 3 + ... + 25 3 5 ) 9 4 36
50
Sumes 45 25 130
16, 64 1,7 = 2,302
45 25
11. Busquem les dades necessaries per a trobar el coeficient de X = = 4,5; Y = = 2,5
10 10
correlació. n
1 130
XY = X i Yi X Y = 4,5 2,5 = 1,75
X Y X2 Y2 X·Y N i 10
0 1 0 1 0
0 2 0 4 0 13. a) Marginal de X:
1 2 1 4 2
X 0 1 2 3 4 5 6
1 3 1 9 3
fi 58 64 41 29 22 11 5
1 3 1 9 3
Marginal de Y:
2 3 4 9 6
2 1 4 1 2 Y 0 1 2 3 4 5 6
2 2 4 4 4 fj 58 66 57 24 12 9 4
3 4 9 16 12
b) Calculem el coeficient de correlació:
3 4 9 16 12
n
15 25 33 73 44 1 406
X = x i fi = = 1,77;
N i =1 230
15 25 m
X = = 1, 5; Y = = 2,5 1 369
10 10 Y = y j fj = = 1,6
M 230
1 2 33 j =1
2
X = X i2 X = 1,52 = 1,05 1 2 91
N i 10 2 = X i2 X = 1,772 = 2,74
X
1 2 73 N i 230
2
Y = Yi 2 Y = 2,52 = 1,05 1 2 91
N i 10 2 = Yi 2 Y = 1,62 = 2,16
Y
n M i 230
1 44
XY = X i Yi X Y = 1,5 2,5 = 0,65 1 n m
971
N i 10 XY = x i y j fij X Y = 1,77 1,6 = 1,4
0,65 N i j 230
XY
r = = = 0,62
2 2 1,05 1,05 XY 1,4
X Y r = = = 0,6
2 2
X Y ( 2,74 ) ( 2,16 )
— El valor del coeficient de correlació s’aproxima més a 1
que a 0; per tant, les variables tenen una forta dependèn-
c) La relació que hi ha entre aquestes dues variables és de
cia lineal positiva.
Figura 6
dependència lineal forta i positiva, ja que el valor del coefi-
cient de correlació s’aproxima més a 1 que a 0.
12. a) Y

4
14. Calculem la covariància entre les variables:
3
n
1 67,63
2 X = xi = = 8,453 75
N i =1 8
1 m
1 73, 2
Y = yj = = 9,15
O 2 4 6 8 10 X M j =1 8
n
1
XY = X i Yi X Y =
N i
12 077,19 187
= 8,453 75 ( 9,15 ) = 1 432,29
8
n
1 67,63
X = xi = = 8,453 75
N 8 BLOC 4. ESTADÍSTICA I PROBABILITAT > UNITAT 13. ESTADÍSTICA BIDIMENSIONAL
i =1
m
1 73, 2
Y = yj = = 9,15
M j =1 8
1 n 18. La taula de doble entrada és la següent:
XY = X i Yi X Y =
N i
Y 5 6 7 8 9 fi Xi · fi
12 077,19 X
= 8,453 75 ( 9,15 ) = 1 432,29
8 5 0 1 5 0 3 9 45

15. a) Construïm la taula de doble entrada: 6 1 2 2 3 0 8 48

Y 7 2 1 2 1 0 6 42
X 0 1 2
8 1 2 2 1 0 6 48
0 7 6 0

1 7 5 1 9 1 3 2 1 0 7 63

2 0 2 1 fi 5 9 13 6 3 36 246

3 0 0 1 Yi·fi 25 54 91 48 27 245

b) Marginal de la variable germans, X: Marginal de X:

X 0 1 2 3
X 5 6 7 8 9
fi 13 13 3 1 fi 9 8 6 6 7

Marginal de la variable germanes, Y: Marginal de Y:

Y 0 1 2
Y 5 6 7 8 9
fj 14 13 3 fi 5 9 13 6 3

c) Trobem el valor de la covariància entre les variables: Covariància entre les variables:
n
1 1 246 245
X = x i fi = ( 0 13 + 1 13 + 2 3 + 3 1) = X = = 6,83; Y = = 6,81
N i =1 30 36 36
n m
22 1
= = 0,73 XY = x i y j fij X Y =
30 N i j
m
1 1 19 1
Y = y j fj = ( 0 14 + 1 13 + 2 3 ) = = 0,63 = ( 5 5 0 + 5 6 1 + ... + 9 9 0 ) 6,83 6,81 = 0,59
M j =1 30 30 36
n m
1 19. a) Si observem el gràfic, veiem que les variables tenen depen-
XY = x i y j fij X Y =
N i j dència lineal positiva feble, de manera que el coeficient de
1 correlació és més gran que zero però no gaire proper a 1.
= ( 0 0 7 + 0 1 6 + ... + 3 2 1) 0, 73 0,63 = 0,24
30 b) En aquest cas, les variables tenen dependència lineal ne-
gativa; per tant, el coeficient de correlació és negatiu i in-
Figura 7 termedi.
16. El núvol de punts corresponent és el següent:
c) En aquest últim gràfic, la dependència entre les variables
Y
és lineal negativa i forta. Així que el coeficient de correlació
10
és proper a 1.
7,5
20. Calculem les dades necessàries per a trobar el coeficient de
5,0
correlació:
2,5
Y 1 2 3 4 5 fj Xi · fi X i2 · fi
X
O 14 16 18 20 X 1 0 2 5 7 11 25 25 25

Ara, calculem la covariància entre les dues variables: 2 5 1 0 2 3 11 22 44

n m 3 2 4 1 8 0 15 45 135
1 155 1 55
X = xi = = 15,5; Y = yj = = 5,5
N 10 N 10 4 3 0 2 0 1 6 24 96
i =1 j =1

1 n
870 5 1 5 6 2 7 21 105 525
XY = X i Yi X Y = 15,5 5,5 = 1,75
N i 10 fi 11 12 14 19 22 78 221 825

Yi·fi 11 24 42 76 110 263


17. Utilitzant les funcions estadístiques de la calculadora, resulten Yi2·fi 11 48 126 304 550 1 039
les mateixes solucions que en l’exercici anterior.

188
BLOC 4. ESTADÍSTICA I PROBABILITAT > UNITAT 13. ESTADÍSTICA BIDIMENSIONAL

221 263 c) Calculem el coeficient de correlació:


X = = 2,83; Y = = 3,37179
78 78 m
1 2 8,83
1 2 825 2
2
Y = y j2 Y = 1,222 = 0,277 6
X
2 = fi x i2 X = 2,83 = 2,549 M j =1 5
N i 78
XY 1,4
1 2 1039 r = = = 0,94
Y
2 = f j y 2j Y = 3,371792 = 1,951 2 2 8 0,277 6
M j 78 X Y

n m
1 d) El coeficient de determinació és el següent:
XY = x i y j fij X Y =
N i j 2
XY 1,96
1 r2 = = = 0,88
= (1 1 0 + 1 2 2 + ... + 5 5 7 ) 2,83 2
X
2
Y 8 0,277 6
78 Figura 8
3,37179 = 0,4
XY 0,4 23. a) Y
r = = = 0,18
2
X
2
Y 2,549 1,951 10

8
— El valor del coeficient de correlació és negatiu i baix, de 6
manera que la dependència és feble i de signe negatiu 4
(quan una de les variables augmenta, l’altra disminu-
2
eix).
O 5 10 15 20 X

b) Recta de regressió de X sobre Y:


3 REGRESSIÓ LINEAL Pàgs. 329 a 331
n m
1 57 1 33
X = xi = = 11, 4; Y = yj = = 6,6
21. a) Les rectes de regressió són decreixents; per tant, el signe N 5 M 5
i =1 j =1
de la correlació és negatiu. D’altra banda, no es tracta de
n
variables independents ni fortament correlacionades, sinó 1 297
XY = X i Yi X Y = 11,4 6,6 = 15,84
d’un cas intermedi. N i 5
b) Aquestes dues rectes no representen rectes de regressió, m
1 2 255
ja que són perpendiculars entre elles, però no són paral-
2
Y = y j2 Y = 6,62 = 7,44
M j =1 5
leles als eixos de coordenades.
XY 15,84
c) Les rectes de regressió són perpendiculars entre elles i x X = (y Y ) x 11, 4 = ( y 6,6 )
2
Y 7,44
paral·leles als eixos de coordenades, de manera que les
variables són independents. x 11,4 = 2,13 ( y 6,6 )
d) Les rectes de regressió són bastant a prop; per tant, el co- x = 25,45 2,13y
eficient de correlació és proper a 1, la qual cosa implica
que la dependència entre les variables és forta. Com que c) Recta de regressió de Y sobre X:
les dues rectes són creixents, el signe de la correlació és n m
1 57 1 33
positiu. X = xi = = 11,4; Y = yj = = 6,6
N i =1 5 M j =1 5
22. a) La fórmula de la recta de regressió de Y sobre X és 1 n
297
XY = X i Yi X Y = 11,4 6,6 = 15,84
y Y = XY
(x X . Calculem les dades que necessi-
) N i 5
2
X m
1 2 821
tem per a aquesta recta: 2
X = xi 2 X = 11,42 = 34,24
M i =1 5
n m
1 60 1 6,1
X = xi = = 12; Y = yj = = 1,22 y Y = XY
(x X )
N i =1 5 M j =1 5 2
X
n
1 80,2 15,84
XY = X i Yi X Y = 12 1,22 = 1,4 y 6,6 = ( x 11,4 )
N i 5 34,24
1 m
2 760 y 6,6 = 0,46 ( x 11,4 ) y = 11,87 0,46x
2 = xi 2 X = 122 = 8 Figura 9
X
M i =1 5
1,4 24. a) Y
y 1,22 = ( x 12 ) y 1,22 = 0,175 ( x 12 )
8 1,0

y = 0,88 + 0,175x 0,5

b) El coeficient de regressió és el pendent de la recta anterior; O


2,5 5,0 7,5 10 X
XY –0,5
és a dir, b = = 0,175.
2
X –1,0

189
BLOC 4. ESTADÍSTICA I PROBABILITAT > UNITAT 13. ESTADÍSTICA BIDIMENSIONAL

b) Recta de regressió de Y sobre X: n


1 7 070
XY = X i Yi X Y = 16,3 76,16 = 65,72
1 n
30 1 m
0 N i 6
X = xi = = 6; Y = yj = =0
N i =1 5 M j =1 5 XY 65,72
r = = = 0,917
n 2 2
1 8,4 X Y 172,2 29,805
XY = X i Yi X Y = 6 0 = 1,68
N i 5
b) Busquem la recta de regressió de l’edat en funció dels
m
1 2 220 anys fumant:
2
X = xi 2 X = 62 =8
M i =1 5
XY
1,68 y Y = (x X )
XY 2
y Y = ( x X ) y 0= ( x 6) X
2
X 8
65,72
y = 0, 21 (x 6) y = 1,26 0,21x y 76,16 =
172,2
(x 16,3 )
c) Recta de regressió de X sobre Y: y 76,16 = 0,38 (x 16,3 ) y = 82,4 0,38x
n m
1 30 1 0
X = xi = = 6; Y = yj = =0 27. a) Calculem el coeficient de correlació:
N i =1 5 M j =1 5
n 1 164 1 149
1 8,4 X = Xi = = 16,4; Y = Yi = = 14,9
XY = X i Yi X Y = 6 0 = 1,68 N i 10 N i 10
N i 5
1 2 2 848
1 m
2 2,06
2
X = X i2 X = 16,42 = 15,84
2
Y = yj 2 Y = 02 = 0,412 N i 10
M 5
j =1 1 2 2 265
1,68
2
Y = Yi 2 Y = 14,92 = 4,49
x X = XY
(y Y ) x 6= (y 0) N i 10
2
Y 0,412 n
1 2 364
x 6 = 2,13y x =6 4,078y XY = X i Yi X Y = 16,4 14,9 = 7,96
N i 10
XY 7,96
25. a) Si observem el gràfic, veiem que les variables tenen una r = = = 0,94
dependència lineal positiva forta, la qual cosa implica que
2
X
2
Y 15,84 4,49
les rectes de regressió són molt a prop entre elles.
Com que aquest valor és proper a 1, podem concloure
b) Les variables d’aquest gràfic tenen una dependència lineal que les variables tenen una forta dependència lineal.
negativa forta, la qual cosa implica que les rectes de re-
b) Trobem la recta de regressió de Y sobre X:
gressió són molt properes entre si.
XY
c) Aquestes dues variables estan relacionades entre elles en y Y = (x X )
2
forma de corba. Aquesta corba és clarament decreixent; si X

calculéssim l’índex de correlació, obtindríem un valor ne- 7, 96


y 14, 9 = ( x 16, 4 )
gatiu i no gaire proper a 1, és a dir, tindríem una relació 15, 84
negativa feble. Les rectes de regressió estarien allunyades. y 14, 9 = 0, 5 ( x 16, 4 ) y = 23,1 0, 5x
d) Aquí veiem que les variables són independents; per tant,
les rectes de regressió són perpendiculars i paral·leles als 28. Per a saber quina d’aquestes distribucions té una relació més
eixos de coordenades. forta, hem de calcular els coeficients de correlació de cada
cas.
e) Aquestes dues variables estan relacionades entre elles en
forma de corba. Aquesta corba és clarament creixent; si XY 0,8
a) ra = = = 0,946
calculéssim l’índex de correlació, obtindríem un valor ne- X Y 0,95 0,89
gatiu i proper a 1; és a dir, tindríem una relació positiva
forta. Les rectes de regressió serien a prop. XY XY
b) y = 0,81x + 0,5 = 0,81 2
x =
f) Les variables tenen una relació de dependència funcional i
2
x 0,81
a més és lineal; les rectes de regressió coincideixen.
XY XY
x = 1,12y 0,7 = 1,12 2
Y =
2 1,12
26. a) Trobem el coeficient de correlació, en què X anys i Y

Y edat. rb = XY
= XY
=
2 2
X Y XY XY
1 98 1 457
X = Xi = = 16, 3; Y = Yi = = 76,16 0,81 1,12
N i 6 M i 6
XY
1 2 2 634 2
= = 0,952
2
X = X i2 X = 16,3 = 172,2 XY
N i 6 0,952
1 2 34 987 2
2
Y = Yi 2 Y = 76,16 = 29,805 La relació entre les variables serà més forta quan el valor
N i 6 del coeficient de correlació sigui més a prop d’1; en aquest

190
BLOC 4. ESTADÍSTICA I PROBABILITAT > UNITAT 13. ESTADÍSTICA BIDIMENSIONAL

n m
1 120 1 10,5
X = xi = = 20; Y = yj = = 1,75
N i =1 6 M j =1 6
cas, el coeficient de correlació de l’apartat b) és més gran 1 n
225
que el de l’apartat a); per tant, la relació entre les variables XY = X i Yi X Y = 20 1,75 = 2,5
Figura N 6
és més forta en b). 10 i
m
1 2 2 650
2 = xi 2 X = 202 = 41,67
29. a) Y X
M i =1 6
5
XY
4 y Y = (x X )
2
3
X
2,5
2 y 1,75 = ( x 20 )
41,67
1
y 1,75 = 0,06
Figura 12 20 )
( x y = 0,55 + 0,06x
1 2 3 4 5 X
Y

b) Recta de regressió de X sobre Y: 2,4

n m 1,8
1 25 1 23
X = xi = = 4,16; Y = yj = = 3,83 1,2
N i =1 6 M j =1 6
n 0,6
1 98
XY = X i Yi X Y = 4,16 3,83 = 0,365
N i 6 O 10 20 30 40 X
1 2 107 2
2
X = X i2 X = 4,16 = 0,477
N i 6 32. a) Calculem el coeficient de correlació:
m
1 2 91 2 1 1776
2
Y = y j2 Y = 3,83 = 0,477 X = Xi = = 177,6;
M j =1 6 N i 10
XY 1 1761
x X = (y Y ) Y = Yi = = 176,1
2
Y M i 10
0,365 1 2 315 970
x 4,16 =
0,477
(y 3,83 ) 2
X =
N
X i2 X =
10
177,62 = 55,24
i

x 4,16 = 0,76 (y 3,83 ) x = 1,24 + 0,76y 1 2 310 445


2
Y = Yi 2 Y = 176,12 = 33,29
N i 10
Recta de regressió de Y sobre X:
n
1
XY 0,365 XY = X i Yi X Y =
y Y =
2
(x X ) y 3,83 =
0,477
(x 4,16 ) N i
X
313145
y 3,83 = 0,76 (x 4,16 ) y = 0,65 + 0,76x = 177,6 176,1 = 39,14
10
XY 39,14
XY 0,365 r = = = 0,91
c) r = = = 0,76 2
X
2
Y 55,24 33,29
2
X
2
Y 0,477 0,477
b) Recta de regressió de Y sobre X:
d) El coeficient de correlació és més proper a 1 que a 0; per
tant, podem dir que l’ajust realitzat és bastant bo. y Y = XY
2
(x X )
X
30. Si utilitzem les eines de la pàgina web indicada, surten els 39,14
mateixos resultats que en l’apartat anterior, encara que aga- y 176,1 = ( x 177,6 )
55,24
fant més decimals
Figura 11dels resultats.
y 176,1 = 0,7 ( x 177,6 ) y = 50,26 + 0,7x
Figura 13
31. a) Y

2,4 33. a) Notes

10
1,8

7,5
1,2

5,0
0,6

2,5
O 10 20 30 40 X
O 5 10 15 20

b) Recta de regressió de Y sobre X: Temps (h)

n m b) A partir del diagrama de dispersió, comprovem que la rela-


1 120 1 10,5
X = xi = = 20; Y = yj = = 1,75 ció entre les hores estudiades i la mitjana de les notes
N i =1 6 M j =1 6 s’aproxima bastant a una recta, de manera que sí, és apro-
1 n
225 piat.
XY = X i Yi X Y = 20 1,75 = 2,5
N i 6
m
1 2 2 650
2
X = xi 2 X = 202 = 41,67 191
M i =1 6
XY
y Y = (x X )
2
X
BLOC 4. ESTADÍSTICA I PROBABILITAT > UNITAT 13. ESTADÍSTICA BIDIMENSIONAL

c) Busquem la recta de regressió de Y sobre X, en què c) Recta de regressió de X sobre Y:


X hores i Y notes.
XY 0,6
n
x X = (y Y ) x 13,3 = (y 4,5 )
1 165
n 1 m 89 2 0,25
X = 1 x i = 165 = 11; Y = 1 m y j =89 = 5,93 Y
X = N i =1
x i = 15 = 11; Y = M y
j =1j = 15= 5,93 x 13,3 = 0,76 (y 4,5 ) x = 1,33 + 2,67y
N i =1 15 M j =1 15
1 nn 1 053
XY = 1 X i Y i X Y 1 053
= 11 5,93 = 4,97 Recta de regressió de Y sobre X:
XY = N i X i Yi X Y = 15 11 5,93 = 4,97
N i 15
1 mm 2 2 075 XY 0,6
2 =
2X =
1 x i X2
2
=2 075 112 = 17,33 y Y =
2
(x X ) y 4,5 =
3,24
(x 13,3 )
X M i =1x i 2 X = 15 112 = 17,33 X
M i =1 15
y Y = XYXY x X y 5,93 =4,97
4,97
( x 11)
y 4,5 = 0,2 (x 13,3 ) y = 1,74 + 0,2x
y Y = 2
2X
x X y 5,93((
= 17,33( x 11)
17,33
))
X d) Tenim que x 13,5; així que substituïm aquest valor en la
y 5,93 = 0,28 ( x 11) y = 2,8 + 0,28x
y 5,93 = 0,28 ( x 11) y = 2,8 + 0,28x recta de regressió de Y sobre X:

d) Un alumne estudia un parell d’hores a la setmana; és a dir, y 1,74 0,2x y 1,74 0,2 · 13,5 y 4,44
x 2. Substituïm aquest valor en l’equació anterior i tin-
e) En aquest cas, tenim que y 5,5, així que substituïm
drem la nota mitjana d’aquest alumne.
aquest valor en la recta de regressió de X sobre Y.
y 2,8 0,28 · 2 y 3,36
x 1,33 2,67y x 1,33 2,67 · 5,5 x 16

34. a) Coeficient de correlació entre les variables:


1 205 1 4 290
36. y = 0,95x + 1 XY
= 0,95 2
x = XY

X = Xi = = 34,16; Y = Yi = = 715
2
x 0,95
N i 6 M i 6
XY XY
x = 1,01y + a = 1,01 2
Y =
1 2 7163 2
2
Y 1,01
2
X = X i2 X = 34,16 = 26,47
N i 6 r = XY
= XY
=
2 2
1 2 3 077 900 X Y XY XY
2
Y = Yi 2 Y = 7152 = 1758,33 0,95 1,01
N i 6
n XY
1 147 790 = = 0,979 5
XY = X i Yi X Y = 34,16 715 = 202,5 XY
N i 6 0,959 5
XY 202,5
r = = = 0,939
2 2 26,47 1758,33
X Y
37. Sabem que r = 0, 99; 2 = 0, 8;
X Y = 1,1; per tant, si substi-
2

b) Recta de regressió de Y sobre X: tuïm en la fórmula del coeficient de correlació, tindrem:

XY XY
y Y = XY r = 0,99 = = 0,928 7
2
(x X ) 2
X
2
Y
0,8 1,1
XY

X
202,5
y 715 =
26,47
(x 34,16 ) 38. a) Per a saber si les variables estan relacionades, hem de
buscar el coeficient de correlació:
y 715 = 7, 65 ( x 34,16 ) y = 453,6 + 7,65x
1 39 1 56
X = Xi = = 4,875; Y = Yi = =7
35. Sigui X inversió i Y rendibilitat. N i 8 M i 8
1 2 225
1 80 1 27
2
X = X i2 X = 4,8752 = 4,359 38
a) X = Xi = = 13, 3; Y = Yi = = 4,5 N i 8
N 6 M 6
i i 1 2 404
2
Y = Yi 2 Y = 72 = 1,5
N 8
1 2 1 086 2 i
X = X i2 X = 13,3 = 1,8 n
N 6 1 283
i XY = X i Yi X Y = 4,875 7 = 1,25
N i 8
1 2 123
Y = Yi 2 Y = 4,52 = 0,5 XY 1,25
N i 6 r = = = 0,49
2
X
2
Y 4,359 38 1,5
b) Coeficient de correlació:
A partir d’aquest resultat, podem afirmar que les variables
n
1 364 estan relacionades, però no fortament, ja que el valor del
XY = X i Yi X Y = 13,3 4,5 = 0,6
N i 6 coeficient de correlació no és gaire proper a 1.

XY 0,6 b) Per resoldre aquest exercici, calculem la recta de regressió


r = = = 0,74
1,8 0,5 de Y sobre X:
X Y

192
BLOC 4. ESTADÍSTICA I PROBABILITAT > UNITAT 13. ESTADÍSTICA BIDIMENSIONAL

XY 1,25 d) La recta de regressió del pes sobre l’alçada és la recta de


y Y = (x X ) y 7= ( x 4,875 ) regressió de Y sobre X:
2
X 4,359 38
y 7 = 0,29 ( x 4,875 ) y = 5,602 + 0,286x y Y = XY
(x X )
2
X
Ara, com que x 7,5, substituïm aquest valor en l’equació
0,327
anterior: y 5,602 0,286 · 7,5 7,75. y 69,63 = (x 1,69 )
0,003
Per tant, la nota que s’espera que obtingui aquest alumne
y 69, 63 = (x 1,69 ) y = 114,58 + 109x
és de 7,75.
e) Tenim que x 1,69, per tant:
39. a) Aquestes rectes poden formar perfectament dues rectes
y 114,58 109 · 1,69 y 69,63
de regressió d’alguna variable bidimensional, ja que si re-
solem el seu sistema resultant obtenim una solució. f) La predicció és bona, ja que per a 1,69, que està entre els
valors 1,68 i 1,7, tenim que els valors del pes estaran entre
3x y = 0,8
x = 0,82; y = 1,65 69 i 72. Com que la nostra predicció per al pes és de
x 1,1y = 1 69,63, que està entre els valors 69 i 72, la predicció és
bona. A més a més, el valor de l’índex de correlació oposat,
b) Siguin r: 3x y 0,8 i s: x 1,1y 1. 0,975, és molt alt.
Suposem que la recta r és la regressió de Y sobre X i que s
és la regressió de X sobre Y. Si això és cert, tindrem que: 41. Utilitzant les eines de la pàgina web indicada, surten els ma-
teixos resultats que en l’apartat anterior, encara que agafant
XY XY
= 3; = 1,1 més decimals dels resultats.
2 2
X Y
1 5,5
El coeficient de Pearson és:
42. a) X = Xi = = 0,55;
N i 10
2
XY XY XY XY 1 17,62
r = r2 = r = Y = Yi = = 1,762
2 2 2 2
X Y X Y X Y M i 10
n
Substituïm les dades que hem obtingut: 1 12,417
XY = X i Yi X Y = 0, 55 1,762 = 0,27
XY XY
N i 10
r = = 1,1 3 = 1,81 > 1
X
2
Y
2 1 2 3,85
b) 2
X = X i2 X = 0,552 = 0,082 5
N i 10
Com que aquest valor és més gran que 1, arribem a una
m
contradicció, ja que aquest valor no pot ser més gran que 1 2 40,607 8
1. És a dir, la recta r és la regressió de X sobre Y i la recta s
2
Y = y j2 Y = 1,7622 = 0,956 136
M j =1 10
és la regressió de Y sobre X.
XY 0, 27
r = = = 0,97
40. Sigui X alçada i Y pes. 2
X
2
Y 0,082 5 0,956 136
1 13,53 Les variables tenen una dependència lineal positiva forta,
a) X = Xi = = 1,69;
N i 8 ja que el valor del coeficient de correlació és molt proper
1 557 a 1.
Y = Yi = = 69,63
M i 8
43. a) Sigui m 1 kg, aleshores:
1 2 22,905 9
2
X = X i2 X = 1,692 = 0,003 t2
N i 8 Y = mg
2
1 2 39 081 Y = mgX Y = gX
2 = Yi 2 Y = 69,632 = 37,5 t2
Y
N 8 X =
i 2
n
1 944,64 Aquesta equació relaciona les dues variables X i Y amb la
b) XY = X i Yi X Y = 1, 69 69,63 = 0,327 gravetat.
N i 8
b) En aquest cas, no es tracta d’una recta de regressió, ja que
XY 0,327 en l’expressió no hi ha terme independent. El valor de g és
r = = = 0,975
2
X Y
2 0,003 37,5 el quocient entre les mitjanes de Y i X.
Figura 14
1 1
c) Pes (kg) X = Xi = (10 + 12 + 15 + 8 ) = 11, 25
80
N i 4
75 1 1
Y = Yi = (100 + 118 + 150 + 80 ) = 112
70 M i 4
65 112
Y = gX 112 = g 11,25 g = g = 9,96
60 11,25

O 1,60 1,65 1,70 1,75 1,80


Per tant, el valor de la constant g per a aquests valors és:
Alçada (m) g 9,96.

193
BLOC 4. ESTADÍSTICA I PROBABILITAT > UNITAT 13. ESTADÍSTICA BIDIMENSIONAL

Figura 16

SÍNTESI Pàgs. 331 i 332 b) Y


5,5

5,0
44. a) Marginal de X: 4,5

4,0

X 10 15 20 25 3,5

3,0
fi 3 12 13 9 2,5

2,0
Marginal de Y: 1,5

1,0
Y 1 2 3 4 0,5

fi 8 9 11 9 O 0,5 1,0 1,5 2,0 2,5 3,0 3,5 4,0 4,5 5,0 5,5 X
Figura 15

b) Y
46. Calculem el valor del coeficient de correlació:
4 1 233 1 19
X = Xi = = 23,3; Y = Yi = = 1,9
3 N i 10 M i 10
1 2 6 067
2 2
X = X i2 X = 23, 32 = 63,81
N i 10
1
1 2 49
2
Y = Yi 2 Y = 1,92 = 1,29
O 7,5 15 22,5 30 X N i 10
n
1 490
XY = X i Yi X Y = 23,3 1,9 = 4,73
n N i 10
1 1
c) X = x i fi = (10 3 + 15 12 + 20 13 + 25 9 ) = XY 4,73
N i =1 37 r = = = 0,52
2 2 63,81 1,29
= 18,78 X Y

1 m
1 La recta de regressió de X sobre Y és la següent:
Y = y j fj = (1 8 + 2 9 + 3 11 + 4 9 ) =
M j =1 37 4,73
XY
x X = (y Y ) x 23,3 = ( y 1,9 )
= 2,57 2 1,29
Y
x 23,3 = 3,67 ( y 1,9 ) x = 16,33 + 3,67y
1 2 13 825
d) X = fi x i2 X = 18,782 = 4,56
N i 37 El valor del coeficient de correlació és més proper a 1 que a
0; per tant, podem dir que la relació entre les variables és
1 2 287 forta.
Y = f j y 2j Y = 2,572 = 1,08
M j 37
47. a) Coeficient de correlació:
n m
1
e) XY = x i y j fij X Y = 1 27,7 1 126
N i j X = Xi = = 5,54; Y = Yi = = 25,2
N i 5 M i 5
1
= (10 1 2 + 10 2 0 + ... + 25 4 5 ) 18,78 2,57 = 1 2 176,09
37
2
X = X i2 X = 5,542 = 4,526 4
N i 5
= 2,68
1 2 3 456
2
Y = Yi 2 Y = 25,22 = 56,16
N i 5
45. a) Construïm la taula de doble entrada, i hi indiquem les mar- 1 n
775,1
ques de classe: XY = X i Yi X Y = 5,54 25,2 = 15,412
N i 5
Yj 0,5 1,5 2,5 3,5 4,5 XY 15,412
r = = = 0,967
Xi Y 0-1 1-2 2-3 3-4 4-5
2
X
2
Y 4,5264 56,16
X
0,5 0-1 1 2 1 2 1 Recta de regressió de X sobre Y:
1,5 1-2 1 2 3 2 0
XY
x X = (y Y )
2
2,5 2-3 2 4 1 1 2 Y
15,412
3,5 3-4 0 3 1 0 0 x 5,54 = (y 25,2 )
56,12
4,5 4-5 0 1 3 1 1
x 5,54 = 0,27 (y 25,2 ) x = 1,38 + 0,27y

194
1 84 1 96
BLOC 4. ESTADÍSTICA I PROBABILITAT > UNITAT 13. ESTADÍSTICA BIDIMENSIONAL X = Xi = = 12; Y = Yi = = 13,714 3
N i 7 M i 7
1 2 1142
2
X = X i2 X = 122 = 19,143
N i 7
Recta de regressió de Y sobre X: 1 2 1578
2
Y = Yi 2 Y = 13,714 32 = 37,347
N i 7
XY
y Y = (x X ) n
2 1 1 326
X
XY = X i Yi X Y = 12 13,714 3 =
15,412 N i 7
y 25,2 = (x 5,54 )
4,5264 = 24,857
y 25,2 = 3,4 (x 5,54 ) y = 6,34 + 3,4x x X = XY
2
(y Y )
Y
b) Sigui x 5; substituïm aquest valor en la recta de regressió 24,857
de Y sobre X: x 12 =
( y 13,714 3 )
37,347
y 6,34 3,4 · 5 y 23,34 kg x 12 = 0,67 ( y 13,714 3 ) x = 2,87 + 0,67y
Sigui y 36; substituïm aquest valor en la recta de regres-
sió de X sobre Y: b) Calculem la recta de regressió de Y sobre X:

x 1,38 0,27 · 36 x 8,34 anys y Y = XY


(x X )
2
X
48. a) Trobem les rectes de regressió lineal: 24,857
y 13,714 3 = ( x 12 )
19,143
1 179 1 239
X = Xi = = 29,83; Y = Yi = = 39,83 y 13,714 3 = 1,298 ( x 12 )
N i 6 M i 6
y = 1,868 + 1,298x
1 2 5 783 2
2
X = X i2 X = 29,83 = 73,805
N i 6 Sigui x 20; substituïm aquest valor en la recta de regres-
sió que acabem de calcular:
1 2 11107 2
2
Y = Yi 2 Y = 39,83 = 264,472 y 1,868 1,298 · 20 y 24,1 persones
N i 6
n
1 7 287 50. a) Hem de buscar la recta de regressió de Y sobre X:
XY = X i Yi X Y = 29,83 39,83 =
N i 6
1 10
= 26,138 X = Xi = = 2, 5;
N i 4
Recta de regressió de X sobre Y: 1 3 462
Y = Yi = = 865,5
M i 4
XY
x X = (y Y ) 1 2 30
2
Y
2
X = X i2 X = 2,52 = 1,25
N i 4
26,138
x 29,83 =
264,472
(y 39, 83 ) 2 =
1
Yi 2 Y
2
=
9163 074
865,52 =
Y
M i 4
x 29,83 = 0,1 (y 39,83 ) x = 25,9 + 0,1y
= 1541678,25
n
Recta de regressió de Y sobre X: 1 13 317
XY = X i Yi X Y = 2,5 865,5 =
N i 4
XY
y Y = (x X ) = 1165,5
2
X
XY
26,138 y Y = (x X )
y 39,83 =
73,805
(x 29,83 ) 2
X
1165,5
y 39,83 = 0,35 (x 29, 83 ) y = 29,268 + 0,35x y 865,5 = ( x 2,5 )
1,25
y 865,5 = 932,4 ( x 2,5 )
XY 26,138
b) r = = = 0,187 y = 1 465,5 + 932,4x
2 2
X Y 73,805 264,472
Aquest resultat és lluny d’1, de manera que la dependèn- Com que volem saber el nombre de bacteris en l’hora se-
cia entre les dues variables és feble. güent, tenim que x 5; Substituïm aquest valor en l’equa-
ció anterior:
49. a) Trobem la recta de regressió de X sobre Y, on X és la varia- y 1 465,5 932,4 · 5 3 196,5
ble que determina les cendres sulfuroses contingudes en
l’aire i Y és el nombre de persones hospitalitzades més de b) Per a saber si aquest ajust és un bon ajust, hem de calcu-
7 dies per problemes respiratoris. lar el coeficient de correlació:

XY 1165,5
1 84 1 96 r = = = 0,84
X = Xi = = 12; Y = Yi = = 13,714 3 2 2 1,25 1541678,25
N i 7 M i 7 X Y

1 2 1142 Malgrat que l’ajust lineal és bo, observem que el creixe-


2
X = X i2 X = 122 = 19,143
N i 7 ment de la funció augmenta amb el pas del temps; així,
1 2 1578
2
Y = Yi 2 Y = 13,714 32 = 37,347
N i 7
1 n
1 326 195
XY = X i Yi X Y = 12 13,714 3 =
N i 7
= 24,857
BLOC 4. ESTADÍSTICA I PROBABILITAT > UNITAT 13. ESTADÍSTICA BIDIMENSIONAL

entre la primera i la segona hores, l’augment és de 48 bac- 2. Construïm una taula de doble entrada per facilitar els càlculs
teris, entre la segona i la tercera, de 345, i entre la tercera dels diferents apartats:
i la quarta, de 2 600. Probablement, una funció exponenci-
al s’ajustaria millor al comportament del cultiu. Yj 1 3 5 7 9 11
XY XY Y
51. a) Sabem que b = 2
ob =
2
on el numerador sem- X 0-2 2-4 4-6 6-8 8-10 10-12 fi
X Y
pre és positiu perquè és un quadrat. Com que XY és po- 0 3 5 8 7 5 3 31
sitiu, aleshores el valor de b no pot ser negatiu.
1 1 2 4 7 3 1 18
XY XY XY
b) Sabem que b = ob = ir = i que els 2 1 1 1 1 1 0 5
2 2
X Y X Y
signes de les variàncies i les desviacions típiques són posi- 3 0 1 1 0 0 1 3
tius. Aleshores, X > 0, Y > 0, X 2 > 0, Y 2 > 0 .
4 0 0 1 0 1 0 2
Com que el valor del coeficient de correlació és positiu,
5 0 0 0 0 0 1 1
aleshores el valor de la covariància també és positiu. Això
contradiu el fet que b sigui negatiu. fi 5 9 15 15 10 6 60
XY 120
c) Sabem que r = = = 10,33 > 1.
2
X
2
Y 9 15 1 n
50
a) X = x i fi = = 0,83;
Això no pot ser, ja que el valor del coeficient de correlació N 60
i =1
ha d’estar entre els valors de 1 i 1. 1 m
368
XY 15 Y = y j fj = = 6,13
d) Sabem que r = = = 0,71 < 1. M j =1 60
2
X
2
Y 30 15
1 2 122
Aquest resultat sí que és possible, ja que el valor del coefi- X
2 = fi x i2 X = 0,832 = 1,34
cient de correlació està entre els valors de 1 i 1. N i 60
1 2 2 732
2 = f j y 2j Y = 6,132 = 7,92
52. a) Aquestes rectes poden formar perfectament dues rectes Y
M 60
j
de regressió d’alguna variable bidimensional, ja que, si re-
solem el seu sistema resultant, obtenim una solució. 1 n m
b) XY = x i y j fij X Y =
3x + 2y = 7 9 4 N i j
x = ;y =
x + 4y = 5 5 5 1
= ( 0 1 3 + 0 3 5 + ... + 5 11 1) 0, 83 6,13 =
b) Suposem que la recta r és la regressió de Y sobre X i que 60
s és la regressió de X sobre Y. Si això és cert, tindrem = 0,42
que:
c) Per a saber la relació que hi ha entre les variables, hem de
r: 3x 2y 7 y 3x/2 7/2 calcular el valor del coeficient de correlació:
s: x 4y 5 x 5 4y XY 0,42
r = = = 0,129
3
2
X
2
Y 1,34 7,92
XY XY
= ; = 4
X
2 2 Y
2
Com que aquest valor és molt proper a 0, la dependència
El coeficient de Pearson és: lineal entre les variables és molt feble.
2
XY XY XY XY
r = r2 = r = 3. Les rectes de regressió es tallen en el centre de gravetat d’una
2 2 2 2
X Y X Y X Y
distribució, és a dir, en el punt X ,Y . Plantegem el sistema ( )
Substituïm les dades que hem obtingut: següent i el resolem:

XY XY 3 y = 2,6x 2,55
r = = ( 4 ) = 2,45 > 1 x = 1,28; y = 0,78
X
2
Y
2 2 x = 0,36y + 1
Com que aquest valor és més gran que 1, arribem a una Per tant, la solució del sistema anterior coincideix amb les
contradicció, de manera que queda falsejada la hipòtesi mitjanes de X i de Y. És a dir, X = 1, 28; Y = 0, 78.
inicial. És a dir, la recta r és la regressió de X sobre Y i la
recta s és la regressió de Y sobre X. Calculem el coeficient de correlació:

XY XY
y = 2,6x 2,55 = 2,6 2 =
Avaluació (pàg. 334) 2
x
x
2,6
XY XY
1. a) El color de cabells i el color d’ulls són variables qualitatives. x = 0,36y + 1 = 0,36 2
Y =
2
Y 0,36
b) El nombre de germans i el nombre d’ordinadors a casa són XY XY XY
variables quantitatives i discretes. r = = = = 0,967
2 2 XY
X Y XY XY
2,6 0,36 0,967

196
BLOC 4. ESTADÍSTICA I PROBABILITAT > UNITAT 13. ESTADÍSTICA BIDIMENSIONAL

4. Busquem el coeficient de correlació: Recta de regressió de Y sobre X:

XY XY
y = 3x 1 =3 2
x = XY 1,94
2
x 3 y Y =
2
(x X ) y 28,3 =
1,805
(x 38,16 )
X
0, 6
XY = 0,6 2
x = 2
x = 0,2 y 28,3 = 1,077 (x 38,16 ) y = 12,77 + 1,077x
3
XY 0,6 0,6
r = = = = 0,67 c) El coeficient de correlació és el següent:
2
X
2
Y 0,2 22 0,2 4

XY 1,94
r = = = 0,734
5. a) Primerament, calculem el coeficient de correlació: 2 2
1,805 3,8
X Y
1 712 1 79
X = Xi = = 142,4; Y = Yi = = 15,8 Aquest valor és molt proper a 1, la qual cosa significa que
N i 5 M i 5
la dependència entre les dues variables és forta.
1 2 102 576
2
X = X i2 X = 142,42 = 37,44
N i 5 7. a) En aquest gràfic observem que les variables tenen una
1 2 1285 dependència lineal positiva feble, ja que els punts estan
2
Y = Yi 2 Y = 15,82 = 7,36
N i 5 bastant dispersos.
n
1 11 320 b) En aquest cas, les variables tenen una dependència lineal
XY = X i Yi X Y = 142,4 15,8 = 14,08
N i 5 negativa forta, ja que els punts es podrien aproximar fàcil-
14,08 ment per una recta. Aquí, la correlació és negativa i prope-
XY
r = = = 0,848 ra a 1.
2
X
2
Y 37,44 7,36
c) Aquest gràfic presenta una dependència lineal positiva
La recta de regressió de Y sobre X és la següent: forta entre les variables, pel mateix motiu que en el gràfic
XY
anterior. La correlació és positiva i propera a 1.
y Y = (x X )
2
X d) Finalment, les variables representades en aquest gràfic
14,08 són independents, ja que els punts no formen cap recta.
y 15,8 = ( x 142,4 ) En aquest cas, la correlació és propera a 0.
37,44
y 15,8 = 0,376 ( x 142,4 ) y = 37,75 + 0,376x 8. a) El signe de la covariància i el del coeficient de correlació
b) Sigui x Figura
135;17
substituïm aquest valor en la recta de re- han de ser els mateixos, ja que les variàncies i les desviaci-
gressió de Y sobre X: ons típiques són sempre positives. Per tant, aquest cas no
es pot donar, ja que la covariància i el coeficient de corre-
y 37,75 0,376 · 135 13,01 milions de tones.
lació tenen signes diferents.
6. a) Y b) Aquest cas tampoc no es pot donar, ja que el valor del co-
32 eficient de correlació no pot ser més gran que 1.
30
c) Aquesta opció sí que és vàlida, perquè els signes de la co-
28
variància i del coeficient de correlació són iguals i el valor
26
de r està entre 1 i 1.
24
d) Aquest resultat també és possible per la mateixa raó ante-
O 36 37 38 39 40 X rior. Els signes són iguals i el valor del coeficient de correla-
b) Calculem les dades necessàries per a trobar les rectes de ció està entre 1 i 1.
regressió:
9. a) Aquestes rectes poden ser perfectament dues rectes de
1 229 1 170
X = Xi = = 38,16; Y = Yi = = 28,3 regressió d’alguna distribució bidimensional, ja que, si
N i 6 M i 6 resolem el seu sistema resultant, obtenim una solució:
1 2 8 751 2
2
X = X i2 X = 38,16 = 1,805 7x + 3y = 2 5 59
N i 6 x = ;y =
2x + 6y = 9 12 36
1 2 4 840 2
2
Y = Yi 2 Y = 28,3 = 3,8
N i 6 b) Siguin r: 7x 3y 2 i s: 2x 6y 9.
n
1 6 500
XY = X i Yi X Y = 38,16 28,3 = 1,94 Suposem que la recta r és la regressió de Y sobre X i que s
N i 6 és la regressió de X sobre Y. Si això és cert, tindrem que:
Recta de regressió de X sobre Y:
7x + 3y = 2 y = 7x / 3 + 2/ 3
XY 1,94 2x + 6y = 9 x = 4,5 – 3y
x X =
2
(y Y ) x 38,16 =
3,8
(y 28,3 )
Y
XY 7 XY
= ; = 3
x 38,16 = 0,5 (y 28,3 ) x = 24 + 0,5y
X
2 3 Y
2

197
BLOC 4. ESTADÍSTICA I PROBABILITAT > UNITAT 13. ESTADÍSTICA BIDIMENSIONAL

El coeficient de Pearson és: Zona (pàg. 335)


2
r = XY
r2 = XY
r = XY XY — El naixement de l’estadística aplicada
2 2 2 2
X Y X Y X Y
El significat del coeficient de Pearson és determinat de la ma-
Substituïm les dades que hem obtingut: nera següent:
XY XY 7 t Si r 0, no hi ha correlació.
r = = ( 3 ) = 2,65 > 1
X
2
Y
2 3
t Si r 1, la correlació és positiva perfecta, i podem dir que hi
Però sabem queFigura
r no pot
18 ser més gran que 1. És a dir, la ha una relació funcional lineal entre totes dues variables.
recta r és la regressió de X sobre Y i la recta s és la regres-
sió de Y sobre X. t Per a 0 r 1, la correlació és positiva.

t Si r 1, la correlació és negativa perfecta, i podem dir que


10. Y
hi ha una relació funcional lineal entre totes dues variables.
10
t Per a 1 r 0, la correlació és negativa.
7,5

5,0
— Correlacions que confonen

2,5 La paraula espuri ve del llatí spurius, i accepta dues definicions:


1) ’bastard’ (degenerat del seu origen) i 2) ’enganyós, fals’.
O 2,5 5,0 7,5 10 X
La segona accepció té el seu millor exemple en estadística, en
Atès que la distribució de punts és creixent, el valor de r ha de
què es parla de, correlació espúria o il·lusòria, que es produeix
ser positiu; per tant, només poden ser els valors a) i d). D’altra
banda, els punts estan bastant dispersos, per la qual cosa el quan tenim una situació en la qual les mesures de dues o més
valor del coeficient de correlació ha de ser més a prop de 0 variables estan estadísticament relacionades, però en realitat
que d’1. Aleshores, l’opció més adequada és l’opció d), 0,4. no hi ha cap relació de causalitat entre elles.

198
BLOC 4. ESTADÍSTICA I PROBABILITAT

14# Probabilitat

En context (pàg. 337) Problemes resolts (pàgs. 347 i 348)

a) Tots els nombres tenen la mateixa probabilitat de sortir.


1. a) Designem R l’esdeveniment «treure un rei», i A, «treure un
Moltes persones prefereixen uns nombres determinats per as». Per treure en total dos reis i dos asos, hi ha sis combi-
motius subjectius, com la coincidència amb una data, però nacions en funció de l’ordre en què apareixen: RRAA,
no tenen una probabilitat més gran de sortir premiats. RARA, RAAR, ARRA, ARAR i AARR. Atès que la probabili-
tat de treure un rei o un as és d’1/10, cadascuna d’aques-
En el sorteig de Reis de la loteria surten premiats una mica
tes combinacions té una probabilitat de (1/10)4. Per tant,
menys del 16 % dels bitllets; en la loteria setmanal, gairebé
la probabilitat total d’obtenir dos reis i dos asos serà sis
el 60 %, i en la primitiva, no arriben al 12 %. Així, la loteria
vegades aquesta probabilitat.
que ens pot donar algun premi amb més facilitat és la loteria
setmanal. 4
1
P(2R2A) = 6 = 0,000 6
b) Resposta oberta a manera de reflexió personal. 10

Imagen 01 b) La probabilitat d’obtenir una figura és de 3/10, i la d’obte-


Amplia (pàg. 339) nir un as, d’1/10. Les combinacions possibles d’obtenir
tres figures i un as, tenint en compte l’ordre, són: AFFF,
(A B) = A B
FAFF, FFAF i FFFA, és a dir, quatre en total.
A
3
B 3 1
A B A B
P(3F 1A) = 4 = 0,010 8
10 10
Imagen 02

2. a) En el joc del dòmino, hi ha 28 fitxes i 7 d’aquestes són


(A B) = A B dobles. Si n’agafo una a l’atzar, la probabilitat que sigui
A
doble és 7/28, la segona, 6/27, i la tercera, 5/26.
B
7 6 5
P(3D) = = 0,010 7
A B = A B A B 28 27 26

b) És més senzill calcular la probabilitat que cap peça no tin-


gui un 1. La probabilitat que sigui doble és 7/28, la de la
Internet (pàg. 339) segona és 20/27 i la de la tercera, 19/26.
Idempotent: A A=A 21 20 19
P(No 1) = = 0,406
Simplificativa: A (A B) = A 28 27 26
P(Algun 1) = 1 P(No 1) = 1 0,406 = 0,594
Involució: A = A

3. a) P(A) = 1 P(A) = 1 0,4 = 0,6


Internet (pàg. 340)
P(A B) = P(A) P(B / A) = 0,6 0,3 = 0,18
Observem que, quan el nombre de llançaments augmenta, la
freqüència relativa de cadascun dels resultats s’apropa pro- b) P(A) + P(B) = P(A B) + P(A B)
gressivament a 1/6. P(B) = P(A B) + P(A B) P(A) = 0,28

c) P(A) = P(A B) + P(A B) P(A B) = 0,42


Internet (pàg. 344) P(B A)
P(B / A) = = 0,7
Diversos esdeveniments són independents si l’ocurrència P(A)
d’un no condiciona la dels altres. En aquests casos, la proba-
d) P(A B) = P(A) + P(B) P(A B) = 0,9
bilitat que es produeixin tots aquests esdeveniments és el
producte de les probabilitats de cadascun.
4. a) La probabilitat que un vehicle tingui un accident serà la
suma que sigui un cotxe i tingui l’accident, un camió i tin-
Internet (pàg. 346) gui un accident, i una moto i tingui un accident.

P(N1 R2 ) P(N1 R2 ) P(Acc.) = 0,6 0,02 + 0,25 0,06 + 0,15 0,12 = 0,045
P(N1 /R2 ) = = =
P(R2 ) P(A1 R2 ) + P(R1 R2 ) + P(N1 R2 )
b) Si sabem que el vehicle ha tingut un accident, calculem la
P(N1)P(R2 /N1) probabilitat condicionada que el vehicle sigui un camió, i,
=
P(A1)P(R2 / A1) + P(R1)P(R2 /R1) + P(N1)P(R2 /N1) a continuació, la que hagi estat una motocicleta.

199
BLOC 4. ESTADÍSTICA I PROBABILITAT > UNITAT 14. PROBABILITAT

P(Cam. Acc.) 0,25 0,06


P(Cam./ Acc.) = = = 0,3 e) D (B C ) {b, c, d, e, f} {a, e} {b, c, d, f}
P(Acc.) 0,045
f) ( A B ) (C D) {a, e, f, g, i, j } {c, e} {a, f, g, i, j }
P(Moto Acc.) 0,15 0,12
P(Moto / Acc.) = = = 0,4
P(Acc.) 0,045
P [ (Cam. Acc.) (Moto Acc.) ] = 0,3 + 0,4 = 0,733 3 PROBABILITAT Pàgs. 349 i 350

5. a) La probabilitat que arribi sense retard és la suma dels que


11. En total hi ha 10 casos possibles.
arriben sense retard en autocar, els que arriben sense re- 3
a) P(Verda) = = 0,3
tard amb avió i els que arriben sense retard amb tren. 10
P(No ret ) = P(No ret Aut.) + P(No ret Avió) + 2
b) P(B) = = 0,2
+ P(No ret Tren) = 0,4 0,1 + 0,25 0,65 + 0,35 0,95 = 10
= 0,535 6
c) P(No verm.) = = 0,6
10
b) La probabilitat d’arribar amb retard és 1 0,535 0,465.
A continuació calculem la probabilitat condicionada que
12. {2, 3, 4, 5, 6, 7, 8, 9, 10, 11, 12}.
viatgi amb avió sabent que va amb retard.
No tots els resultats són igualment probables; per exemple,
P(Avió /Ret.) 0,25 0,35
P(Avió /Ret.) = = = 0,188 perquè el resultat sigui 2 només hi ha una combinació (1, 1),
P(Ret.) 0,465 perquè el resultat sigui 3 hi ha dues combinacions, (1, 2) i (2,
1); el resultat més probable és 7, ja que hi ha sis combinaci-
ons: (1, 6), (2, 5), (3, 4), (4, 3), (5, 2) i (6, 1).
Exercicis i problemes (pàgs. 349 a 352)
C. favorables 1
P(2) = = = 0,027 8 = P(12)
C. possibles 36
1 EXPERIMENTS ALEATORIS Pàg. 349 P(3) = P(11) =
2
= 0,055 6
36
6. a) Sí, ja que no podem predir el resultat. 3
P(4) = P(10) = = 0,083 3
b) No, ja que de segur que la bola extreta és blanca. 36
4
c) No, ja que la física prediu aquest resultat. P(5) = P(9) = = 0,1111
36
7. a) {(v,v), (v,g ), (v,b), (g,v), (g,g), (g,b), (b,v), (b,g), (b,b)}
P(6) = P(8) =
5
= 0,138 9
b) {(v,v), (v,g ), (v,b), (g,v), (g,b), (b,v), (b,g), (b,b)} 36
6
P(7) = = 0,166 7
8. {(Esp-Ita, Bra-Arg), (Esp-Bra, Ita-Arg), (Esp-Arg, Ita-Bra)} 36
Si considerem que un equip pot jugar com a local o com a vi-
sitant, l’espai mostral és més gran:
13. Partim dels resultats del problema 12.
a) Són múltiples de 3 els nombres 3, 6, 9 i 12. Sumem les
{(Esp-Ita, Bra-Arg), (Esp-Ita, Arg-Bra), (Esp-Bra, Ita-Arg),
probabilitats corresponents:
(Esp-Bra, Arg-Ita), (Esp-Arg, Ita-Bra), (Esp-Arg, Bra-Ita),
(Ita-Esp, Bra-Arg), (Ita-Esp, Arg-Bra), (Bra-Esp, Ita-Arg), 2 5 4 1 12
P(Múlt. 3) = + + + = = 0,33
(Bra-Esp, Arg-Ita), (Arg-Esp, Ita-Bra), (Arg-Esp, Bra-Ita)} 36 36 36 36 36

b) Són divisibles per 4 els nombres 4, 8 i 12. Sumem les pro-


babilitats corresponents:
2 ESDEVENIMENTS Pàg. 349
3 5 1 9
P(Div. 4) = + + = = 0,25
9. a) {(v, 1), (v, 3), (v, 5)} 36 36 36 36
b) {(v, 3), (v, 6), (n, 3), (n, 6)}
1 1 1 5
c) {(n, 2)} 14. a) P(A B) = P(A) + P(B) P(A B) = + =
3 4 6 12
d) {(v, 2), (v, 4), (v, 6), (n, 2), (n, 4), (n, 6)} 1 2
b) P(A) + P(A) = 1 P(A) = 1 P(A) = 1 =
3 3
10. a) A B {a, e, f, g, j } {b, c, d, f, g, h, j } {f, g, j }
c) P(A B) + P(A B) = P(A)
b) A B {b, c, d, h, i} {b, c, d, f, g, h, j } {b, c, d, h} 1 1 1
P(A B) = P(A) P(A B) = =
c) B C D
( ) {a, e, i} {a, b, d, f, g, h, i, j } {a, b, d, e, 3 6 6
f, g, h, i, j }
5 7
d) P(A B) = 1 P(A B) = 1 =
d) C D {a, c, e, g} {a, g, h, i, j } {a, c, e, g, h, i, j } 12 12

200
BLOC 4. ESTADÍSTICA I PROBABILITAT > UNITAT 14. PROBABILITAT

15. La probabilitat de superar alguna de les proves és la probabi- Sumem totes aquestes combinacions, i tenim 144 casos
litat de la unió d’aprovar la teòrica i la pràctica. favorables.
P(T P) = P(T ) + P(P) P(T P) = 144 144
P(3F ) = = = 0,002
= 0,45 + 0,4 0,3 = 0,55 V40
3 40 39 38

d) Calculem la probabilitat que no surti cap or i després res-


16. El nombre total d’ordinadors amb errors de programari és
tem aquesta probabilitat a 1.
17  40 57; respecte a un total de 600 ordinadors, la proba-
bilitat d’errors al programari és: V30
3 30 29 28
P(No or ) = = = 0,410 9
57 V40
3 40 39 38
P(E . P.) = = 0,095
600 P(Algun or ) = 1 P(No or ) = 1 0,410 9 = 0,5891
El nombre d’ordinadors infectats amb algun virus és de 17
23 40. Per tant: 20. Designem d el fet que sigui una noia, i f, l’esdeveniment que
40 hagi triat francès. Els fets que siguin nois o hagin triat alemany
P(Virus) = = 0,066 7 seran designats per les negacions dels anteriors.
600
15 19
P(d) = ; P(d) + P(d) = 1 P(d) = 1 P(d) =
17. a) La probabilitat que alguna arribi tard és la unió que la Irene 34 34
arribi tard i que la Maria arribi tard. 16 18
P(f ) = ; P(f ) + P(f ) = 1 P(f ) = 1 P(f ) =
P(MT IT ) = P(MT ) + P(IT ) P(MT IT ) = 34 34
= 0,34 + 0,2 0,1 = 0,44 10
P(f d) =
34
b) La probabilitat que cap no arribi tard és 1 menys la proba-
bilitat que alguna arribi tard. a) P(f d) + P(f d) = P(d)

P(0T ) = 1 P(MT IT ) = 1 0,44 = 0,66 19 10 9


P(f d) = P(d) P(f d) = = = 0,264 7
34 34 34
18. En el cas del dau, {1, 2, 3, 4, 5, 6}. Sabem que la proba- b) P(f d) + P(f d) = P(f )
bilitat que el resultat sigui imparell és el doble que sigui parell;
a més a més, la suma de les probabilitats ha de ser 1. 18 9 9
P(f d) = P(f ) P(f d) = = = 0,2647
34 34 34
P(i ) = P(1) = P(3) = P(5) P(p) = P(2) = P(4) = P(6)
P(i ) = 2P(p)
21. L’espai mostral és {1, 2, 3, 4, 5, 6}. La probabilitat de
1 2 cada resultat és inversament proporcional al resultat, de ma-
1 = 3P(i ) + 3P(p) = 9P(p) P(p) = P(i ) =
9 9 nera que

Els divisors de 6 són 1, 2, 3 i 6. Per tant, la probabilitat que no k k k k k k


P(1) = ; P(2) = ; P(3) = ; P(4) = ; P(5) = ; P(6) =
surti divisible entre 6 és: 1 2 3 4 5 6
1 2 3 1 La probabilitat total ha de sumar 1; per tant:
P(No div. 6) = P(4) + P(5) = + = =
9 9 9 3
k k k k k k 147k 20
+ + + + + =1 =1 k =
1 2 3 4 5 6 60 49
V43 4 3 2
19. a) P(3A) = = = 0,000 4 La probabilitat que sigui un múltiple de 3 o de 5 és:
V40
3 40 39 37
k k k 10
b) En aquest cas, hem de sumar la probabilitat que surtin P(Múlt. 3) = P(3) + P(6) = + = = = 0,204 1
tres figures amb la probabilitat obtinguda en l’apartat an- 3 6 2 49
terior. 4 k
P(Múlt. 5) = P(5) = = = 0,0816
5 49
V123 12 11 10 P(Múlt.. 3 Múlt. 5) = 0,204 1 + 0,0816 = 0,285 7
P(3F ) = = = 0,022 3
V40
3 40 39 38
P(3F 3A) = 0,022 3 + 0,000 4 = 0,022 7 22. Definim les nostres variables, P (An ) és la probabilitat que un
participant parli només anglès, P (Al ) i P (F ) són les probabili-
c) Calculem el nombre de casos favorables. Es poden donar tats que parli només alemany o francès, respectivament.
tres casos: P (An-Al), P (An-F ) i P (Al-F ) són les probabilitats que sàpiga
només anglès i alemany, només anglès i francès o només
— La primera carta és un rei, la segona un rei i la tercera
alemany i francès. P (An-F-Al ) és la probabilitat de parlar tots
un tres. Hi ha 4 3 4 48 combinacions.
tres idiomes. La suma de totes aquestes probabilitats ha de
— La primera carta és un rei, la segona un tres i la tercera ser 1, i, a més a més, podem escriure les condicions de
un rei. Hi ha 4 4 3 48 combinacions. l’enunciat en forma de sis equacions més.
— La primera carta és un tres, la segona un rei i la tercera P(An) + P(F ) + P(Al ) + P(An-F ) + P(An-Al ) + P(F -Al ) +
un rei. Hi ha 4 4 3 48 combinacions. + P(An-F -Al ) = 1

201
BLOC 4. ESTADÍSTICA I PROBABILITAT > UNITAT 14. PROBABILITAT

P(An) + (An-F ) + P(An-Al ) + P(An-F -Al ) = 0,65 En el segon cas, calculem la probabilitat condicionada que
les natilles toquin el dimarts, sabent que no han tocat el di-
P(Al ) + P(An-Al ) + P(F -Al ) + P(An-F -Al ) = 0,59 lluns.
P(F ) + P(An-F ) + P(F -Al ) + P(An-F -Al ) = 0,5 1
P(Dimarts Dilluns) 5 = 0,25
P(An-Al ) + P(An-F -Al ) = 0,32 P(Dimarts / Dilluns) = =
P(Dilluns) 4
5
P(An-F ) + P(An-F -Al ) = 0,21
P(F -Al ) + P(An-F -Al ) = 0,22 3 1
27. a) P(B) = P(B) = 1 P(B) =
4 4
Resolem aquest sistema d’equacions, i obtenim:
P(A B) = P(A) + P(B) P(A B)
P(An) = 0,13; P(F ) = 0,08; P(Al ) = 0,06; P(An-F ) = 0,2 1 1 17 1
P(A B) = P(A) + P(B) P(A B) = + =
P(An-Al ) = 0,31; P(F -Al ) = 0,21; P(An-F -Al ) = 0,01 3 4 36 9
a) Tenint en compte els resultats anteriors, veiem que la pro- P(A B) 19 4
P(A /B) = = =
babilitat que parli els tres idiomes és P (An-F-Al ) 0,01. P(B) 14 9

b) P (An-Al ) P (F-Al ) P (An-F-Al ) 0,53 b) P(A B) + P(A B) = P(B)


c) P (An ) P (Al ) P (F ) 0,27 1 1 5
P(A B) = P(B) P(A B) = =
4 9 36
23. a) La gràfica representa l’evolució de la freqüència d’obtenir cara P(A B) 5 36 5
o creu a mesura que augmenten el nombre de llançaments. P(A / B) = = =
P(B) 14 9
b) A mesura que augmentem el nombre de llançaments, la
c) P ( A B ) B = P(A B), ja que no es poden produir
freqüència s’estabilitza al voltant del valor 0,5.
simultàniament B i el seu complementari.
c) Si reiniciem el simulador, veurem que les freqüències inici-
als en cada cas són diferents. P(A B) + P(A B) = P(A)
1 1 2
P(A B) = P(A) P(A B) = =
3 9 9
4 PROBABILITAT
2
P (A B ) B 9 = 8
CONDICIONADA Pàgs. 350 i 351 P (A B ) / B = =
P(B) 3 27
C. favorables 4 4
24. a) P(Primer ) = = = 0,4
C. possibles 10
V124 12 11 10 9
3 28. P(4 nois) = = = 0,018
b) P(Múlt. 3) = = 0,3 V30
4 30 29 28 27
10
7
c) P(X < 4 X 7) = = 0,7 1
10 29. Probabilitat de treure tres sisos: P(6, 6, 6) = = 0, 004 6
63
4
d) P(Parell 5) = = 0,4 Probabilitat condicionada de treure tres sisos després d’haver
10
tret dos sisos:
P((4 8) Parell ) 2
e) P(4 8/Parell ) = = 10 = 0,4 1
P(Parell ) 5 P(6, 6, 6 6, 6) 63 = 0,166 7
10 P(6, 6, 6/6, 6) = =
P(6, 6) 1
62
1
25. a) A i B no són incompatibles, ja que P(A , de ma- B) = Com veiem, es tracta d’esdeveniments independents, ja que
6
el resultat de llançar el tercer dau no depèn del resultat dels
nera que hi ha una certa probabilitat que s’esdevinguin
llançaments anteriors.
alhora.
P(A B) 1 1 30. La probabilitat que una persona estigui a l’atur és la suma de
b) P(A /B) = = 6 = = P(A)
P(B) 1 2 la probabilitat que sigui dona i estigui a l’atur més la que sigui
3
un home i estigui a l’atur.
P(B A) 1
P(B / A) = 6 = 1 = P(B)
= P(Atur ) = P(Atur D) + P(Atur H) =
P(A) 3 1
2 = 0,213 0,516 + 0,208 0,484 = 0,210 6
La probabilitat que es produeixi A condicionada a B és
igual a la probabilitat que s’esdevingui A. Dit d’una altra 31. Designem C l’esdeveniment «portar el cinturó», i DG, «sofrir
manera, la probabilitat de A no varia pel fet que es produ- danys greus».
eixi B, de manera que es tracta d’esdeveniments indepen-
dents. Podem dir el mateix sobre B. P(DG) = 0,725; P(C DG) = 0,905; P(C) = 0,9
a) P(C DG) = P(C) + P(DG) P(C DG)
26. Em perdré les natilles si han tocat el dilluns, atès que tots els
dies tenen la mateixa probabilitat: P(C DG) = P(C) + P(DG) P(C DG) =
= 0,9 + 0,725 0,905 = 0,72
1
P(Dilluns) = = 0,2
5 b) P(DG) = P(C DG) + P(C DG)

202
BLOC 4. ESTADÍSTICA I PROBABILITAT > UNITAT 14. PROBABILITAT

P(C DG) = P(DG) P(C DG) = 0,725 0,72 = 0,005 que hi ha 100 combinacions de dos nombres, hi haurà
sengles xifres capicues.
P(C DG) 0,005
P DG / C =
( ) = = 0,05 100
P(C) 0,1 P(Capicua) = = 0,01
10 000
32. La probabilitat de trobar feina el primer any és la suma de la d) Entre el 0000 i el 9 996 hi ha 2 500 múltiples de 4, de ma-
probabilitat d’acabar la carrera i trobar feina el primer any, nera que
més la de no acabar la carrera i trobar-ne.
2 500
P(F ) = P(F C) + P(F C) = 0,8 0,7 + 0,2 0,58 = 0,676 P(Múlti. 4) = = 0,25
10 000

33. a) És la suma dels que han superat la prova i són del primer e) Són més grans o iguals a 9 000 un total de 1 000 nombres.
institut, els que l’han superat i són del segon i els que Són parells 5 000 nombres. Són més grans o iguals a 9 000
l’han superat i són del tercer. Sigui A l’esdeveniment i parells 500 nombres.
«aprovar», i I1, I2 i I3 el fet de ser del primer, del segon o
del tercer institut. El nombre total d’alumnes és 175, de P(X 9 000 Parell) = P(X 9 000) + P(Parell)
manera que 1 000 5 000 500
P(X 9 000 Parell) = + = 0,55
P(A) = P(A I1) + P(A I2) + P(A I 3) = 10 000 10 000 10 000
40 60 75 f) Calculem la probabilitat que no contingui cap 7 i després
= 0,8 + 0,9 + 0,82 = 0,842 9
175 175 175 la restem a 1. Per a cada xifra, hi ha una probabilitat de
9/10 que no sigui un 7.
b) P(A) = 0,842 9 P(A) = 1 P(A) = 0,1571
9 9 9 9
40 P(7) = = 0,656 1
P(1I A) = 0,2 = 0,045 71 10 10 10 10
175
P(7) = 1 P(7) = 0,343 9
P(1I A) 0, 045 71
P(1I / A) = = = 0,291
P(A) 0,1571 2 000
g) P(Div. 5) = = 0,2
10 000
P(A B) P(A B) 0,2 200
34. a) P(A / B) = P(B) = = = 0,4 P(X 9 000 Div. 5) = = 0,02
P(B) P(A / B) 0,5 10 000
b) P(B) = P(B A) + P(B A) P(X 9 000 Div. 5) 0,02
P(X 9 000/Div. 5) = = = 0,1
P(B A) = P(B) P(B A) = 0,4 0,2 = 0,2 P(Div. 5) 0,2
P(A) = P(A B) + P(A B) = 0,2 + 0,3 = 0,5
P(A) = 1 P(A) = 0,5 37. a) Calculem la probabilitat que no hi hagi cap empat i que
n’hi hagi un. Restem aquestes dues probabilitats a 1.
P(A B) = P(A) + P(B) P(A B) =
15
= 0,5 + 0,4 0,2 = 0,7 5
P(X = 0) = = 0,064 9
6
c) Com hem calculat en l’apartat b), P(A) = 0,5
14
P(A B) 0,3 1 5
d) P(B / A) = = = 0,6 P(X = 1) = 15 = 0,194 7
P(A) 0,5 6 6
P(X 2) = 1 P(0X ) P(1X ) = 0,74
35. Designem P l’esdeveniment «punxar», i A, M i B, les rodes de
qualitat alta, mitjana i baixa, respectivament. b) La probabilitat de treure un 1 és de 3/6 = 1/2, i la de treure
un 2 és de 2/6 = 1/3. La probabilitat de treure, per exem-
a) Triar cada tipus de roda té una probabilitat d’1/3. ple, primer nou uns i després sis dosos és de
P(P) = P(P A) + P(P M) + P(P B) = 9 6
1 1
1 1 1 P(9 1,6 2) = = 2,68 10 6
= 0,01 + 0,1 + 0,2 = 0,103 3 2 3
3 3 3
1 Tanmateix, hi ha múltiples maneres d’ordenar nou uns i sis
0,2
P(B P) dosos en quinze vacants; concretament:
b) P(B /P) = = 3 = 0,645
B(P) 0,103 3 15 !
C15
9 = = 5 005
9! 6!
36. Hi ha 10 000 combinacions de 4 xifres.
5 000 La probabilitat de treure nou uns i sis dosos serà el produc-
a) P(Parell ) = = 0,5
10 000 te de treure una d’aquestes combinacions pel nombre
1 000 d’ordenacions possibles; és a dir:
b) P(X < 1 000) = = 0,1
10 000 P(9 1 6 2) = 2,68 10 6 5 005 = 0,013 4
c) Hem de saber quants nombres capicua hi ha; donats els
dos primers nombres, hi ha una combinació de dos nom- c) De totes les combinacions possibles, necessitem saber en
bres darrers que fa que la xifra final sigui capicua. Com quantes hi ha sis dosos:

203
BLOC 4. ESTADÍSTICA I PROBABILITAT > UNITAT 14. PROBABILITAT

6 9 vermella a l’urna, de manera que la probabilitat que això es


1 2
P(6 2) = C15
6 = 5 005 3,57 10 5 = 0,178 6 produeixi és 0,25 0,25 0,5.
3 3
42. Dibuixem un diagrama en arbre per entendre millor el proble-
P (( 9 1 6 2) 6 2) ma. La probabilitat d’extreure la bola de la primera urna és
P(9 1 6 2/6 2) = =
P(6 2) 1/3, i la d’extreure-la de la segona és 2/3.
P(9 1 6 2) 0,013 4
= = = 0,075 5/11
P(6 2) 0,178 6 Blanca

1/3 Primera 2/11


38. Designem M l’esdeveniment «tenir mòbil», i O, «tenir ordina- Vermella
urna
dor».
4/11
P(O M) = 0,48; P(O /M) = 0,6; Negra
P(O M) P(O M) Experiment
P(O /M) = P(M) = = 0,8 3/13
P(M) P(O /M) Blanca

P(M) = 1 P(M) = 0,2 2/3 Segona 6/13


Vermella
urna
39. Calculem la probabilitat que no surti cap dels temes que ha 4/13
estudiat. La probabilitat que el primer tema no hagi estat estu- Negra
diat és de 8/12, la del segon, 7/11, i la del tercer, 6/10.
La probabilitat que sigui blanca és la suma de treure-la de la
8 7 6 336
P(A) = = = 0,254 5 primera urna més la de treure-la de la segona urna.
12 11 10 1 320
1 5 2 3
P(A) = 1 P(A) = 0,745 5 P(B) = P(1U B) + P(2U B) = + =
3 11 3 13
= 0,305 4
40. Dibuixem un diagrama en arbre:
1 2 2 6
Primer tir Segon tir P(V ) = P(1U V ) + P(2U V) = + =
3 11 3 13
0,62 = 0,368 3
Encerta
1 4 2 4
Tirs P(N) = P(1U N) + P(2U N) = + =
lliures 3 11 3 13
0,52 = 0,326 3
Encerta
0,38
Falla
0,48 43. Dibuixem un diagrama en arbre per esquematitzar les dife-
Falla
rents situacions.

La probabilitat d’encistellar el primer és de 0,62; la de fallar el C1 C2 C3


primer i encertar el segon és de 0,38 0,52 0,197 6; la pro-
babilitat de fallar els dos és de 0,38 0,48 0,182 4. Així, la 0,5
Obert
probabilitat de marcar algun dels tirs és 0,62 0,197 6
0,817 6. Obert
0,5 0,5
Tancat
41. Representem la situació amb un diagrama en arbre: Obert
Llançament 1 Llançament 2 0,5
Obert
0,5 0,5
0,5 Tancat
Cara
0,5 0,5
Tancat
Cara
0,5 Circuit
Creu
Llançaments 0,5
Obert
de moneda
0,5 Obert
Cara
0,5 0,5 0,5 0,5
Creu Tancat
0,5
Creu Tancat
0,5
Obert
La probabilitat de treure cara en el primer llançament i creu 0,5
en el segon és de 0,5 0,5 0,25; aquesta probabilitat és Tancat
idèntica a la de treure creu en el primer llançament i cara en 0,5
Tancat
el segon. En tots dos casos, hi haurà una bola verda i una bola

204
BLOC 4. ESTADÍSTICA I PROBABILITAT > UNITAT 14. PROBABILITAT

a) Cadascun dels camins té una probabilitat de 0,53 0,125.


Com que hi ha 5 casos favorables, P (Sí ) 5 0,125 SÍNTESI Pàg. 352
0,625.
46. {(cara, blanca), (cara, negra), (creu, vermella), (creu,
negra)}
1/3 1
Porta A Or
47. Designem F l’esdeveniment «ser fumador», i PR, l’esdeveni-
1/2 ment «experimentar problemes respiratoris».
1/3 Or
916
Porta B a) P(F ) = 1 P(F ) = 1 = 0,6017
1/2 2 300
Inici Plata
984
b) P(PR) = = 0,427 8
1/3 2 300
Or
732
P(PR F) 2 300
1/3 c) P(PR / F ) = = = 0, 7991
1/3 P(F ) 916
Porta C Plata 2 300
P(PR) = P(PR F ) + P(PR F)
1/3
Res 984 732 252
P(PR F ) = P(PR) P(PR F) = =
2 300 2 300 2 300
P(C1 Sí ) 3 252
b) P(C1 /Sí ) = = 8 = 0,6 P(PR F) 2 300
P(Sí ) 5 P(PR /F ) = = = 0,1821
8 P(F ) 1 384
2 300
44. Representem les diferents opcions en un diagrama en arbre. 48. La probabilitat que les tres siguin negres és
a) La probabilitat d’obtenir una moneda d’or és la suma de 5 4 3 2
les probabilitats dels camins que contenen aquestes mo- P(3N) = =
7 6 5 7
nedes.
La probabilitat que alguna sigui vermella és l’oposat de la pro-
P(Or ) = P(A Or ) + P(B Or ) + P(C Or ) = babilitat que totes siguin negres:
1 1 1 1 1 11
= 1+ + = 2 5
3 3 2 3 3 18 P(V ) = 1 =
7 7
1 1 1 1 5
b) P(Pla.) = P(B Pla.) + P(C Pla.) = + =
3 2 3 3 18 2 1 2
49. P(A B) = ; P(A B) = ; P(B) =
1 1 1 3 9 3
P(B Pla.) = =
3 2 6 2 1
a) P(B) + P(B) = 1 P(B) = 1 P(B) = 1 =
P(B Pla.) 1 3 3
P(B /Pla.) = = 6 = 0,6
P(Pla.) 5 b) P ( A B ) = P ( A ) + P (B ) P (A B)
18
2 1 1 4
P(A) = P(A B) + P(A B) P(B) = + =
3 9 3 9
45. Dibuixem un diagrama en arbre per aclarir la situació.
c) P(A) = P(A B) + P(A B)
Extracció: Primera Segona 4 1 1
P(A B) = P(A) P(A B) = =
5/14 9 9 3
Blanca 1
6/14 P (A B) 9 = 1
Blanca d) P ( A /B ) = =
P (B ) 1 3
9/14 3
Negra
Inici 50. Designem D l’esdeveniment «ser dona», i Dir, «ocupar un lloc
7/14 directiu».
Blanca
8/14 P(D) = 0,6; P(Dir ./D) = 0,05; P(Dir ./D) = 0,2
Negra
7/14
Negra P(Dir . D)
a) P(Dir ./D) =
P(D)
a) P(2B) = P(1N 2B) + P(1B 2B) = P(Dir . D) = P(D) P(Dir ./D) = 0,6 0,05 = 0,03

8 7 6 5 56 + 30 b) P(D) = 1 P(D) = 1 0,6 = 0,4


= + = = 0,438 8
14 14 14 14 196 P(Dir . D)
P(Dir ./D) =
P(2N) = P(1N 2N) + P(1B 2N) = P(M)
8 7 6 9 56 + 54 P(Dir . D) = P(D) P(Dir ./D) = 0,4 0,2 = 0,08
= + = = 0,5612
14 1414 14 196 P(Dir .) = P(Dir . D) + P(Dir . D) = 0,03 + 0,08 = 0,11
P(1N 2N) 56
b) P(1N / 2N) = = 196 = 0,5091 P(D Dir .) 0,03
P(2N) 110 P(D /Dir .) = = = 0,272 7
196 P(Dir .) 0,11

205
BLOC 4. ESTADÍSTICA I PROBABILITAT > UNITAT 14. PROBABILITAT

51. Sigui E una persona que practica esport, i R, una persona que P(2R) = P(2R B) + P(2R P) + P(2R M) =
manté una dieta saludable. = 0,5 0,08 + 0,15 0,25 + 0,35 0,12 = 0,119 5
P(E ) = 0,1; P(R) = 0,26; P(E R) = 0,05
55. En primer lloc, calculem la quantitat de taps que fabrica cada
a) P(E R) = P(E ) + P(R) P(E R) = màquina.
= 0,1 + 0,26 0,05 = 0,31
A
b) P E R +P E R = (P (E ) P (E R )) + =B
( ) ( ) 2 A A 11
A +B +C = A + + = A
+ ( P (R ) P (E R ) ) = ( 0,1 0,05 ) + ( 0,26 0,05 ) = 0,26 A 2 3 6
=C
3
52. L’esdeveniment d’encertar té un cas favorable del total de sis En total, produeixen 11A/6. Si dividim la quantitat de taps que
casos possibles. produeix cada màquina entre aquesta quantitat, tindrem la
1 probabilitat que, en triar un tap a l’atzar, hagi estat fabricat per
P(Encertar ) = = 0,166 7 aquesta màquina.
6
A 6 A A
En el cas del meu germà petit, té una probabilitat d’encert de P(A) = = ; P(B) = 2 = 3 ; P(C) = 3 = 2
cinc casos possibles. 11A 11 11A 11 11A 11
6 6 6
1
P(Germà) = = 0,2 A continuació, dibuixem un diagrama en arbre per esquema-
5
titzar les condicions del problema.

53. Dibuixem un diagrama en arbre per aclarir la situació. Màquina Tipus de peça

Temps Resposta peça 2/150


Defectuosa
0,38 Calor A
148/150
excessiva 6/11 Correcta
Pluja
0,65 3/200
0,62 Calor no Defectuosa
excessiva 3/11
Inici B
Inici 197/200
Correcta
0,53 Calor
excessiva 1/60
0,35 2/11 Defectuosa
No pluja
C
0,47 Calor no 59/60
Correcta
excessiva

P(C) = P(C P) + P(C P) = 0,65 0,38 + 0,35 0,53 = a) P(D) = P(D A) + P(D B) + P(D C) =
= 0,432 5
6 2 3 3 2 1
= + + = 0,014 4
11 150 11 200 11 60
54. Dibuixem un diagrama en arbre per esquematitzar les dife-
rents situacions. b) P(D) = P(D A) + P(D B) + P(D C) =
6 148
3 197 2 59
1 rovell = + + = 0,985 6
11 150
11 200 11 60
Blanc 3 197
0,08 P(D B) = = 0,268 6
0,5 2 rovells
11 200
P(D B) 0,268 6
P(B / D) = = = 0,272 5
1 rovell P(D) 0,985 6
0,15
Inici Pigallat
0,25
2 rovells 56. Designem H l’esdeveniment que l’home visqui als 80 anys, i
D, l’esdeveniment que visqui la dona.
0,35 1 rovell P(H) = 0,76; P(D) = 0,82; P(H D) = 0,62
Marró
0,12 a) P(H D) = P(H) + P(D) P(H D) = 0,96
2 rovells
b) P(H D) = 1 P(H D) = 0,04
La probabilitat que surti amb dos rovells és la suma de les
probabilitats que sigui blanc i amb dos rovells, pigallat i amb c) P(D) = P(H D) + P(H D)
dos rovells i marró i amb dos rovells. P(H D) = P(D) P(H D) = 0,82 0,62 = 0,2

206
BLOC 4. ESTADÍSTICA I PROBABILITAT > UNITAT 14. PROBABILITAT

d) P(H) = P(H D) + P(H D) 2. Confeccionem la taula:


P(H D) = P(H) P(H D) = 0,76 0,62 = 0,14 Resultats daus 1 2 3 4 5 6
1 2 3 4 5 6 7
57. Definim els diferents esdeveniments A, B, C i D.
2 3 4 5 6 7 8
(2,C,C),(4,C,C),(6,C,C), (2,C, +),(4,C, +),(6,C, +)
A= 3 4 5 6 7 8 9
(2, +,C),(4, +,C),(6, +,C)
4 5 6 7 8 9 10
B = {(5, +),(6,C, +),(6, +,C)}
5 6 7 8 9 10 11
C = {(1,C),(1, +),(3,C), (3, +),(5,C),(5, +)} 6 7 8 9 10 11 12
D = {(2,C,C),(4,C,C),(6,C,C), (1, +),(3, +),(5, +)} 7 8 9 10 11 12 13

Per tant: 8 9 10 11 12 13 14
9 10 11 12 13 14 15
A (B D) = {(2,C,C),(4,C,C),(6,C,C),(6,C, +),(6, +,C)}
10 11 12 13 14 15 16
C D = {(1,C),(3,C),(5,C)}
11 12 13 14 15 16 17
D (B C) = {(2,C,C),(4,C,C),(6,C,C)}
12 13 14 15 16 17 18

58. a) En general, no és cert; la suma dóna la probabilitat de A, a) Pel fet de considerar com a esdeveniment el valor de la
que pot ser més petita o igual que 1. suma, no tots tenen la mateixa probabilitat; per exemple, el
resultat 3 té menys probabilitat que el resultat 10.
b) Sí que és cert, la suma és, sabent que B es produeix, la
probabilitat que A s’esdevingui més la que A no ocorri. 10
b) P(X > 14) = = 0,138 9
72
c) En general no és cert; dos esdeveniments són indepen-
dents si la probabilitat que tots dos es produeixin és el
producte de les dues probabilitats. Si assumim que P (A) i 3 1 4
3. P(A) = ; P(B / A) = ; P(A B) =
P (B) són més grans que 0, el producte d’aquestes dues 5 2 5
quantitats no pot donar 0.
2
a) P(A) = 1 P(A) =
d) En general no són independents, ja que la probabilitat que 5
tots dos esdeveniments es produeixin alhora és 0, i no el P(B A) 1
producte de les dues probabilitats com passa amb els es- P(B / A) = P(B A) = P(B / A) P(A) =
P(A) 5
deveniments independents. A més a més, instintivament
podem entendre que la probabilitat d’un esdeveniment no P(A) + P(B) = P(A B) + P(A B)
pot ser independent de la probabilitat del seu complemen- P(B) = P(A B) + P(A B) P(A) =
tari. 4 1 2 3
= + = = 0,6
e) En general no, la igualtat que es compleix és: 5 5 5 5
P(A B) = P(A) + P(B) P(A B) 2
b) P(B) = 1 P(B) =
5
Així, només es compliria si A i B fossin incompatibles.
P(A) = P(A B) + P(A B)
f) Tots els esdeveniments elementals de A estan continguts en 2 1 1
B, de manera que P (B) no pot ser mai més petita que P (A). P(A B) = P(A) P(A B) = =
5 5 5
L’afirmació és certa.
P(A B) = P(A) + P(B) P(A B) =
g) La independència de A i B garanteix que
2 2 1 3
= + = = 0,6
P(A B) = P(A) P(B) 5 5 5 5
Aquesta relació s’ha de complir necessàriament per a un P(A B) 1
subconjunt de A. c) P(A /B) = = 5 = 1 = 0,5
P(B) 2 2
5

Avaluació (pàg. 354) 4. Escrivim les condicions del problema:

1. a) {ccc, cc , c c, cc, c , c . c, } P(2) + P(4) + P(6) = 0,8


b) A {ccc, cc , c c, cc, c , c . c}, P(4) + P(5) + P(6) = 0,75
B {c , c . c, }, C {ccc, c c}. P(5) = 0,2
c) A B = {c + +, + c+, + +c } Restem les dues primeres equacions i, posteriorment, substi-
B C ={ } tuïm el valor de P (5).
A C = {cc+, + cc, c + +, + c+, + +c } P(2) P(5) = 0,05
A (B C) = {ccc, c + c, c + +, + c+, + +c } P(2) = 0,05 + P(5) = 0,25

207
BLOC 4. ESTADÍSTICA I PROBABILITAT > UNITAT 14. PROBABILITAT

1 1 2 45
5. P(A) = ; P(B / A) = ; P(A B) = c) P(B /H) = = 0,090 9
2 3 3 495
P(A B) 8. Designem F l’esdeveniment «fallar un aparell», i E, «cometre
a) P(B / A) =
P(A) un error en prendre les mesures».
1 1 1 P(F ) = 0,05; P(E ) = 0,15; P(F E ) = 0,18
P(A B) = P(B / A) P(A) = =
3 2 6 P(F E ) = P(F ) + P(E ) P(F E) =
P(A) + P(B) = P(A B) + P(A B)
= 0,05 + 0,15 0,18 = 0,02
2 1 1 1
P(B) = P(A B) + P(A B) P(A) = + =
3 6 2 3 9. Designem A l’esdeveniment «plaga d’algues», N, l’esdeveni-
1 1 1 ment «falta de nitrogen», i S, el fet que el peix disc sobrevisqui
P(A) P(B) = = = P(A B)
2 3 6 el primer mes.

Com que la probabilitat de la intersecció de A i B és igual al P(A) = 0,3; P(N) = 0,15; P S ( (A N ) ) = 0,6
producte de la probabilitat de A per la de B, es tracta d’es- P(S / A) = 0,4; P(S /N) = 0,2
deveniments independents.
1 a) Suposant que el fet que hi hagi la plaga d’algues és inde-
b) No són excloents, ja que P(A B) = >0 pendent de la falta de nitrogen, la probabilitat que tots dos
6
es produeixin és de:
c) P (B), com hem calculat en l’apartat a), val 1/3 0,333.
P(A N) = P(A) P(N) = 0,045
d) P(A) = P(A B) + P(A B)
La probabilitat que ocorri A i no N, i viceversa, és determi-
1 1 1
P(A B) = P(A) P(A B) = = nat per:
2 6 3
2 P(A N) = P(A) P(N) = 0,255
P(B) = 1 P(B) =
3 P(A N) = P(A) P(N) = 0,105
P(A B) 1 1
P(A /B) = = 3 = = 0,5 En cas que hi hagi una plaga d’algues i falta de nitrogen,
P(B) 2 2
3 considerarem que la probabilitat de supervivència és el
producte de les probabilitats de cada fenomen, és a dir,
0,2 0,4 0,08. Fem una taula amb els diferents casos
6. Designem P l’esdeveniment «ploure matí», i V, l’esdeveniment
que es poden produir:
«que bufi vent».
P(P) = 0,53; P(V ) = 0,62; P(P V ) = 0,25 A N A N A N A N

a) La probabilitat que no plogui ni faci vent és l’oposat a la Probabilitat


0,595 0,255 0,105 0,045
que ocorri
probabilitat que plogui o faci vent.
Probabilitat de
P(P V ) = 1 P(P V) = 1 0,25 = 0,75 0,6 0,4 0,2 0,08
supervivència
P(P V ) = P(P) + P(V ) P(P V)
P(P V ) = P(P) + P(V ) P(P V) = Així, la probabilitat de supervivència és la suma de les pro-
babilitats que sobrevisqui en cada cas.
= 0,53 + 0,62 0,75 = 0,4
P(S) = 0,595 0,6 + 0,255 0,4 + 0,105 0,2 +
b) La probabilitat que es produeixi només un dels dos esde-
+ 0,045 0,08 = 0,483 6
veniments és la de la unió de tots dos menys la probabilitat
que els dos tinguin lloc. P(N S) 0,105 0,2 + 0,045 0,08
b) P(N /S) = = =
P(S) 0,483 6
P(P V ) + P(P V ) = P(P V ) P(P V) =
= 0,050 9
= 0,75 0,4 = 0,35

10. Designem Ca el fet de venir caminant, B, venir amb bici, i Co,


7. Completem la taula per aclarir la informació de la qual dispo- venir amb cotxe.
sem.
a) P(Ca) = 3P(B) P(B) = P(Co)
Bàsquet Futbol Cap
P(Ca) = 3P(B)
Nois 45 100 350 P(B) = P(Co) P(Ca) = 0,6; P(B) = P(Co) = 0,2
Noies 60 65 280 P(Ca) + P(B) + P(Co) = 1

270 b) P(T ) = P(T Ca) + P(T B) + P(T Co) =


a) P(E ) = = 0,3
900 = 0,2 0,6 + 0,15 0,2 + 0,3 0,2 = 0,21
350 P(B T) 0,2 0,15
b) P(H E) = = 0,388 9 c) P(B /T ) = = = 0,142 8
900 P(T ) 0,21

208
BLOC 4. ESTADÍSTICA I PROBABILITAT > UNITAT 14. PROBABILITAT

Zona (pàg. 355) — La probabilitat de Déu

— Xarxes bayesianes Podem resumir el plantejament de Pascal mitjançant la taula se-


güent:
t Les variables són el tipus d’accident, l’edat del conductor, els
factors atmosfèrics, el sexe del conductor, la il·luminació, el Déu existeix Déu no existeix
nombre de ferits i el nombre de persones implicades en l’ac-
Creure en Déu (cel) 0
cident. Es basa en el teorema de Bayes.
No creure en Déu (infern) 0
t Actualment, hi ha molts camps en els quals s’utilitzen les
xarxes bayesianes, com la intel·ligència artificial, la presa de En canvi, per a Richard Dawkins, també hi havia la possibilitat que, si
decisions, la meteorologia, el reconeixement facial, l’estudi Déu existeix, premiés els qui creuen en ell o no. En la taula següent es
d’infeccions, etcètera. resumeix el seu punt de vista:

— La intuïció hi entén de probabilitats? Déu premia Déu premia Déu no


els creients els ateus existeix
t Probablement, escollirà el nombre 00005 per la seva parti-
cularitat, però els dos nombres són igual de probables. D’en- Creure en Déu ∞ (cel) No especificat 0
trada, no hi ha uns nombres més probables que uns altres, No creure en Déu No especificat ∞ (cel) 0
de manera que atribuir més probabilitat d’èxit a un nombre o
un altre no té cap sentit. t No es tracten d’arguments matemàtics.

209

You might also like